You are on page 1of 71

Solutions for SM1001908

Chapter – 1 589 = 252 – 62 = (19) (31)


(Numbers – Ι) ∴ Choice (B) is not prime.
Choice (C) is divisible by 3.
Concept Review Questions Choice (D) is prime. Choice (D)

Solutions for questions 1 to 80: 6. Twin primes are prime numbers, which differ by 2.
In Choice (A), 133 is divisible by 7 and hence it is not a prime
1. (21) (22) (23) (24) (25) = 2(1 + 2 + 3 + 4 + 5) In Choice (B), the numbers are twin primes.
= 215 = 32768 Choice (C) In Choice (C), 159 is divisible by 3 and hence it is not prime.
Choice (D)
2. (a) The sum of an even number of odd numbers is always
even. Choice (A) 7. Choice (A)
Sum of the digits in the odd places= 32
(b) The product of any number of numbers is odd only if Sum of the digits in the even places = 21
all of them are odd. As the parities of the composite (Sum of the digits in the odd places)
numbers are unknown, we cannot comment on the – (Sum of the digits in the even places) is divisible by 11.
parity of the product. ∴ Choice (A) is divisible by 11.
Choice (C) Choices (B) and (C) are not divisible by 11. Choice (A)
(c) We get different parities for the sum for different sets
8. Sum of the digits of the given number i.e. 28 + X must be
of composite numbers.
divisible by 9.
∴ we cannot say. Choice (C)
∴ X must be 8. Ans: (8)
(d) If one or more of the numbers is/are 2, the product will
be even. Otherwise the product will be odd. As the 9. The given number must have the number formed by its last
parities of the prime numbers are unknown, we cannot 5 digits divisible by 32. The number formed by the last
comment on the parity of the product. Choice (C) 5 digits of the number = 10000 U + 8672.
8672 is divisible by 32. If U is odd, 10000 U is not divisible by
(e) The first prime number is even and the remaining 32. If U is even, 10000 U is divisible by 32.∴ We cannot say.
prime numbers are odd. The sum of N odd numbers Choice (C)
will be even if N is even and will be odd if N is odd. As
there are 9 odd numbers, their sum is odd.∴ The sum 10. The number formed by the last k digits of a number must be
will be odd. Choice (B) divisible by 5k for the number to be divisible by 5k.
∴ The number formed by the last 4 digits of the number i.e.
3. 19019 = 19 (1001) = (19) (13) (11) (7) 9025 must be divisible by 625 for the number to be divisible
∴ 19019 has 4 prime factors. Ans: (4) by 625. As 9025 is not divisible by 625, the number is not
divisible by 625. Choice (B)
4. (a) Let x = 0. 255 = 0.25
11. The difference between any number and the sum of its
10 x = 2.5 -- (1), digits is always divisible by 9. Choice (B)
100x = 25.5 --(2)
Subtracting (1) from (2) 12. As an – a is divisible by 10, i.e. the last digit of an is a.
23 ∴ a is 0, 1, 5 or 6 Choice (A)
x= Choice (A)
90 13. The only composite number n, for which (n – 1)! is not
divisible by n, is 4. Ans: (4)
(b) Let x = 0. 321
10x = 3.21 --(1) 14. One more than the product of any 4 consecutive natural
must be a perfect square. Choice (A)
1000x = 321.21 --(2)
318 53 15. If the odd natural number is more than or equal to
Subtracting (1) from (2), x = = 3 its factorial’s parity would be even
990 165
Choice (A) 1! = 1.∴ 1! is the only odd number satisfying the given
condition. Ans: (1)
(c) Let x = 0.321
16. The product of any N consecutive natural numbers is
100x = 32.1 --(1) divisible by N!, any for all values of N.
1000x = 321.1 --(2) ∴ When N = 7, any such product is divisible by 7! = 5040.
Subtracting (1) from (2), Choice (A)
289
x= Choice (A) 17. 210 × (102) = (210) (2)2 (5)2 = 212 (52)
900 Number of factors of 210 × (102) = (12 + 1) (2 + 1) = 39
Ans: (39)
(d) Let x = 1.116
10x = 11.16 18. A number, which has an even number of factors, is not a
perfect square. It would be a perfect cube only if the index
1000x = 1116 .16 of each of the prime factors is divisible by 3.
1000x – 10x = 1105 Choice (D)
⇒ x = 1105/990 = 221/198
Choice (B) 19. A number of the form (ax) (by) (cz) ….where a, b, c, ….. are
all prime numbers can be written as a product of
5. Choice (A) 2 coprimes in 2n – 1 ways, where n is the number of distinct
851 = 302 – 72 = (23) (37) prime factors.
∴ Choice (A) is not prime As 54 76 has 2 distinct prime factors, it can be written as a
Choice (B) product of 2 co primes in 22 - 1 i.e., in 2 ways. Choice (B)

Triumphant Institute of Management Education Pvt. Ltd. (T.I.M.E.) HO: 95B, 2nd Floor, Siddamsetty Complex, Secunderabad – 500 003.
Tel : 040–27898195 Fax : 040–27847334 email : info@time4education.com website : www.time4education.com SM1001962/1
20. A number of the form (am) (bn) (c)p ….where a, b, c, … are all
prime numbers can be written as a product of 2 distinct numbers (a + b + c)
((a − b)
2
+ (b − c )2 + (a − c )2 ) =0
in
(m + 1) (n + 1) (p + 1) ..... ways if it is not a perfect 2
a + b + c = 0 or a – b = b – c = c – a = 0 i.e. a = b = c or both.
2 Choice (D)
square and in
(m + 1) (n + 1) (p + 1) − 1 ways if it is
2 30. 10.233 − 4.773 = 10.23 – 4.77 = 5.46
a perfect square. 10.232 + 4.772 + (10.23) ( 4.77 )
The given number is a perfect square.  
Q a − b
3 3
∴ It can be written as a product of 2 distinct numbers in = a − b Ans: (5.46)
 a 2 + b 2 + ab 
(4 + 1) (6 + 1) − 1 i.e. 17 ways. Choice (D)
 
2
31. Product of the numbers = (LCM) (HCF)
(30) (Other number) = (120) (6)
21. The sum of the factors of a number of the form am bn cp….
Other number = 24 Choice (C)
where a, b, c, …. are all primes is given by
a m + 1 − 1 b n +1 − 1 c p + 1 − 1 32. If the LCM of two or more numbers equals their product,
. . they must be coprime ∴ HCF (P, R) = 1 Choice (A)
a−1 b−1 c −1
25 − 1 34 − 1 33. L.C.M of any number of mutual co-primes is equal to their
The sum of the factors of 24 × (33)= . product. Choice (A)
2 − 1 3 − 1
= 31 × (40) = 1240 Ans: (1240) 34. LCM (150, 180, 270) = LCM (30 × 5, 30 × 6, 30 × 9)
= 30 LCM (5, 6, 9) = 30 (90) = 2700 Ans: (2700)
22. For any perfect number, the sum of its factors is twice the
number. Choice (B) 35. HCF (63, 42, 105) = HCF (7 × 9, 7 × 6, 7 × 21)
= 7 HCF (9, 6, 21) = 7 (3) = 21 Ans: (21)
23. There are N odd natural numbers less than any even
2 36. (a) LCM of 42,72,90
number N. 2 × 3 × 7, 23 × 32, 2 × 32 × 5
LCM = 2520 (23 × 32 × 7 × 5)
∴ N numbers are coprime to N. HCF = 2 × 3 = 6. Choice (A)
2
24 (b) 810 = 34 × 2 × 5
∴ 2 = 223 numbers are less than 224 and coprime to it. 720 = 24 × 32 × 5
2
Choice (B) LCM = 24 × 34 × 5 = 6480
HCF = 32 × 2 × 5 = 90. Choice (D)
24. The number of numbers less than N and coprime to it (c) 1830 = 61 × 3 × 2 × 5
= N 1 − 1  1 − 1  1 − 1  = 4 N Choice (B) 1098 = 2 × 549 = 2 × 32 × 61
 2  3  5  15 LCM = 61 × 32 × 2 × 5 = 5490
HCF = 61 × 2 × 3 = 366. Choice (C)
25. The sum of all the coprimes of N less than N (d) LCM of numerators = 10
= N (Number of co primes to N less than N) HCF of numerators = 1
2 LCM of denominators = 24
72 = 23 × (32) HCF of denominators = 1
Sum of all co primes of 72 less than 72 LCM of fractions = LCM of num/HCF of den = 10/1 = 10
HCF of fractions = HCF of num/LCM of den =1/24
 
= 72  72 1 − 1  1 − 1   = 864 Ans: (864) Choice (B)
2   2  3  
(e) LCM of numerators = 176
HCF of numerators = 1
1 LCM of denominators = 50
26. (a) b + =4
b HCF of denominators = 5
2 LCM of fractions = LCM of num/HCF of den = 176/5
Squaring both sides, 2 + b2 +  1  = 16 HCF of fractions = HCF of num/LCM of den = 1/50
b Choice (A)
b2 + 1 = 14 Choice (D)
b2  15 25 45 
37. LCM  , , 
 4 6 8 
(b) b –
1
=4 LCM (15, 25, 45 )
=
b HCF (4, 6, 8 )
2
Squaring both sides, – 2 + b2 +  1  = 16 LCM (5 × 3, 5 × 5, 5 × 9 )
=
b HCF (2 × 2, 2 × 3, 2 × 4 )
5 LCM (3, 5, 9 ) 5 (9) (5)
1
b2 + = 18 Choice (B) 225
= = = Choice (B)
b2 2 HCF (2, 3, 4 ) 2 2
27. a3 + b3 = (a + b)3 – 3ab (a + b) 38. Time to toll together again = LCM of 5, 6, 10, 12, 15
= 27000 – 3(176) (30) = 11160 Choice (C) = 60 seconds. Ans: (60)
28. If a + b + c = 0, a3 + b3 + c3 = 3abc Choice (C) 39. Let the side of the smallest square be S cm.
S = 8a = 6b where a and b are natural numbers.
29. a3 b3 c3
+ + = 3abc S = LCM (8a, 6b) = 24 LCM (a, b)
a3 + b3 + c3 – 3abc = 0 Which is minimum when LCM (a, b) = 1
(a + b + c) (a2 + b2 + c2 – ab – bc – ca) = 0 ∴ Required area = 242 = 576. Ans: (576)

Triumphant Institute of Management Education Pvt. Ltd. (T.I.M.E.) HO: 95B, 2nd Floor, Siddamsetty Complex, Secunderabad – 500 003.
Tel : 040–27898195 Fax : 040–27847334 email : info@time4education.com website : www.time4education.com SM1001962/2
40. Two numbers whose HCF equals their LCM must be equal. (b) General value of N is (8) (6) (4) k + 165
Choice (C) i.e. 192 k + 165, where k = 0,1,2 -----
Hence the tenth number in the sequence is 192(9) + 165
41. The number formed by the last three digits of a number = 1893 Choice (A)
must be divisible by 8 for the number to be divisible by 8.
The least natural number which should be added to the 56. Number of zeros at the end of 150! = Index of the highest
number formed by the last 3 digits of the given number to power of 5 in 150!
make it divisible by 8 is 3. Ans: (3) 30 numbers have 5 in them, 6 have 52 in them and one
has 53 in it
42. Any prime number greater than 3 must be in the form ∴ Index of the greatest power of 5 in 150! = 30 + 6 + 1 = 37
6 (A natural number) ± 1. Ans: (37)
∴ mk must be divisible by 6.
Choice (D) 57. 256! has 128(21s) + 64(22s) + 32(23s) + (16 24s), 8(25s)
+ 4(26s) + 2(27s) + 1(28s).
43. The greatest number which divides the product of any ∴ The greatest power of 2 that divides 256! is 2255.
10 even numbers is 210. Choice (B) Note: The index of the greatest power of 2 that divides 2n!
is 2n – 1. Choice (B)
44. (a) To obtain a perfect square, the index of each of the
prime factor must be even.
58. (a) For a number to be divisible by 2, the number must be
∴ Least natural number = (3) (5) = 15
even. Also the divisibility rule of 9 is sum of the digits
Choice (B)
of the number should be a multiple of 9. Satisfying
(b) Least natural number = 3. Choice (C) both the conditions, the number is 4032
Choice (C)
45. Least natural number = (Least perfect cube greater than (b) For a number to be divisible by the numbers
599) – 599 = 729 – 599 = 130. Choice (C) 2, 3, 4, 6, 8, 9 it is enough that the number is divisible
by 8 and 9. Divisibility rule of 8: last three digits of the
46. Given 1764 is a perfect cube
given number is a multiple of 8
i.e. (49) (36) k is a perfect cube ⇒ 72 (62 ) k is a perfect cube Divisibility rule of 9: Sum of the digits of the number is
∴ Least value of k is (6) (7) = 42 Ans: (42) a multiple of 9.
Satisfying both the conditions the number is 4608
47. N = (24) (33) (73) K is a perfect square and a perfect cube.
Choice (B)
Hence the index of each of its prime factors must be
divisible by 6 (c) For a number to be divisible by 3, 8 and 12, it is
∴ Least value of k is 22 (34) (73) enough to check the divisibility of 3 and 8.
∴ Total number of factors of k is (3) (5) (4) = 60 Divisibility rule of 3: sum of the digits of the number is
Choice (D) a multiple of 3.
Divisibility rule of 8 is known in the earlier problem.
48. Any perfect square having its last 2 digits equal must end Satisfying the conditions, the number is 4248
with a 4. 144 is an example of such a number. Choice (A)
Choice (B)
(d) For a number to be divisible by 2, 4, 8 and 11 it is
49. The least natural number = LCM (7, 8) + 2 = 58 enough to check the divisibility of the number by 8 and
Choice (C) 11. Satisfying the conditions, the number is 4752
Choice (C)
50. The least natural number = LCM (18, 24) – 7 = 65
Ans: (65) (e) For a number to be divisible by 2, 3, 9, 5 and 10 check
the divisibility of the number by 9 and 10. Satisfying
51. The general form of the numbers leaving remainders of 6 the conditions, the number is 3780. Choice (B)
and 8 when divided by 7 and 11 respectively are 7 K1 + 6
and 11 K2 + 8 where K1 and K2 are natural numbers. (f) 24 = 8 × 3

7K1 + 6 = 11 K2 + 8 K1 = K2 + 2 (2 K 2 + 1)
For a number to be divisible by 24, check the
divisibility of the numbers by 8 and 3.
7
The number satisfying the conditions is 3384.
The least value of K2 satisfying the condition that K1 is Choice (B)
a natural number is 3.
∴ The least natural number = 11 (3) + 8 = 41 (g) 22 = 11 × 2
Choice (C) 33 = 11 × 3
For a number to be divisible by both 22 and 33 check
52. The largest number = HCF (127 – 7, 156 – 6) = 30 the divisibility of the number by 2, 3 and 11.
Ans: (30)
Divisibility rule of 11:
53. 349247 is odd, Take the sum of the alternate digits starting from units
∴ All the powers of 2 are co-prime to it. There are an digit i.e., units sum also the sum and of the alternate
infinite number of powers of 2. digits starting from ten’s digit i.e. ten’s sum.
An infinite number of positive integers are co-prime to it. The difference of units sum and tens sum must be 0 or
Choice (D) a multiple of 11.
The number divisible by 2, 3 and 11 is 4356.
54. Four-digit numbers divisible by 5, 12 and 18 are divisible by Choice (B)
LCM (5, 12, 18) i.e. 180. They are of the form 180 K where
K is a natural number. (h) 36 = 9 × 4; 24 = 8 × 3 Check out the divisibility of the
1000 < 180 K < 10000 number of 8 and 9. The number satisfying the
condition is 6336.
5 5
5 < K < 55 i.e., K = 6, 7,………….55 Choice (D)
9 9
∴ K has 50 possible values. Ans: (50) (i) 40 = 5 × 8; 72 = 8 × 9 its enough to check out the
divisibility of the number by 5, 8 and 9
55. (a) Least number = [3(6) +2] (8) + 5 =165 ∴ the number divisible is 7560.
Choice (D) Choice (A)

Triumphant Institute of Management Education Pvt. Ltd. (T.I.M.E.) HO: 95B, 2nd Floor, Siddamsetty Complex, Secunderabad – 500 003.
Tel : 040–27898195 Fax : 040–27847334 email : info@time4education.com website : www.time4education.com SM1001962/3
59. For a number to be a multiple of 9, the sum of the digits must (ii) (a) 77 × 335 + 37 × 665 – 40 × 335
be a multiple of 9. If the sum of the digits is not a multiple of = (77 – 40) 335 + 37 × 665 = 37 × 335 + 37 × 665
9, add the required number to make it a multiple of 9 = 37(335 + 665) = 37000 Choice (D)
(a) 3 + 2 + 0 + 5 + 2 + 6 = 18 (b) 27/12 + 3¼ – 1½ + 2 1/6 – 21/3= 4 2/12 = 41/6
As the number is already a multiple of 9, answer is 0 Choice (B)
Choice (C)
(b) 2 + 4 + 3 + 1 + 2 = 12 (c) (3.13)2 + (4.25)2 + (2.62)2 + 6.26 × 4.25 + 8.5 × 2.62 +
6 is to be added to make it a multiple of 9 5.24 × 3.13
Choice (A) = (3.13)2 + (4.25)2 + (2.62)2 + 2 × 3.13 × 4.25 + 2 ×
4.25 × 2.62 + 2 × 2.62 × 3.13 = (3.13 + 4.25 + 2.62)2
(c) 2 + 3 + 4 + 7 + 9 + 0 + 4 = 29
[i.e., a2 + b2 + c2 + 2ab + 2bc + 2ca = (a + b + c)2]
7 is to be added to make it a multiple of 9
= (10)2 = 100 Choice (B)
Choice (B)
(d) 7 + 8 + 9 + 4 + 5 + 7 = 40 63. (a) 1 1 71 61 131
5 is to be added Choice (D) 1
(e) 1 + 2 + 3 + 4 + 5 + 6 + 7 + 8 + 9 = 45 ---------
0 is to be added Choice (D) 23 71
69
60. (a) 243741 ---------
units sum = 12 261 261
tens sum = 9 261
difference = 3 ---------
as units sum > tens sum, add (11 –difference) to the 0
number to make it a multiple of 11 -------
i.e., 11 – 3 = 8 Choice (B) ∴ 17161 = 131 Choice (C)
(b) 321423
units sum = 9 (b) 7 5929 77
tens sum = 6 49
units sum > tens sum, add(11 –difference ) ---------
i.e., 11 – 3 = 8 Choice (D) 147 1029
(c) 243081 1029
units sum = 5 ---------
tens sum = 13 0
difference = 8 ---------
units sum < tens sum, add difference i.e., 8 5929 = 77 Choice (A)
Choice (A)
(d) 723111 (c) 1 24964 158
units sum = 4 1
tens sum = 11 -----
difference = 7 25 149
units sum < tens sum, add difference i.e., 7 125
Choice (C) ---------
(e) 123456789 308 2464
units sum = 25 2464
tens sum = 20 ---------
difference = 5 0
units sum > tens sum, add (11 – difference) ---------
i.e., 11 – 5 = 6. Choice (B)
24964 = 158 Choice (A)
61. (a) 9000 = 9 × 1000
= 32 × 103 = 23 × 32 × 53 Choice (C)
(b) 1936 = 11 × 176 = 11 × 11 × 16 (d) 5 2809 53
25
= 11² × 24 Choice (C)
---------
(c) 3969 = 9 × 441 103 309
= 3² × 3² × 7² = 34 × 7² Choice (C) 309
(d) 14553 = 11 × 1323 --------
0
= 11 × 3 × 21² = 11 × 33 × 72 Choice (C)
--------
62. (i) (a) 248 × 555 +148 × 445 2809 = 53 Choice (D)
= (100+ 148) 555 + 148 × 445
= 100 × 555 + 148 (555 + 445) (e) 1 231.04 15.2
= 203500 Choice (A) 1
-----
(b) 4 1/2 + 3 1/5 – 2 1/10 – 4 1/20 = 111/20
25 131
Choice (B)
125
---------
(3.37)3 + 3 × 3.37 (6.63)2 + 3 × 6.63 (3.37)2 + (6.63)3 30.2 6.04
(c)
(3.37)2 + 2 × 3.37 × 6.63 + (6.63)2 6.04
(3.37 + 6.63)3 --------
0
=
(3.37 + 6.63)2 = 3.37 + 6.63 = 10
--------
Choice (C) 231.04 = 15.2 Choice (D)
nd
Triumphant Institute of Management Education Pvt. Ltd. (T.I.M.E.) HO: 95B, 2 Floor, Siddamsetty Complex, Secunderabad – 500 003.
Tel : 040–27898195 Fax : 040–27847334 email : info@time4education.com website : www.time4education.com SM1001962/4
(f) 1 1 76 89 133 74. LCM of 38 and 57 = 114
1 Remainder when 1994 is divided by 114 is 56. Number to
------ be added to 1994 to make it a multiple of 114 is 58. in order
23 76 to leave a remainder of 28, the number to be added is
69 58 + 28 = 86 Ans: (86)
---------
263 789 75. p = 23 . 32. 72. 116
789 q = 22. 31. 54. 112. 132
-------- The common prime factors of p and q are 2, 3 and 11.
0 ∴ GCD (p, q) must have only these prime factors.
-------- ∴ GCD (p, q) = 2 min (3, 2). 3min (2, 1). 11min (6, 2)
= 22. 31. 112 = (4) (3) (121) = 1452
∴ 17689 = 133. Choice (C) Choice (B)

64. Two numbers having no common factor except one are 76. Least positive integer divisible by 22.3.5,3.52.7 and 5.7.112
called as relative primes. Among the options the pairs is their L.C.M. = 22. 3.52. 7. 112.
which are relative primes are 57,61; 396,455; and 6561, ∴ Its distinct prime factors are 2, 3, 5, 7 and 11.
1024 Choice (B) Number of distinct prime factors = 5. Ans: (5)

65. For any two numbers, LCM × GCD = Product of the two Solutions for questions 77 and 78:
numbers.
∴ (432) (18) = (54) (other number) When finding the LCM/HCF of 2 or more numbers, each number
Other number =
(432 )(18 ) = 144 must be involved in the LCM/HCF functions at least once.
54
Ans: (144) 77. Choice (A)

66. Least number satisfying the given conditions is 9. 78. Choice (A)
Ans: (9)
N (N + 1)
79. Sum of the first N natural numbers =
67. LCM of 22,33,55 = 2 × 3 × 5 × 11 = 330 2
Smallest three digit number = 330 + 5 = 335 N (N + 1) N (N + 1)
Largest three digit number = 330 × 3 + 5 = 995 = = x2 ⇒ is a perfect square.
2 2
Choice (B) Let us go by the choices.
N (N + 1)
68. As the difference between the divisors and the remainder is Choice (A): When N = 1, = 1 which is a perfect square
same in both the cases 2
Smallest number = LCM of divisors – (common difference
of divisors and remainders) N (N + 1)
When N = 9, = 45 which is not a perfect square.
= LCM of (8,12) – 5 = 24 – 5 =19. 2
Choice (D) ∴ Choice (A) is ruled out.
N (N + 1)
69. Two different dividends and two remainders, Choice (B): When N = 1, is a perfect square.
∴ take HCF of difference of dividends and remainders 2
6850 – 50 = 6800 N (N + 1)
When N = 7, = 28 which is not a perfect square.
2575 – 25 = 2550 2
6800 = 17 × 24 × 52 ∴ Choice (B) is ruled out.
2550 = 17 × 150 = 17 × 2 × 3 × 52 N (N + 1)
HCF = 17 × 2 × 52 = 850 Choice (C): When N = 1, which is a perfect square.
2
∴HCF = 850 is the greatest number
Choice (B) N (N + 1) 8 (9 )
When N = 8, = = 36 which is a perfect square
2 2
70. Three dividends and the same remainder (not mentioned)
then take difference of two pairs of dividends and find the HCF N (N + 1)
When N = 48, = (24) (49) which is not a perfect square
134 – 96= 38 2
229 – 134 = 95 ∴ Choice (C) is ruled out.
HCF of 38, 95 = 19 N (N + 1)
∴HCF = 19 Ans: (19) Choice (D): When N = 1 or 8, is a perfect square.
2
71. Take difference of two pairs of dividends and find their HCF (proved above)
140 – 68 = 72 N (N + 1)
248 – 140 = 108 When N = 49, = (49), (25) which is a perfect square
2
HCF of 72, 108
Choice (D) follows. Choice (D)
36 × 2 = 72
36 × 3 = 108
HCF = 36. Choice (A) 80. If x = 2 and y = 3 ,

72. The largest number = HCF (218 – 146, 434 – 218) x + y – xy = 2 + 3 – 6


= HCF (72, 216) = 72 In this case, x + y – xy is irrational.
Choice (B) If x = 2 and y = – 2,
73. Take HCF of difference of dividers and remainders. x + y – xy = 2 + ( − 2 ) – ( 2 ) (– 2 ) = 2
HCF of 3300 – 23 = 3277 and 3640 – 24 = 3616 In this case, x + y – xy is rational.
3616 = 25 × 113
∴ We can only conclude that x + y – xy is real
Checking divisibility of 3277 by 113
3277 = 29 × 113 (∵ Any real number is one which is either rational or
HCF = 113 Ans: (113) irrational) Choice (A)

Triumphant Institute of Management Education Pvt. Ltd. (T.I.M.E.) HO: 95B, 2nd Floor, Siddamsetty Complex, Secunderabad – 500 003.
Tel : 040–27898195 Fax : 040–27847334 email : info@time4education.com website : www.time4education.com SM1001962/5
Exercise – 1(a) When the divisor = 7, remainder = 3 and when the divisor
= 5, remainder is 1. The number N leaves a remainder of
Solutions for questions 1 to 40: 5 when divided by 6. Hence N = 6a + 5 where a is the
quotient when N is divided by 6
1. In the number 315642, the 1st, 3rd and 5th digits. (i.e., 3, N = 6a + 5 = 35 k – 4 ⇒ 6a + 5 + 4 = 35k
5 and 4 respectively) are termed being 3, 5 and 4 and 6a + 9 = 35k –––––– (1)
would be termed the odd digits. The 2nd, 4th and 6th digits When we put various integer values of a starting from 1, in
(i.e., 1, 6 and 2 respectively) are termed the even digits. (1) and see where we get k as an integer, we see that only
Sum of odd digits = 3 + 5 + 4 = 12. when a is at least 16, k assumes an integer value of 3 for
Sum of even digits = 1 + 6 + 2 = 9 the first time.
Hence the sum of the odd digits – the sum of the even digits Hence, N = 35 (3) – 4 = 105 – 4 = 101 Choice (A)
= 12 – 9 = 3
Hence if 3 is added to the number, sum of the even digits 9. LCM of 9 and 11 is 99. When the smallest four-digit number
= 9 + 3 = 12 = sum of the odd digits, thereby the number 1000 is divided by 99, we have the remainder as 10.
formed becomes divisible by 11. Ans: (3) Hence, 1000 – 10 = 990 is divisible by 99
Thus the smallest four-digit number which is divisible by
2. The given number is 5668x25y and this is divisible by 48. 9 and 11 is 990 + 99 = 1089
⇒ The number is divisible by 8, 16 and 3. The smallest four-digit number which when divided by
⇒ 25y is divisible by 8; ⇒ y = 6. 9 leaves a remainder 5 and when divided by 11 leaves a
Because 16 is a factor, x25y is divisible by 16. remainder 7 = 1089 – C, where C is the common difference
But y = 6; i.e., x256 is divisible by 16. Whenever, x has any between the divisor and the remainder in both the cases.
of the even numbers 2, 4, 6 or 8 as its value, x256 is C = 9 – 5 = 4 or C = 11 – 7 = 4
divisible by 16. ⇒ x = 2 or 4 or 6 or 8. Hence the smallest four-digit number required
Sum of the digits of the number = (38 + x). For this to be = 1089 – 4 = 1085 Choice (D)
divisible by 3, x shall be 4, (from among the above
10. LCM of 7, 9 and 11 = 7 x 9 x 11 = 63 x 11 = 693.
4 alternatives). x + y = 4 + 6 = 10. Choice (A)
Dividing the largest 4-digit number 9999 by 693 we get a
remainder of 297. Subtracting 297 form 9999, we have
3. Let the even natural numbers be 2k, 2k + 2, 2k + 4 and
9999 – 297 = 9702 which is exactly divisible by 693 and
2k + 6.
hence by 7, 9 and 11. The largest 4-digit number which when
N = 16 + (2k) (2k + 2) (2k + 4) (2k + 6)
divided by 7, 9 and 11 leaves a remainder of 5 in each case
= 16(1 + k(k + 1) (k + 2) (k + 3))
= 9702 + 5 = 9707 Choice (C)
= 16(1 + k(k + 3) (k + 1) (k + 2))
= 16(1 + (k2 + 3k) (k2 + 3k + 2) 11. If the soldiers are arranged in rows of 8 or 15 or 20,
= 16(1 + (k2 + 3k)2 + 2(k2 + 3k)) one soldier is left to stand alone in the last row. Hence if the
= 16(k2 + 3k + 1)2 total number of soldiers is divided by 8 or 15 or 20, the
k2 + 3k + 1 is odd for any positive integral value of k. remainder will be 1. Similarly, if the total number of soldiers
k ⇒ (k2 + 3k + 1)2 is also odd. is divided by 9 or 13, the remainder will be 4. An option
∴ 16(k2 + 3k + 1)2 is a perfect square divisible by 16. satisfying both these conditions is only.
Hence only (B) and (D) are true. Choice (C) Alternate method:
Number of soldiers on the field = LCM (8, 15, 20)c + 1
4. Let the four prime numbers be a, b, c and d. = (120c + 1), where c is a constant
Given a × b × c = 2431 and b × c × d = 4199 Number of soldiers on the field = LCM (9, 13)k + 4
a ×b × c 2431 a 11 = 117k + 4 where k is a constant.
∴ = ⇒ = ∴ d = 19 Ans: (19) Hence 120c + 1 = 117k + 4.
b × c × d 4199 d 19
The above equation is satisfied when k = 1 for c = 1
Thus number of soldiers in the field = 1(120) + 1 = 121
5. Prime numbers less than 5 are 2 and 3. Choice (D)
23 and 33 leave respective remainders of 2 and 3 when
divided by 6. 12. Let the number of sweets with Rohan we N.
Prime numbers greater than or equal to 5 are of the form N = K1 LCM(12, 16, 18) + 1 = 17 K2, where K1 and K2 are
6k ± 1 where k is a natural number. natural numbers.
(6k + 1)3 leaves a remainder of 1 when divided by 6. 144K1 + 1 = 17K2
(6k − 1)3 leaves a remainder of 5 when divided by 6.
8K1 + 8K1 + 1 = K 2
∴ Sum of all the distinct possible remainders is 11. 17
Choice (B) The least value of K2 (17) is realized when K1 = 2.
∴ The least value of N = 289.
6. The required divisor is obtained by considering the HCF of N must be of the form 289 + K LCM (144, 17) = 2448K +
(698 – 9, 450 – 8) 289, where K is a whole number.
HCF of (689, 442) is 13. Choice (B)
∴ 2448K + 289 < 10000
∴ K can be 0, 1, 2 or 3 i.e., it has 4 possible values.
7. Let the number be N.
Choice (C)
68488 = N.K1 + R and
67516 = NK2 + R where K1 and K2 are natural numbers and 13. If side of each identical square tile is x, then the number of
R is the remainder 68488 − 67515 = N(K1 − K2). tiles required
972 = N(K1 − K2) Area of the floor 870 cm × 638 cm
N must be a factor of 972. = =
972 = 1 × 972 = 2 × 480 Area of each square tile x× x
= 3 × 324 The number of identical square tiles will be minimum if area
= 4 × 243 = 6 × 162 = 9 × 108 of each identical square tile (x2) is maximum. Also, to
∴ N has 6 possibilities. Ans: (6) completely each of the cover the floor, the side of the tile
should be a factor of the dimensions of the room.
8. Let the smallest three-digit number required be N. N leaves ⇒ x must be the HCF of 870 and 638
a remainder of 3 when divided by 7 and when divided by ∴x = 58
5 leaves a remainder of 1. Hence N is of the form k[LCM Hence minimum number of identical square tiles
(7, 5)] – 4= 35k – 4 where k is a positive integer and 4 is 870 cm x 638 cm
= = 165 Ans: (165)
the difference between the divisor and the remainder. 58 cm x 58 cm

Triumphant Institute of Management Education Pvt. Ltd. (T.I.M.E.) HO: 95B, 2nd Floor, Siddamsetty Complex, Secunderabad – 500 003.
Tel : 040–27898195 Fax : 040–27847334 email : info@time4education.com website : www.time4education.com SM1001962/6
14. Weight of each piece (in kg) 21. N = 68 86 = 226 38. ∴ N has 243 factors.

( )
 1 1 3 15  243
= HCF  6 , 10 , 8 , 3 
 8 2 4 16  The product of all these factors of N is N243/2 = 6 8 8 6 2
= (64 83)243. Choice (C)
= HCF  49 , 21 , 35 , 63  = HCF ( 44, 21, 35, 63 ) = 7
 8 2 4 16  LCM (8, 2, 4, 16 ) 16 22. X2 − 8X = (Y2 + 2Y)2 − 8(Y2 + 2Y) = (Y2 + 2Y) (Y2 + 2Y − 8)
49 21 35 63 = Y(Y + 2) (Y + 4) (Y − 2)
+ + + Let Y = 2a, where a is a natural number.
Number of pieces obtained = 8 2 4 16 = 67
7 ∴ X2 − 8X = 2a(2a + 2) (2a + 4) (2a − 2)
= 16(a − 2) (a − 1) a(a + 2)
16
= 16 (Product of 4 consecutive natural numbers).
Ans: (67) The product of 4 consecutive natural numbers is always
divisible by 24.
15. The two numbers are of the form 6x and 6y since the HCF of
∴ X2 − 8X is always divisible by 384 but not always
the two numbers is 6, where x and y are co-primes.
divisible by 384(2) or 768. Eg. when a = 1, X2−8X = 384,
(6x) (6y) = 4320 36xy = 4320
which is not divisible by 768.
xy = 120
Choice (C)
Now 120 = 23 × 31 × 51
∴Number of possible pairs = 2n – 1 where n is the number of 23. Number of three-digit numbers = 900
distinct prime factors of 120, Number of three-digit numbers which are divisible neither
∴Number of pairs = 23 – 1 = 22 = 4 Choice (C) by 2 nor by 3 = 900 × (1 – 1/2) × (1 – 1/3)
= 900 × 1/2 × 2/3 = 300 Ans: (300)
16. Since the HCF of the two numbers is 7, we have 7x and 7y
as the two numbers where x and y are co-primes. 7x – 7y
24. Let sum of all co-primes to 2016, which are less than 2016 = S
= 7(x – y) = 21, x – y = 21/7 = 3, x = y + 3
2016 = 2 x 1008 = 2 x 2 x 504 = 2 x 2 x 2 x 7 x 36
The LCM of the two numbers is 7xy = 196
= 23 x 7 x 32 x 22 = 25 x 7 x 32
xy = 196/7 = 28 ⇒ (y + 3)y = 28 ⇒ y2 + 3y = 28
2016
y2 + 3y – 28 = 0 ⇒ (y + 7) (y – 4) = 0 Hence S = x 2016 x (1 – 1/2) x (1 – 1/7) (1 – 1/3)
Since y can’t be negative, y = 4 2
x = 28/y = 28/4 = 7 = 2016/2 x 2016 x 1/2 x 6/7 x 2/3 = 580608 Choice (D)
Hence the larger of the two numbers is 7x = 7 x 7 = 49
25. 24 = 23 × 3. The largest power of 3 contained in 360! can
Alternate method: be calculated by the method indicated below.
Going by the options, option (A) says the larger number is
3 360 = 120
28. The smaller number would then be 28 – 21 = 7. LCM of
3 120 = 40
28 and 7 is 28. Option (B) says the larger number is 35.
3 40 = 13
Since 196 is not a multiple of 35, option (B) is ruled out.
3 13 =4
Option (C) says the larger number is 42; smaller number
3 4 =1 Total = 178
would then be 42 – 21 = 21
LCM of 42 and 21 is 42. Hence not possible. Hence the largest power of 3 in 360! is 178.
Option (D) says the larger number is 49. Smaller number Similarly we can calculate the largest power of 2 in 360!, by
would then be 49 – 21 = 28 the method indicated below.
LCM of 49 and 28 is 196 Choice (D)
2 360 = 180
17. 11111111 = 11(1010101) = 11(101) (10001) 2 180 = 90
= 11(101) (11025 −1024) = 11(101) (1052 − 322) 2 90 = 45
= 11(101) (137) (73) 2 45 = 22
1012 − 1 137 2 − 1 2 22 = 11
Sum of all the factors = 11 − 1 .
2
. . 2 11 =5
11 − 1 101 − 1 137 − 1
2 5 =2
732 − 1 2 2 =1 Total = 356
= 12 (102) (138) (74) = 12499488
73 − 1
Hence the largest power of 2 in 360! is 356. The largest
Ans: (12499488) power of 23 in 360! is the quotient of 356/3 = 118.
Hence the largest power of 23 x 3 in 360! is 118 which is
18. Expressing 152100 as product of prime factors, we have the largest common power of (23 × 3) contained in 360!
152100 = 22 × 32 × 52 × 132 Choice (B)
Number of ways in which 152100 can be expressed as
a product of two different factors 26. Suppose, a = 3, b = 4, c = 5 and d = 2
= 1/2 [(2 + 1)(2 + 1)(2 + 1)(2 + 1) – 1] Then they satisfy ad + bd = cd
= 1/2 [81 – 1] = 1/2 [80] = 40 Choice (D) In this case, the minimum of a, b and c is at least d.
Choice (A)
19. Total number of factors of N = 45.
If N = ap, bq, cr ….., then, (p + 1) (q + 1) (r + 1) …. = 45
27. Given divisors are 8, 6 and 5 and their respective
45 can be written as 3 × 3 × 5, if the number of factors of 45 remainders are 1, 1 and 2.
is to be maximum.
∴The number is of the form of (8 × 6 × 5)k + (8 × (2 × 6 +
⇒ (p + 1) (q + 1) (r + 1) ….. = 3 × 3 × 5; 1) + 1) = 240k + 105
⇒ N has a maximum of 3 distinct prime factors with As the HCF of 240 and 105 is 15, the number is divisible by
respective powers p, q and r. both 3 and 5. Choice (D)
Maximum number of distinct prime factors of N = 3.
Ans: (3) 28. Let the number missed be x.
The correct sum = 800 + x.
N(N + 1)
20. N = 10! = 2(3)(22)(5) 2(3) 7(23) (32) 2(5) = 28345271
∴ 10! has 9(5)(3)(2) or 270 factors This must be in the form =
The product of all these factors is (10!)135. 2
Choice (A) where N is a natural number.

Triumphant Institute of Management Education Pvt. Ltd. (T.I.M.E.) HO: 95B, 2nd Floor, Siddamsetty Complex, Secunderabad – 500 003.
Tel : 040–27898195 Fax : 040–27847334 email : info@time4education.com website : www.time4education.com SM1001962/7
N(N + 1) 83 623 14323
When N = 39, = 780 ∴ 0.754 + 0.692 = + =
2 110 900 9900
( )
When N = 40, N N + 1 = 820
Choice (C)
2
N(N + 1)
35. Let N = a2 − b2 = (a − b) (a + b)
When N = 41, = 861 If a − b and a + b are of opposite parity, a and b will not be
2 natural numbers.
As 800 + x > 800, N ≥ 40 If N = 40, x = 20 ∴ For a and b to be natural numbers, a − b and a + b must
For every increase in N by 1. x will increase by N + 1. be positive.
∴ If N ≥ 41, x > N. This is not possible. ∴ K(N) represents the number of ways of expressing N in
∴ N = 40 and x = 20 Ans: (20) the form (a − b) (a + b), where a − b and a + b are positive.
The numbers in the 5 choices (N), their prime factors, and
29. N7 − N = N (N6 − 1) = N((N3)2 − 1) = N(N3 − 1) (N3 + 1) k(N) are tabulated below.
N(N − 1) (N2 + N + 1) (N + 1) (N2 − N + 1)
= N(N − 1) (N + 1) (N2 + N + 1) (N2 − N + 1) N Prime factors K(N)
N − 1, N and N + 1 are consecutive integers. The product of 110 2(5) (11) 0
any three consecutive integers is divisible by 6. 105 (3)(5)(7) 4
∴ N7 − N is divisible by 6. 216 23 33 4
∴ the remainder is 0. Choice (A) 384 27 3 6
450 213252 0
30. There are 9 single digits pages. Number of digits required If there is only one 2 in N, K(N) = 0
to number single digit pages = 9. If there is no.2, K(N) = number of ways of expressing N as
There are 90 two digit pages. Number of digits required to a product of two factors.
number two digit page s= 180. If there is more than one 2, (say if these are 2m or 2m+1)
There are 900 there digit pages. 2’s), the 2’s can be split in m ways. If there are n of some
Number of digits required to number three digit pages other prime factors, those factors can be split it (n+1) ways.
= 2700. ∴ For N = 216 = 2333, the 2’s can be split as 2, 22, i.e., in
Number of four digit pages = 501. 1 way. The 3 threes can be split as 0.3; 1.2; 2.1 or 3.0,
Number of digits required to number four digits pages i.e., in 4 ways.
= 2004.
∴ K(N) = 4.
∴ Total number of keys to be pressed For N = 384 = 27(3), the 2’s can be split as 2, 26; 22, 25 or
= 9 + 180 + 2700 + 2004 = 4893. 23, 24 . For each of these split, the one 3 can go with either part.
Ans: (4893)
∴ k(N) = 3 (N) = 6. This is the maximum. Choice (D)
31. X is a set of integers whose elements when arranged in
36. Let S be the sum of the 62 page numbers. The sum of the
ascending order form an arithmetic progression whose
first 62 numbers is 31(63) = 1953.(i.e., in this case
first term is 9 and common difference is 6. Let us say it has
S = 1953). If first leaf is left intact and instead leaf
n elements.
32 (comprising pages 63, 64) is torn off, S would be
375 = 9 + (n − 1)6 ; 62 = n 1953 – (1 + 2) + (63 + 64) or 1953 + 124. If the leaves torn
In an arithmetic progression with even number of terms off are 3 to 33, S would be 1953 + 2(124). In general,
(say n), the sum of the kth term, from the start and kth term S would be of the form 1953 + 124n, i.e, 1953, 2077, 2201,
from the end will be the same. ………..Among the choices, only 2201 is of this form.
∴ Maximum number of elements of y will occur when Choice (B)
y = {9, 15, 21, . . . . 189}
or v = {195, 201, 207, . . ., 375}. 37. Consider an n-digit number.
In either case, y has 31 elements. Choice (C) Consider n = 3 and 4
Let P = abc = 100a + 10b + c
32. Given w + x + y + z = 8 m + 10. The reverse Q = cba = 100c + 10b + a
In m = 1, x + y + z = 6m + 10 = 16 ∴ P – Q = 99(a – c)
When the sum of three natural numbers is constant, the Let P = abcd = 1000a + 100b + 10c + d
sum of their squares is minimum when the numbers are as The reverse Q = dcba = 1000d + 100c + 10b + a
close as possible. ∴ P – Q = 999(a – d) + 90(b – c)
So the four numbers must be 2m + 2, 2m + 2, 2m + 3 and For n = 3, P – Q is divisible by both 9 and 11
2m + 3. For n = 4, P – Q is divisible by 9 but not necessarily by 11.
∴ the minimum value of w2 + x2 + y2 + z2 = (2m + 2)2 + In general when n is odd, the difference is divisible by 9
(2m + 2)2 + (2m + 3)2 + (2m + 3)2 = 16m2 + 40m + 26 and 11. When n is even, the difference is divisible by 9 but
Choice (D) need not be divisible by 11. In the given problem, N is 40,
an even number. ∴ The difference must be divisible by 9
33. If x is of the form 6k + 1 where k is a natural number, none but not necessarily by 11. Choice (A)
of the elements of Xx will be divisible by 6. If x is of the form
6k or 6k + 2 or 6k + 3 or 6k + 4 or 6k + 5, one of the 38. The largest power of 2 in 32! is 31
elements of Xx will be divisible by 6. 32! + 33! + 34! + ……90!
∴ of the sets X1 to X78, there will be 13 sets which do not = 231 (k + 33k + 34.33k + 35.34.33k + 36.35.34.33.k+ …..)
contain 6, or its multiple. where k is odd
∴ 65 sets will contain 6 or its multiple X79 will not contain = 231(34.k (1 + 33 + 35.33) + a multiple of 36)
6 or its multiple. X80 will contain 6 or its multiple. = 231(34k1 + a multiple of 36) where k1 is odd
∴ a total of 66 sets contain 6 or its multiple. = 231(34k1 + 36k2) (say)
Choice (B) = 231(2(17k1 + 18k2)) = 232(an odd number)
Largest power of 2 in the sum is 32. Choice (A)
34 Given value is 0.754 + 0.692 39. 5x + 4y = 9k1 + 4 and 4x + 5y = 9k2 + 5
754 − 7 747 83 ∴ x – y = (5x + 4y) – (4x + 5y) = 9(k1 – k2) – 1
Fractional value of 0.754 = = =
Rem x − y = –1. The corresponding positive remainder is
990 990 110
692 − 69 623 9
Fractional value of 0.692 = =
900 900 –1 + 9 i.e. 8. Ans: (8)
nd
Triumphant Institute of Management Education Pvt. Ltd. (T.I.M.E.) HO: 95B, 2 Floor, Siddamsetty Complex, Secunderabad – 500 003.
Tel : 040–27898195 Fax : 040–27847334 email : info@time4education.com website : www.time4education.com SM1001962/8
40. N = 21600 = 63 102 = 253352 9. The number n, the number of times it occur in N and the
N has 6 (4) (3), i.e. 72 factors. Any factor in of the form number of digits it contributes and the total number of digits
2a3b5c are tabulated below.
24 = 23 31 No. of Total number
72 = 23 32 n No. of digits
occurrences of digits
We want all the factors of N, which are multiples of 24 but 1 1 1 1
not of 72. Therefore, a can be 3,4 or 5, b has to be 1, and c 2 2 2 3
can be 0, 1 or 2. The number of such factors = 3 (1) (3) = 9. 3 3 3 6
Ans: (9) -- -- -- --
9 9 9 45
Exercise – 1(b) 10 10 20 65
11 11 22 87
Solutions for questions 1 to 60: 12 6 12 99
1 (of 12) Half 1 100
1. (a) 2
of 45 ÷ 5 ×  24 − 1 ÷ 90  ∴ The last 4 digits of N are 2121
3   The first 2 comes from the 5th 12
Applying BODMAS Rule, we have The 12 comes from the 6th 12
90 ÷ 5 × (15 ÷ 90) = 30 ÷ 5 × 1 = 1 The 1 is part of the 7th 12.
N 2121
3 6 ∴ Rem = Rem =9 Ans: (9)
Choice (B) 16 16
10. If n = 2, 25 – 2 = 32 – 2 = 30
of 9 – 4 − 5 + 2 7 + 3 
1
(b) 5 + 6 x If n = 3, 35 – 3 = 243 – 3 = 240
3  8 8 4
If n = 4, 45 – 4 = 1024 – 4 = 1020
1 5 6 In all the cases n5 – n is divisible by both 3 and 5.
=5+6x x 9 – {4 – +2+ 7+ }
3 8 8 8 Only 5 is there in the options. Choice (A)
6+7−5 General proof
= 5 + 18 – {4 + 2 + } Any value of n where n is a positive integer can be
5
expressed in the form 5k or 5k – 1 or 5k – 2 or 5k – 3 or
= 5 + 18 – {6 + 1} = 5 + 18 – 7 = 16 5k – 4 where k is an integer.
Choice (C) If n = 5k, n is always divisible by 5
If n = 5k – 1, n + 1 = 5k is always divisible by 5
2. 753 – 503 – 253 = 753 + (–50)3 + (–25)3
If n = 5k – 2, n2 + 1 = 5k’ is always divisible by 5
Now, 75 + (–50) + (–25) = 0, If n = 5k – 3, n2 + 1 = 5k’ is always divisible by 5
When a + b + c = 0, we have a3 + b3 + c3 = 3abc If n = 5k – 4, n – 1 = 5k – 4 – 1 = 5k – 5 = 5(k – 1) and is
∴753 – 503 – 253 always divisible by 5.
= 3 x 75 x (–50) x (–25) = 281250 Ans: (281250) Hence in general, for any n,
n(n – 1) (n + 1) (n2 + 1) = n5 –n is always divisible by 5.
3. For number of this type (i.e., 1234 . . . n n – 1 . . . 1), the
number of digits in the square root of the number will be 11. n(2n + 1) (n2 – 1) (4n2 + 4n) = n(n2 – 1) 4(n) (n + 1)
equal to the middle digit of the number. (2n + 1)
Choice (C) = n(n – 1) (n + 1) 4 n(n + 1) (2n + 1)
We know that n(n – 1) (n + 1) is a product of three
4. The given expression is of the form consecutive numbers and hence is divisible by 3! = 6.
a3 + b3 − c 3 + 3abc n(n + 1)(2n + 1) can be written as n(n + 1)[(n + 2) + (n – 1)]
= ⇒ n(n + 1)(n + 2) + n(n + 1) (n – 1)
a + b 2 + c 2 − ab + bc + ca
2
∴n(n + 1)(2n + 1) is divisible by 6.
where a = 0.68, b = 0.67 and c = 0.5 ∴n(2n + 1)(n2 – 1) 4(n2 + n) is divisible by 6 × 6 × 4
a3 + b3 + c 3 − 3abc = 144 Choice (D)
=a+b−c
a 2 + b 2 + c 2 − ab − bc − ca 12. abcde = 10000a + 1000b + 100c + 10d + e
= 0.68 + 0.67 − 0.5 = 0.85 acdbe = 10000a + 1000c + 100d + 10b + e
Choice (D) The difference of abcde and acdbe
= (10000a + 1000b + 100c + 10d + e) – (10000a + 1000c +
5. When 546789 is divided by 7, the remainder is 5. Hence 100d + 10b + e) = (990b – 900c – 90d)
5 should be subtracted from 546789 so that it becomes a 18 (55b – 50c – 5d) which is always divisible by 9 and 18
multiple of 7. Choice (A) Choice (D)

6. The number is divisible by 10. ∴ c = 0 13. 22 and 32 when divided by 6 leave remainders of 4 and
The number is divisible by 8 ∴ B = 1, 3, 5, 7 or 9. 3 respectively. The square of all other primes are of the
The number is divisible by 9. The sum of all the known digits form (6k ± 1)2 which when divided by 6 leave a remainder
of the number leaves a remainder of 1. of 1. Hence the sum of the distinct possible remainders
∴ A + B = 8 or 17. is 8. Ans: (8)
∴ (A, B) can be (7, 1) (5, 3) (3, 5) (7, 1) or (8, 9).
14. Given, 35 + Dq = N; 1750 + D(50q) = 50N
It can take 5 values. Choice (D)
As 50N leaves a remainder of 11, 1750-11 or 1739 is a
7. Given the expression X(X3 + 2X2 + 3X + 4) + 36. multiple of D or D is a factor of 1739 = 1764 − 25 = (42 − 5)
In order for the expression to be divisible by X, 36 must be (42 + 5). We have to consider those factors of 1739, which
divisible by X. are greater than 35 and 11 (∴ 35), because the divisor has
∴ Number of values that x can assume = Number of to be greater than the remainders. We see that D can be
factors of 36 = 9. Ans: (9) 37, 47 or 1739. Choice (D)

8. N3 + 6N2 + 8N = N(N2 + 6N + 8) = N(N + 2) (N + 4)] 15. HCF of 1/5, 4/15 and 8/25 = 1/75
LCM of 1/5, 4/15 and 8/25 = 8/5
= 8M (M+1) (M+2), where N = 2M. The product of
(LCM of 1/5, 4/15 and 8/25) 8 / 5 8
3 consecutive numbers is always divisible by 6. Hence = x75 = 120
∴ The given expression is always divisible by 48. HCF of (1/5, 4/15 and 8/25) 1/ 75 5
Choice (D) Ans: (120)

Triumphant Institute of Management Education Pvt. Ltd. (T.I.M.E.) HO: 95B, 2nd Floor, Siddamsetty Complex, Secunderabad – 500 003.
Tel : 040–27898195 Fax : 040–27847334 email : info@time4education.com website : www.time4education.com SM1001962/9
16. Let the number be x. ∴Number of ways of expressing it as product of two
⇒ x is the HCF of (971 – 3), (852 – 5) 1
= H.C.F of (968, 847) is 121 numbers = [(2 + 1) (3 + 1) (1 + 1) (1 + 1)] = 24
2
∴ x = 121 Choice (C) Choice (C)
17. General form of number = K [LCM (5, 6, 7)] − 3 26. The given number is 2304 = 4(576) = 4(26)(32) = 2832
(L.C.M. Model 2) We need the number of numbers between 1000 and 2000,
where K is a natural number. that are relatively prime 2, 3 or 6.
∴ 210K1 − 3 = 47K2 − 6. In any group of 6 consecutive numbers, there are φ(6) or
210K1 + 3 = 47K2 φ(2)φ(3) or 1(2) i.e., 2 numbers that are relatively prime to
22K1 + 3 6. In the 166 groups, each of 6 consecutive numbers, i.e.,
4K1 + = K2
47 1003-1008 to 1993-1998, there are 2(166) or 332 numbers
Least value of K2 is realized when K1 = 2. that are coprime to 6. Besides these, 1001 and 1999 are
∴ Least number = 417 Choice (B) also coprime to 6. ∴ The number of numbers between
1000 and 2000 and coprime to 6 (or 2304) is 334.
18. The number is of the form 9k + 6 Choice (B)
This number when divided by 7 leaves a remainder 5.
So, 9k + 6 – 5 is divisible by 7 27. To find the highest power of 3 that will exactly divide 100!,
Substituting the smallest value 3 for k the multiple of 7 is we successively divide 100 with 3
obtained. 3 100 33
∴The smallest number = 9k + 6 = 9 x 3 + 6 = 33 3 33 11
So, the general form of the number satisfying the condition 3 11 3
is 33 + 63k 3 3 1
Dividing 999, the largest 3-digit number that is possible, by We stop here because 1 is less than 3.
63, the quotient is 15. Hence, k = 15 and 63k +33 = 978; Adding all the quotients,
which is the largest 3-digit number Choice (D) 33 + 11 + 3 + 1 = 48 is the highest power of 3
Choice (C)
19. Each runner will be at the starting point at time intervals
which are multiples of 200S, 300S, 360S and 450S. All the 28. b = a + 2 and c = a + 4
four will be together at the starting point at time intervals If a is even, b and c are also even. As a, b and c are prime,
which are multiples of all the four given time intervals. this is not possible.
∴ Required time interval in seconds ∴ a, b and c are odd.
= LCM (200, 300, 360, 450) = 1800 Ans: (1800) Least possible value of a is 3.
If a = 3, b = 5 and c = 7
20. Divisors →7 5 If a > 3, it must be in the form 6k ± 1 where k is a natural
remainders →4 2 number. If a is of the form 6k − 1, c is not prime.
(a, b, c) = (3, 5, 7) is the only possibility. Ans: (1)
∴Required number = 7 × 2 + 4 = 18
Remainder when 18 is divided by 17 is 1 Choice (A) 29. If (R – 1)! is not divisible by R, R is a prime number.
Since 1 ≤ R ≤ 50, the objective is to find the number of
21. Number of chocolates received by Rajesh must be in the prime numbers between 1 and 50. On calculating we find
force 3k + 1 where k is the number of chocolates distributed there are 15 primes between 1 and 50. Hence there are
to each of his friends. One of his friends distributed 15 values of R such that (R – 1)! is not divisible by R.
k chocolates equally among four friends. Also for R = 4, (R – 1)! i.e., 3! is not divisible by 4. Hence,
∴ k must be divisible by 4. the total number of values is (15 + 1) = 16 Ans: (16)
∴ k = 4 (a) where a is a natural number.
∴ k = 4a where a is a natural number. 30. 1 = 2+
1
2+
∴ 3k + 1 = 12a + 1 Only Choice (C) is of this form. 2 2
2+ 2+
Choice (C) 2 2
2+ 2+
2 2
22. Divisors : 6 × 5 × 4 2+ 2+
1 5
+ + 2+
2 2
1 1
Remainders: 4 3 2 = 2+ = 2+
Smallest value of the number [2(5) + 3]6 + 4 = 82 2 2
2+ 2+
General form of the number = (6) (5) (4) k + 82, where k is 2 2
2+ 2+
a whole number. 4 14
When the number has the largest four-digit value 2+
5 5
(6) (5) (4)k + 82 < 10000 and k is maximum.
1 1 1
= 2+ =2+ = 2+
∴ k =  9918  = 82 2 2 2
 120  2+ 2+ 2+
10 28 10 38
∴ Largest value = 120 (82) + 82 = 9922 Ans: (9922) 2+ +
14 14 14 14
1 1 1 38
23. 88400 – 4(22100) = 4(221) (100) = (24) (52) (13) (17) =2+ = 2+ = 2+ = 2+
28 76 28 104 104
Number of factors of 88400 = (4 + 1) (2 + 1) (1 + 1) (1 + 1) = 60. 2+ +
Choice (C) 38 38 38 38
208 38 246
= + = = 123/52 Choice (C)
24. 24700 = (247) (100) = (19) (13) (122) (52) 104 104 104
Number of ways in which a number having n prime factors
can be expressed as a product a two co-prime factors = 2n−1 31. Choice (A)
∴ required number of ways = 24−1 = 8 Choice (C) As q and r are odd, q2 and r3 are odd.
∴ pq2r3 is odd
25. Expressing 3780 as product of prime factors we have 3780 ∴ Choice (A) is always true.
= 22 × 33 × 51 × 71. Choice (B)
Triumphant Institute of Management Education Pvt. Ltd. (T.I.M.E.) HO: 95B, 2nd Floor, Siddamsetty Complex, Secunderabad – 500 003.
Tel : 040–27898195 Fax : 040–27847334 email : info@time4education.com website : www.time4education.com SM1001962/10
(p + q)2 is even. Here, (57, 1) and (19, 3) can be eliminated because, the
∴ (p + q)2 r3 is even. higher the H.C.F, the higher is the sum and minimum sum
∴ Choice (B) is always true. is required.
From (i), H.C.F =1 and L.C.M = 58
Choice (C) ∴(a, b) can be (1, 58) or (2, 29)
q + r is even. From (iii), H.C.F = 3 and L.C.M = 60
∴ (p − q + r)2 (q + r) is even. ∴(a, b) can be (12, 15) or (3, 60)
∴ Choice (C) is always true. ∴Minimum possible sum for a and b is 12 + 15 = 27
Choice (D) Choice (B)
If p = 1, q = 3 and r = 5, pqr leaves a remainder of 3 when
divided by 4. 40. Going by the options, the first option is 206.
The sum of the first 20 natural numbers is
If p = 3, q = 5 and r = 7, pqr leaves a remainder of 1 when 20(21)/2 = 210
divided by 4. Hence 206 which is just 4 less cannot be the sum of
∴ Choice (D) is not always true. Choice (D) consecutive natural numbers starting from 1.
The sum of the first 29 natural numbers is
32. The square of any natural number ends with the same units (29) (30)/9 = 870/2 = 435
digit as that of the number if the number ends with Since 439 is just 4 more than 435, it cannot be the sum of
0, 1, 5 or 6. consecutive natural numbers starting from 1.
∴ (AB)2 = CCB means that B can be 0, 1, 5 or 6. As B is a The sum of the first 40 natural numbers is
natural number, B can be 1, 5 or 6. As CCB < 1000, AB ≥ 31. (40) (41)/2 = 850.
As A and B are distinct, AB can be 15, 16,. 21, 25, 26 or 31. Since 805 is just 45 less than 850, it cannot represent the
Only 152 and 212 are in the form CCB. sum of consecutive natural numbers starting from 1.
∴ AB has 2 possibilities. Choice (C) The sum of the first 50 natural numbers is
(50) (51)/2 = 1275
33. Let the smaller number be n and the larger number be N.
N² + n3 = 593 Alternate method:
⇒ N < 25 and N + 55 = n², ⇒ n² < 55 + 25 ⇒ n < 9 Sum of the first n natural numbers is n(n + 1)/2.
for n = 8, n3 = 512 and N² = 593 – 512 = 81 ⇒ N = 9 The given options refer to this value; alternatively n(n + 1) still
∴N–n=1 Ans: (1) be equal to double the values given under options.
⇒ we have to find which one among (2 × 206), (2 × 439),
34. N = 2162 = 66 = 2636. (2 × 805) and 2(1275) is equal to n(n + 1).
The number of factors of N (say φ) is 49. Among the numbers 412, 878, 1680 and 2550, the only
The product of all the factors of N is Nφ/2 number which can be expressed as the product of two
The product of all the factors of N is (2162)49/2 = 21649. consecutive numbers is 2550 (50 and 51). It represents the
Choice (A) sum of first n natural numbers. Choice (B)

35. N = 1296000 = 64 103 = 27 34 53. ∴ N has 8(5) (4) or 160 41.


800 !
= 800 (799) …….(401)
factors. The product of all these factors is (1296000)80. 400 !
Choice (A)
The index of the greatest power (IGP) of 11 that divides
36. A minimum of 5 coins are required to pay 69 paise. 800 !
is obtained as follows.
(1 50 p, 1 10p, 1 5p and 2 2p). 400 !
A minimum 3 coins are required to pay `1.05
(2 50p and 1 5p) 800 72 6 400 36 3
A minimum of 3 coins are required to pay 85p 11 11 11 11
(1 50 p, 1 25 p and 1 10 p)
∴ IGP of 11 in 800! is 72 + 6 = 78 while IGP of 11 in 400! is
∴ Minimum number of coins required in total = 11
36 + 3 = 39
Choice (D)
800 !
IGP of 11 in = 78 – 39 = 39 Ans: (39)
37. Required divisor = 2 × 16 – 9 = 23 Ans: (23) 400 !

38. Let the number added twice be x. 42. Let each integer satisfying the given condition be denoted
The correct sum = 860 – x. by N.
N(N + 1) N2 exceeds a perfect square by 113.
This must be in the form =
2 Let us denote the perfect square by x2
where N is a natural number. N2 − x2 = 113.
N(N + 1) (N − x) (N + x) = 113.
When N = 39, = 780 113 is prime.
2
∴ (N − x, N + x) = (−1, −113) or (1, 113)or (113, 1) or
N(N + 1) (−113, −1).
When N = 40, = 820
2 ∴ (N, x) = (−57, − 56) or (57, 56) or (57, −56) or (−57, 56)
N(N + 1) N has two possible values, –57 or 57. Choice (D)
When N = 41, = 861
2
43. x2 − y2 = 255
860 – x < 860
(x − y) (x + y) = 255.
∴ N ≤ 40.
Both x − y and x + y must be positive.
If N = 40, x = 40.
If N < 40, x > N. This is not possible. Choice (B) Also x − y < x + y.
∴ (x − y, x + y) = (1, 255);(3, 85);(5, 51) or (15, 17)
39. Let the L.C.M and H.C.F of a and b be ℓ and h respectively. Choice (A)
Given ℓ – h = 57.
Let k times the h be ℓ 44. In the first round, employees 2, 4, ….180 made an exit. In
∴kh – h = 57 ⇒ h(k – 1) = 57 the second round, 3, 9, 15 made an exit.
Now 57 can be expressed as product of two numbers in the In the third round, 5, 25, 35, 55, 65, 85, 95 made an exit.
following ways. The number of round is equal to the number of primes less
(i) (1, 57) (ii) (57, 1) (iii) (3, 19) (iv) (19, 3) than 100, which is 25. Choice (A)

Triumphant Institute of Management Education Pvt. Ltd. (T.I.M.E.) HO: 95B, 2nd Floor, Siddamsetty Complex, Secunderabad – 500 003.
Tel : 040–27898195 Fax : 040–27847334 email : info@time4education.com website : www.time4education.com SM1001962/11
45. Let the quotients when N is divided by 7, 8, 9 be q1, q2, q3 This is the only way to express the number as the product
respectively. of 7 numbers between 5 and 19. The sum of these 7
N = 7q1 + 5 = 8q2 + 6 = 9q3 + 7 factors is 79. Ans: (79)
N + 2 = 7(q1 + 1) = 8(q2 + 1) = 9(q3 + 1)
N is the least integer satisfying the given conditions. 52. The largest power of 2 in 15! is 11
∴ N + 2 must also be the least. 16! = 16(15!) = 24(15!). ∴ Largest power of 2 in 16! is 15.
∴ N + 2 must be the LCM(7, 8, 9) i.e., 504. ∴ Each of the factorials 16!, 17!, …..100! have the largest
∴ N = 502 power of 2 greater than or equal to 15.
15! + (16! + 17!+ …… 100!) = 211 (an odd number) + 215
N (an integer)
Rem =9 Choice (A)
17 = 211 (an odd number + 16 (an integer))
= 211(an odd number)
46. Let the original four digit number be abcd The largest power of 2 in 15! + 16! + 17! + …. 100! Is 11.
K = abcd – dcba = 1000 a + 100 b + 10 c + d Choice (A)
– (1000 d + 100 c + 10 b + a) 53. The given expression E has the form
= 999 (a – b) + 90 (b – c) = (37) (27) (a – b) + 90 (b – c) p3 + q3 + r3 – 3pqr where p + q + r = 0 (p = 4a + 8b – 12c, q
This is divisible by 74 i.e. by both 37 and 2 = 8a – 12b + 4c and r = –12a + 4b + 8c)
K is divisible by 37. ∴ 90 (b – c) must be divisible by 37. E = (p + q + r) (p2 + q2 + r2 – pq – qr – rp)
∴ c – b must be divisible by 37. ∴ Only possibility is b – c This is 0 since p + q + r is 0. ∴ E is both non-negative and
is 0 i.e. b = c ------------ (1) non-positive. Choice (C)
K is divisible by 2 i.e. K is even. ∴ (37) (27) (a – d) must be
even. ∴ a – d must be even. The least value of a – d is 2. 54. X(p, q, r, s, t) = 32 – 16 (Σp) + 8(Σpq) – 4(Σpqr)
(If a – d = 0, then ‘abcd’ = abba and K = abba – abba = 0, + 2(Σpqrs) – pqrst = 32 – 16(p + q + r + s + t )
which is not positive) + 8 (pq+ pr + ps + pt + qr + qs + qt + …+st)
From (1) and (2), the least value of abcd is 2000. This lies –4 (pqr + pqs + ….+ rst) + 2(pqrs + pqrt + pqst + prst) –
between 1900 and 2200. Choice (B) pqrst = (2 – p) (2 – q) (2 – r) (2 – s) (2 – t)
 16 15 14 13 12   16  15 
47. The numbers between 2000 and 2400 (both inclusive), which X , , , ,  =  2 −  2 − 14 
 15 14 13 12 11   15  
have only even digits and which are multiplies of 3 are listed
below. (The sum of the digits has to be a multiple of 6)  14  13  12  14  13   12   11   10 
2 −  2 −  2 − =     
2004 2202 2400  13  12  11  15  14   13   12   11 
2022 2208
2
2028 2220 = Choice (A)
2040 2226 3
2046 2244
2064 2262 55. 34 52 = 81 (25) = 2025
2082 2268 di is either in the form 4K + 1 or 4k – 1
di −1
2088 2280 If it is in the first form (− 1) 2 = (− 1)2k = 1
2286
θi −1
There are 18 such numbers. Ans: (18) If θi is in the second from, (− 1) 2 = (− 1)2k −1 = –1
48. Let the numbers be a, b, c All the factors of 34, 52 are odd. Its factors are 1, 3, 9, 27,
a2 + b2 = ab + c2, a2 + c2 = ac + b2, b2 + c2 = bc + a2 81, 25, 15, 75, 45, 225, 135, 675, 405, 2025. Of these, 6
Adding these, we have a2 + b2 + c2 = ab + bc + ac are of the form 4K – 1 and the remaining 9 are of the form
∴ 2(a2 + b2 + c2) = 2(ab + bc +ac) 4K + 1
(a – b)2 + (b – c)2 + (c – a)2 = 0 F(34 52) = 6(–1) + 9(1) = 3 Choice (B)
This is possible only when a – b = b – c = c – a = 0 i.e,
 2x 
a=b=c Choice (C) 56. By remainder theorem, Re m 6  = 1 and
 7 
 
49. Let the three-digit number be abc.
b=a+c  6 2 x +1 
Re m  = –1 i.e, 6
The greatest odd number must have the greatest possible  7 
 
value of a and an odd value of c. (Also b≤9)
∴a = 8, c = 1 and b = 9 Let 62x = 7a + 1 ∴ 62x + 1 = 42a + 6
 2x 
∴  6  =  7a + 1 = a + 1 and
or abc = 891 = 111(34)
Number of factors of 891 is (1+1) (4+1) = 10
 7   7 
Note: b = a + c means the number is divisible by 11.
Ans: (10)  6 2x +1   42a + 6 
  =  = 6a + 1
50. Consider the number 4620 rather than 4624.  7   7 
4620 = 11 (420) = 11 (7)(22)(3)(5) Required sum = (a + 1) + (6a + 1) = (7a + 1) + 1 = 62x + 1
Number of positive integers up to 4620 which are not Choice (C)
divisible by any of 2, 7 or 11
 1  1  1   1   1 
 1  1  1 57. 1 −  1 −  1 −  1 −  …… 1 − 
= 46201 − 1 − 1 −  = 1800  4  9  16   25   900 
 2  7  11 
 1    1   1   1 
Of the other four numbers upto 4624, 4621 and 4623 are = 1 −  1 − 1  1 −  1 − 2  ….. 1 − 
not divisible by any of 2, 7 or 11.  22   3 2   42   5   30 2 
A total of 1802 positive integers up to 4624 are not divisible
 1  1   1   1
= 1 − 1  1 +  1 −   1 + 1 −  1 +  ….
by any of 2, 7 and 11. ∴ 2822 positive integers up to 4624 1
are divisible by at least one of 2, 7 and 11. Choice (A)  2   2  3    3  4   4
 1   1 
51. 16081065 = 5 (3216213) = 5(9) 357357 1 −  1 + 
= 5(9)(357)(1001)  30   30 
= 5(9)3(7)(17)7(11)(13) 31
= 7(7)(9)15(11)(13)(17) = 1 × 1 × 1 × ……. 1× = 31 . Choice (B)
2 30 60

Triumphant Institute of Management Education Pvt. Ltd. (T.I.M.E.) HO: 95B, 2nd Floor, Siddamsetty Complex, Secunderabad – 500 003.
Tel : 040–27898195 Fax : 040–27847334 email : info@time4education.com website : www.time4education.com SM1001962/12
58. 5400 = 2 x 10 x 270 = 2 x 10 x 27 x 10 Using both the statements,
= 2 x 2 x 5 x 33 x 2 x 5 = 23 x 52 x 33 Adding equation (1) and (2) we get
Hence sum of all the factors of 5400 2x = d (k1 + k2)
24 − 1 53 − 1 3 4 − 1 15 124 80 Since d is odd, k1 + k2 is even.
= x x = x x = 18600 x k1 + k 2
2 −1 5 −1 3 −1 1 4 2 = = integer.
Ans: (18600) d 2
∴ x is divisible by d.
59. (N1 ⊕ 8) # (N2 ⊕ 7) = 21. In other words, the product of the Similarly, y is also divisible by d. Choice (C)
remainders of N1 divided by 8 and N2 divided by 7 is 21. a
∴ The only possible remainders when N1 and N2 are 67. = 1⇒ a = b – c so we have to find a
b−c b
divided by 8 and 7 are 7 and 3 respectively.
∴ Statement Ι alone is sufficient.
(∵ Any remainder must be less than the divisor).
From statement ΙΙ, a and b are co-primes
N1 and N2 are natural numbers not more than 100.
N1 can be 7, 15, 23, ……. 95 So a may be 3 or 7 or any other such value
b 5 9
N2 can be 3, 10, 17, ……. 94
N1 has 12 possible values and N2 has 14 possible values. So unique value is not possible. Choice (A)
∴ (N1, N2) has 168 possible values. Ans: (168) 68. 10 < 3n < 300 so n = 3, 4, or 5
60. Let the numbers be 3x, 4x and 5x. From statement Ι, n is the square of an integer.
LCM (3x, 4x, 5x) = x LCM (3, 4, 5) = 60x ∴n = 4.
Given 60x = 480 From statement ΙΙ, 3n is the square of an integer.
x=8 ∴ 3n = 81 ⇒ n = 4.
sum 3x + 4x + 5x =12x = 96 Choice (A) ∴Either statement alone is sufficient. Choice (B)

Solutions for questions 61 to 75: 69. From statement Ι,


1 + 2 + 3 + 4 + 6 = 16 (the only possibility)
61. From Ι, some of the values of x are 101, 116, ………… So, Ι alone is sufficient.
From ΙΙ, some of the values of x are 106, 117, ………… From statement ΙΙ, 1 × 2 × 3 × 4 × 5 = 120 (the only
From Ι and ΙΙ, the difference between divisors and possibility)
remainders is constant i.e., 4, hence (15 × 11) – 4 = 161 is So, ΙΙ alone is also sufficient. Choice (B)
the only number in the range of (100 and 265).
Choice (C) 70. GCD of (2a, 2b) = 10 ⇒ Let 2a = 10 k and 2b = 10 m,
where k and m are co-primes.
62. x < 0. ⇒ a = 5k; b = 5m
From statement Ι, at least one of y and z is < 0 ∴ GCD of a and b is 5.
If y and z are negative, then xyz < 0 From Statement Ι alone, we can answer.
If only one of y and z is negative, then xyz > 0. Statement ΙΙ does not give any information to solve.
So statement Ι alone is not sufficient Choice (A)
From statement ΙΙ, y + z > 0 ⇒ at least one of y and z is
positive or both are positive. 71. From statement Ι, x = Nk and y = Nr
If both are positive then xyz < 0. Only if k and r are co-primes, then N is the HCF of x and y
If only one is positive then xyz > 0, so statement ΙΙ alone is other wise not. ∴ Ι alone is not sufficient.
y
not sufficient. From statement ΙΙ, x = 2N k1 and = 2N r1
Combining statements Ι and ΙΙ, between y and z, one is 2 4
negative and other is positive. So xyz > 0. Choice (C) ∴ 4N divides x and y.
∴ N is not HCF of x and y.
63. From Ι, a2 + b2 + c2 = ab + bc + ca ∴ Statement ΙΙ alone is sufficient. Choice (A)
⇒ 2(a2 + b2 + c2) = 2(ab + bc + ca)
⇒ (a2 + b2 – 2ab) + (a2 + c2 – 2ca) + (b2 + c2 – 2bc) = 0. 72. From statement Ι,
⇒ (a – b)2 = (b – c)2 = (c – a)2 = 0. x = 5k + odd positive integer where k is a non-negative
∴ a = b = c. integer.
As abc ≠ 0, a3 + b3 + c3 ≠ 0. If k = 1 then x is even.
If a + b + c = 0, a3 + b3 + c3 = 3abc. As abc ≠ 0, a3 + b3 +c3 ≠ 0 If k = 2 then x is odd.
∴ Either statement is sufficient to answer the question. Statement Ι alone is not sufficient
Choice (B) From statement ΙΙ,
a = 4P + odd positive integer, where P is a non-negative
64. From statement Ι, x = n². integer.
From statement ΙΙ, x = k3. If P is odd or even x is always odd
Combining statements Ι and ΙΙ So statement ΙΙ alone is sufficient. Choice (A)
If x = 729 it is a perfect square and a cube.
If x = 64 it is also a perfect square and a cube 73. From statement Ι, when x is divided by 8 the remainder is
∴x can be even or odd. 3. So x = 8k + 3, where k is a whole number, when 8k + 3 is
Hence, both statements together are also not sufficient. divided by 4 the remainder is 3. So statement Ι alone is
Choice (D) sufficient.
From statement ΙΙ, when x = 5 the remainder when x is
65. Let x be the number of soldiers. divided by 4 is 1 but when x = 10 the remainder is 2.
From statement Ι, x is a multiple of the LCM of 3, 5 and 7. So, the question cannot be answered by statement ΙΙ
∴x = 105 k ⇒ x can be 105 or 210. alone. Choice (A)
So statement Ι alone is not sufficient.
74. 2a + 4b + a – b + c = 3 (a + b) + c.
From statement ΙΙ, x is even.
Combining both the statements, we get x = 210. From statement Ι, we don’t know whether c is divisible by
Choice (C) 3 or not, so we can’t answer the question.
From statement ΙΙ, c is divisible by 3.
66. From statement Ι, x + y = dk1……….(1) ∴ 3(a + b) + c is divisible by 3.
From statement ΙΙ, x – y = dk2……….(2) Statement ΙΙ alone is sufficient. Choice (A)

Triumphant Institute of Management Education Pvt. Ltd. (T.I.M.E.) HO: 95B, 2nd Floor, Siddamsetty Complex, Secunderabad – 500 003.
Tel : 040–27898195 Fax : 040–27847334 email : info@time4education.com website : www.time4education.com SM1001962/13
75. pq = rq (b) The only perfect square of the form A5B is 256.
From statement Ι, if q = 3, p = r. If q = 6, p = ±r. Ι is not ∴ A is even. Choice (B)
sufficient. (c) 121 as well as 441 are perfect squares ∴ We cannot
From statement ΙΙ, p = r as q is odd ΙΙ is sufficient. say. Choice (C)
Choice (A)
16. 39 511 1513 = 39 511 (3.5)13 = 39 511 313 513 = 322 524
Chapter – 2 The index of each of the prime factors of 39 511 1513 is even.
(Numbers – ΙΙ) ∴ The number is a perfect square. Choice (A)

Concept Review Questions 17. The product of a x digit number and a y digit number must
have either (x + y – 1) digits or (x + y) digits.∴ The product
Solutions for questions 1 to 25: can have either 18 or 19 digits. Choice (C)
1. an + bn is a multiple of a + b if n is odd. 18. The product of a x digit number, y digit number and a z digit
∴ 11103 + 14103 is a multiple of 25. Choice (B) number must have either (x + y + z – 2), (x + y + z – 1) or
(x + y + z) digits.∴ The product can have either 22, 23 or
2. 292n – 112n = 841n – 121n 24 digits. Choice (D)
an – bn is divisible by a – b, for all positive integral values of n.
∴ 841n – 121n is always divisible by 720. Ans: (720) 19. The least 13-digit number is 1012. Its square root is 106 i.e. a
7 digit number. The greatest 13 digit number is 1013 – 1. Its
3. Let the number be N. square root is less than the square root of 1013 = 10(106)2
N = 48q + 31 where q is the quotient when N is divided by
48; N = 24 (2q + 1) + 7 1013 = 106 (3.3) which has 7 digits.
∴The remainder when N is divided by 24 is 7. ∴ The square root must have 7 digits. Choice (A)
Choice (B)
20. 3000 < 3PQR < 4000
4. Let the number be N. N = 18 q + 15 where q is the quotient (3000)4 < (3PQR)4 < (4000)4
when N is divided by 18. (81) (1012) < (3PQR)4 < (256) (1012)
If q is of the form 4k + r, where k is a whole number and r is (81) (1012) has 14 digits while (256) (1012) has 15 digits.
the remainder when q is divided by 4, N = 72k + 18r + 15 ∴ (3PQR)4 has either 14 or 15 digits. Choice (C)
r can be 0 or 1 or 2 or 3.
∴ The remainder of N divided by 72 can be 18(0) + 15, 21. The smallest and largest 25 digit numbers are
18(1) + 15 or 18(2) + 15 i.e. 15, 33 or 51. Choice (D) 1024 and 1025 – 1 respectively. Their respective cube roots
are 108 and 3 10 (108). In either case, the cube root of a
18168 18168
5. = 25-digit number will be a 9-digit number. Choice (A)
19 18 − ( −1)
22. The factorial of any natural number greater than 4 ends
By remainder theorem, remainder is (–1)168 = 1 Ans: (1)
with a 0.∴ The units digit of the sum = units digit of
6. The 3rd odd natural number is 5. 1! + 2! + 3! + 4! = 3 Ans: (3)
∴ The product ends with 5. Ans: (5) 23. (a) Considering only the units digit of the numbers in
multiplication, units digit of the product = 8 × 4 × 2 × 3
7. The 5th and 10theven natural numbers are 10 and 20
=2 Choice (A)
respectively.
(b) Consider each of the numbers and find their units
The last two digits of the product are both 0. Choice (D)
place.
8. The remainder of any number divided by 9 is the remainder Units place of 748
of the sum of its digits divided by 9. The sum of the digits of Cycle of 7 = 7,9,3,1 i.e., period is 4
the given number is 37. ∴ The required remainder is 1. 48/4 = 12, no remainder i.e. 12 cycles
Ans: (1) ∴units place of 748 is 1.
Units place of 356
9. The remainder of any number divided by 25 is the Cycle of 3 is 3,9,7,1 i.e., period of 4
remainder of the number formed by its last 2 digits divided 56/4 = 14, no remainder i.e. 14 cycles.
by 25. The number formed by the last two digits is 37. ∴units place of 356 is 1
The remainder is 12. Choice (B) Units place of 16535
Units place of 5 raised to any power is 5.
10. The required number must be the 4-digit number of the ∴Units place of 16535 is 5
form 19k+7, where k is the greatest natural number
∴Units place of 748 × 356 × 16535 is 1 × 1 × 5 = 5
satisfying 19 k + 7 < 10000
Choice (B)
9993 18 (c) Units digit of 84n:
k< = 525
19 19 4n is a multiple of 4
∴ k = 525 cycle of 8 is 8, 4, 2, 6 i.e., period is 4
∴ The required number = (19) (525) + 7 = 9982 4n/4 = n no remainder i.e., n cycles.
Ans: (9982) ∴Units place of 84n is 6.
Units digit of 6n:
11. The tens digit of a perfect square ending in 6 must be odd. Units place of 6 raised to any power is 6.
∴ pqr26 cannot be a perfect square. Choice (B) ∴Units place of 6n is 6
Units digit of 92n
12. Any perfect square ending with 5 must have a tens digit of 2. 2n is a multiple of 2
∴ 1a4b75 is not a perfect square. Choice (B) Cycle of 9 is 9, 1 i.e., period is 2.
2n/2 = n i.e., no remainder i.e. n cycles.
13. 3036 is not a perfect square, whereas 1936 is a perfect
∴Units place of 92n is 1.
square. ∴ We cannot say Choice (C)
Units digit of 84n × 6n × 92n = 6 × 6 × 1 = 6
14. For any value of C from 1 to 9, C36 cannot be a perfect square. Choice (C)
Choice (B) (d) Units digit of the product 31. 32. 33. - - - - - 39 = units
digit of 1. 2. 3. 4. 5. - - - -9 = the units digit of product
15. (a) Perfect squares of the form P6Q are 169, 361 and 961. of 2 and 5 which is 0.
∴ P must be odd. Choice (A) ∴ Required units digit = 0. Choice (A)

Triumphant Institute of Management Education Pvt. Ltd. (T.I.M.E.) HO: 95B, 2nd Floor, Siddamsetty Complex, Secunderabad – 500 003.
Tel : 040–27898195 Fax : 040–27847334 email : info@time4education.com website : www.time4education.com SM1001962/14
24. 23n – 1 = (23)n – 1 = 8n – 1n A more general statement is that, if n is odd, the remainder
xn – yn is divisible by (x – y ) for all the values of n i.e., even of An divided by A + 1 is always A.
or odd
Rem   = Rem  (11k + 10) − ( 11( 68) + 2)  = 8
N
∴8n – 1 is divisible by 7 for all values of n Choice (C)
 11   11 
25. When 21 is divided by 7 the remainder is 2 Ι is true.
When 22 is divided by 7 the remainder is 4 ΙΙ. 1051 = (7 + 3)51 = (7 + 3) multiplied 51 times.
When 23 is divided by 7 the remainder is 1 (7 + 3) (7 + 3) = M(7), where M(7) denotes an unspecified
When 24 is divided by 7 the remainder is 2 multiple of 7+ 32
∴the cycle is 2,4,1 its period is 3 (7 + 3)3 = (7 + 3) (M(7)+ 3)
when 63/3, remainder is 0 i.e., 21 cycles. = A multiple of M(7) + 33.
∴The remainder is 1 Ans: (1) It follows in general that (7 + 3)N = M(7) + 3N
1051 = M(7)+ 351.
Exercise – 2(a)  51   51   3 17 
Rem  10  = Rem  3  = Rem  (3 ) 
 7   7   7 
Solutions for questions 1 to 25:      
 2717 
 = Rem ( 28 − 1)
17
1. (a) The units digit of 8 repeats after every four powers. = Rem 
Expressing 173 in terms of 4, we have 8173 = 84 x 43 + 1  7  7
 
and hence the last digit of 8173 and 81 should be the
same. Hence 8173 has the units digit of 8.  ( −1)17 
= Rem   = −1
Choice (C)  7 
 
(b) Last digit of 518163 is the same as the last digit of 8163; ∴ 2717 is 1 less than a multiple of 7, (or 6 more than a
8163 = 8(4 x 40) + 3. Since the last digit of the power of 8 is multiple of 7) while 750 = 7 (107) + 1
3, 8163 will have the same units digit as 83 whose last
digit is 2, 142157 will have same units digit as 2157 2157 ∴ Rem  N  = Rem  (M(7) + 6 )− (7(10 7) + 1)  = Rem  5  = 5.
 
= 2(4 x 39 +1) cycle for the last digit of power of 2 is also 7  7  7
4. ΙΙ is true. Both Ι and ΙΙ are true. Choice (C)
Hence 2157 will have the same units digit as 21 whose
units digit is 2.Hence 518163 + 142157 will have the last Note: 1051 = 251 × 551. Remainders of 2N divided by 7 have
digit of 2 + 2 = 4 Choice (B)
 51 
(c) 1567143 has the same last digit as 7143 a cycle of 3. ∴ Rem  2  can be found ----- (1)
 7 
7143 = 7(4 x 35) + 3  
Since the last digit of the power of 7 has a cycle of 4, 7143  551   3 17   17 
will have the same last digit as 73 i.e., 3. Rem   = Rem  (5 )  = Rem  (18 (7) − 1) 
 7   7   7 
1239197 has the same last digit as 9197.      
For 9197, since power of 9 is odd its last digit is 9. = (−1)17 = −1 --------------(2)
Hence, 1239197 has last digit of 9.
25661027 has the same last digit as 61027, i.e., a 6, From (1) and (2), Rem  N  can be found.
since 6 raised to any power will always have a last 7
digit of 6.
Hence, last digit of 1567143 x 1239197 x 25661027 will be 7. Let E = 58n+4 + 44n+2 – 10
the last digit of 3 x 9 x 6 = 162
i.e., 2 Choice (A) ( )
= 54
2n +1
( )
+ 42
2n +1
– 10
= 6252n+1 + 162n+1 – 10
2. This is of the form (43)a – (21)a, where a = 5n. This is aN + bN is always divisible by a + b when N is odd.
always divisible by 43 – 21 = 22. 6252n+1 + 162n+1 is divisible by 641.
Hence it is also divisible by 11. Choice (A)
Rem 
E 
 = –10. This is equivalent to the positive
3. By observation, factorial of any number greater than 6, is  641 
divisible by 7. remainder of 631. Choice (C)
∴The effective remainder of 1! + 2! + 3! + . . . . . + 49! is
nothing but the reminder obtained when 1! + 2! + 3! + 4! + 8. P and Q have the respective forms 20k + 1 and 20k + 2
5! + 6! is divided by 7. ∴ They have the respective forms 4k1 + 1 and 4k1 + 2
1! + 2! + 3! . . . . 6! = 873 (∵ k1 = 5k)
The remainder when 873 is divided by 7 is 5. Ans: (5)
Units digits of the power of 2 and 8 have cycles of 4 each.
4. Given dividend is 3147 and divisor is 11. ∴ 2P and 8Q have respective units digits of those of 21and
Looking at the remainders when 3147 is divided by 11, they 82 i.e 2 and 4.
are as follows. Ι: As P has the form 20k + 1, its units digit is 1. ∴The units
For 31 = 3; 32 = 9; 33 = 5; 34 = 4; 35 = 1 digit of 2P is 2.
∴For every 5 powers remainders are repeated. Both 2P and 2P have the same units digit.
∴ Ι is true.
3147 35 × 29 + 2 32 ΙΙ. 8Q has units digit of 6.
∴ = , Remainder is that of = , i.e., 9
11 11 11 (8Q + 8Q) ends with 0 i.e 8Q + 8Q is divisible by 10.
Choice (B) ∴ ΙΙ is true.
Both Ι and ΙΙ are true. Choice (C)
5. 213 + 233 + 253 + 273 = (24−3)3 + (24+ 3)3 + (24−1)3 + (24+1)3
= 2[243 + 3(24) (32)] + 2[243 + 3(24) (1)2] 9. N = ((48) (98) + 7) ((48) (98) + 9) ((48) (98) + 11)
= 2[2(243) + 3(24) (10)] = 96[242 + 15] ∴ Required remainder = Remainder when (7) (9) (11)
∴ (213 + 233 + 253 + 273) when divided by 96 leaves a divided by 48 = 21 Ans: (21)
remainder of 0. Ans: (0)
10. As (a+b)3 − a3 − b3 = 3ab(a+b), N = 1613 − 773 − 843
6. N = 1051 − 750 = 3(77)(84)(161) = 32(7)3(11)(4)(23)
Ι: The remainder of 10n, where n is any odd number, when ∴ N is divisible by 4, 23, 11, 7 but not by 8. Choice (D) is false.
divided by 11 is always 10. Choice (D)

Triumphant Institute of Management Education Pvt. Ltd. (T.I.M.E.) HO: 95B, 2nd Floor, Siddamsetty Complex, Secunderabad – 500 003.
Tel : 040–27898195 Fax : 040–27847334 email : info@time4education.com website : www.time4education.com SM1001962/15
11. Let N = 7923, 7923,……… 7923 (comprising 400 digits or 100100 1100
100 groups) Similarly, Rem = Rem =1
= 79,23,79,23,. ………… 79,23 (comprising 200 groups) 99 99
N 23(100 ) − 79(100) 98100 + 100100
Re m = Re m = 23 (–1) – 79(–1) ∴ Rem =2 Choice (A)
101 101 99
= 56. (∵ By remainder theorem) Ans: (56) 18. (1 – 3x + x2)55 = 1 + a1x + a2x2 + ……. + a110 x100
Setting x = 1, 1 + a1 + a2+ ------- a110 = (– 1)55 = –1
12. Let N= 2424………… 24(comprising 300 digits) Choice (B)
= 242, 424, ……242, 424 (comprising 100 groups) 123
N 424(50 ) + 242(50 ) 19. We need Re m 2 . As the index is close to a multiple of
Re m = Re m 61
999 999
the divisor which is prime, we think of Fermat’s Little
666(50 ) 33,300
= Re m = Re m = 33 + 300 = 333. 2 60 2120
999 999 theorem = Re m = 1 ⇒ Re m =1
Choice (A) 61 61
2123 23
2
∴ Re m = Re m = 8. Ans: (8)
N 73 61 61
13. Let N = 73382 Re m = Re m (We can leave out all
100 100
5329 20. Let R = Rem 10
400
(
= Rem )
(10198 )210 4
= Re m
10,000
the 20’s in the index) = Re m = 29. Choice (C) 199 199 199
100
50(200 )
= Re m = 50 (1) = 50 Choice (D)
200 − 1
N 87 760 +17 8717
14. Let N = 787777. So, Rem = Rem = Rem

872 ≡ 69
100 100 100
21. Re m
14 400
= Rem
14 14399
= 7 Rem
( )
2 14 399 ( ) = M(say)
1393 7(199 ) 199
692 ≡ 61
612 ≡ 21
Re m
(
2 14 399
= 2 Re m
)(14198 )2 14 3
= 2 Re m
(14 )(196 )
212 ≡ 41 199 199 199
∴ 8717 = 8716 87 ≡ (41)(87) ≡ 67 ≡ 2(14) (–3) ≡ –84
Alternate method:
∴ Re m
(
2 14 399 )
= 199 – 84 = 115
787 777 87 17 199
Re m = Re m ∴ M = 7(115) = 805. Choice (B)
100 100
(87) (87) ≡ 87 ≡ 1 ⇒ 87 ≡ 87 = (87 )
17 3 20 17 –3 –1 3 22. By Wilson’s Theorem, 96! = 97k + 96
Where 87–1 is not 1/87, but the inverse of 87 or that number ∴ 97! = 972k + 96(97) and 100! = 98(99) (100) [972k + 96(97)]
96(97 )(98 )(99 )(100 )
(or one number), which when multiplied by 87 produces ∴ Re m 100! = Re m
1(i.e. produces a number of the form 100k + 1) 97 2 97 2
This is an LCM model 3 problem. We are looking for
96(98 )(99 )(100 )
a multiple of 87 which leaves a remainder of 1, when = 97 Re m
divided by 100 97
i.e. 87x = 100y + 1 -------- (1) ≡ 97 (–1) (1) (2) (3) ≡ – 582 ≡ 972 – 582
⇒ 13y = 87y1 + 86 ------(2) (See N1) = 9409 – 582 = 8827. Choice (D)
⇒ 9y1 = 13y2 + 5 ------- (3) (See N2) 23. By Wilson’s Theorem 46! = 47k + 46
⇒ 4y2 = 9y3 + 4 ⇒ 45! 46 = 47k + 46
y3 = 0 ⇒ y2 = 1 ⇒ y1 = 2 ⇒ y = 20 ⇒ x = 23 where k is an integer.
i.e. the inverse of 87 is 23 ∴ 87-3 ≡ 233 ≡ (29) (23) ≡ 67 It follows from this equation that k must be a multiple of 46.
Choice (D) ∴ Dividing by 46 on both sides, we get 45! = 47 (an int) + 1.
Note: (N1) As the second term on the RHS of (1) which
45!
leaves a remainder of 1 when divided by 87, the first term ∴ Re m = 1. Ans: (1)
i.e. 100y (as equivalently 13y) leaves a remainder of 86. 47
(N2) As the second term of the RHS of (2) leaves a 24. 81(6425) = 81(8)50 = 81 (9 – 1)50
remainder of 8 when divided by 13, the first term leaves a = 81[950 – 50C1 (9)49 + 50C2 (9)48…….+50C48(92) – 50C1 (9) + 1]
remainder of 5. = A multiple of 94 + 81(1 – 50(9))
Similarly, we can keep decreasing the coefficients until they = A multiple of 94 – 36369
are small enough for us to see the solution. [94 = (6561) and 94(6) = 39366 and –36369 = –39366 +
2997].
N 48 8 ∴ 81(6425) = A multiple of 94 – 94(6) + 2997 = A multiple of
15. Let N = 948728 Re m = Re m
100 100 94 + 2997
( 2304 ) 4 44 ∴ Remainder is 2997.
= Re m = Re m = 56. Ans: (56)
100 100 Alternative method:
81(64 25 ) 64 25 8 50
6 6 Rem = 81 Rem = 81 Rem
N 26 4 81 81
16. Let N = 674586 Re m = Re m 74 = Re m 9
100 100 100
= 81 Rem
(9 − 1)50
3
= Re m 76 = 76. Choice (B) 81
100 (9 – 1)50 =
950 – 50C1 9491 + …. + 50C48 92(148) – 50C49 9 (149) + 150
98100 (99 − 1)100 (9 − 1)50 = − Re m 50(9 ) − 1 449
17. Rem = Rem ∴Rem = − Re m = − 44
99 99 81 81 81
99k + ( −1)100 which is equivalent to 37.
= Rem =1 ∴ The required remainder is 37(81) = 2997 Choice (B)
99
Triumphant Institute of Management Education Pvt. Ltd. (T.I.M.E.) HO: 95B, 2nd Floor, Siddamsetty Complex, Secunderabad – 500 003.
Tel : 040–27898195 Fax : 040–27847334 email : info@time4education.com website : www.time4education.com SM1001962/16
25. (7N + N3) ends with 0. ∴ It is even. 7N is odd. As powers of 767 will have a cyclicity of 20, we can find
∴ N3 must also be odd. ∴ N must be odd.  9
∴ N has the form 4K + 1 or 4K + 3 Rem  (767 ) 
If N has the form 4K + 1, 7N ends with 7.  100 

= Rem [67 ] (only last 2 digits affect the last 2 digits)


∴ N3 must end with 3 and N must end with 7. 9
(ie., N = 4k + 1 = 10p + 7) 100
∴ N is 17, 37, 57, 77 or 97. 671 has last 2 digits = 67
If N has the form 4K + 3, 7N ends with 3. ∴ N3 must end 672 has last 2 digits = 89 (672)
with 7. ∴ N must end with 3. (i.e., N = 4k + 3 = 10p + 3) 674 has last 2 digits = 21 (892)
N is 3, 23, 43, 63, or 83. 678 has last 2 digits = 41 (212)
N has a total of 10 values. Ans: (10) 679 has last 2 digits = 47 (41 × 67)
N 47
Exercise – 2(b) ⇒ Rem   = Rem Re m[N / 100] = = 22
 25  25 25
Solutions for questions 1 to 35: Choice (D)

1. 314779 + 149138 has the same units digit as that of 4779 + 9138 9. All powers of numbers ending in 76 end in 76.
which has the same units digit as that of 41+92 whose units Choice (A)
digit is 4 + 1 = 5. Ans: (5)
N 14
= Re m 68
2. 269/5 = 269/22 + 1 =
2x2 68
=
( )
2x 2 2 34
10. Let N = 7681234 Re m
100 100
22 + 1 22 + 1 3214
= Re m (Q 68 ≡ – 32)
Since 22 + 1 divides 2 ( )
22
34
, replacing 22 by (– 1), the 100
remainder is 2(– 1)34 = 2 Choice (A) 2 70 210
= Re m = Re m = 24. Choice (A)
100 100
3. As the remainder is odd, it can be 1 or 3 only.
∴ The prime numbers must be of the form 5k + 1 or 5k + 3, N 9419
where k is a whole number. 11. Let N = 994499 , Re m = Re m
5k ends with a 0 and 5. 100 100
∴ 5k + 1 ends with a 1 and 6.
5k + 3 ends with a 3 or 8. Method 1:
As 5k + 1 and 5k + 3 must be prime, they must end with (94)19(94) ends in 76. Any even number raised to the power
1 and 3 respectively. of any multiple of 20 ends in 76. We can try out the options.
∴5k + 1 has 5 possibilities (11, 31, 41, 61, 71) and 5k + 3 04(94) ≡ 76 (We need not actually try out the other options)
has 7 possibilities (3, 13, 23, 43, 53, 73, 83). 24(94) ≡ 56
∴ 12 such prime numbers exist. Ans: (12) 64(94) ≡16
84(94) ≡ 96
4. Consider 2125/33 ∴ 9419 ends in 04.
= (25)25/25 – (–1) = –1
Actual remainder = –1 + 33 = 32, as divisor is 33. Method 2:
In the case of 2125/11, we divide 32 by 11 and get 10 as the Any (non-zero) even number raised to 20 ends with 76.
remainder Choice (B) ∴ 9420 ends with 76.
9419 ends with 4. (∵ 419 = 4 raised to an odd number and
5. Let N = 987654, 987654, ………. 987654
hence ends with 4).
(comprising 750 digits or 125 groups)
Let the tens digit of 9419 be x. Then 76 = last two digits of
= 987, 654, 987, 654, ……..987, 654,
x4 × 94 = (10x + 4) (90 + 4). The tens digit of this product
(comprising 250 groups)
= units digit (7 + y) where y is the units digit of 4x..
N (987 + 654)125
Re m = Re m ∴ 7 + y = 7. ∴ y = 0.
999 999 ∴ x = 0 or 5. But if x = 5, 9419 will not be divisible by 4.
(1641)(125 )  1641 125  ∴ x = 0. ∴ 9419 ends with 04. Ans: (4)
= Re m = Re m Re m Re m 
999  999 999 
99 99 99
(642)(125 ) ( 640)(125 ) + 250 12. (3 + 2x)99 = C 0 399 + C1 398 (2x) + …….. + C99 (2x)99
= Re m = Re m
999 999 Setting x = 1,
80,250 we get 99 C 0 399 + 99 C1 (2) + ….. + 99 C99 (2)99 = 599.
= Re m = 80 + 250 = 330. Choice (C)
999 Choice (A)

6. Let N = 445, 445, ………445 739 739


13. N = 624 = (625 – 1)
(comprising 525 digits or 175 groups)
624 739 625k + ( −1) 739
U = 445 (88) while Th = 445(87) ∴ Re m = Rem = – 1 ≡ 124.
445(88 ) − 445(87 ) 125 125
∴ Re m N = Re m = 445. Ans: (124)
1001 1001
Choice (C)
17 325
14. We need Re m . As the index is close to a multiple of

7. 71000 72
= 2
( )500 109
the divisor, which is prime, we apply Fermat’s theorem
50 7 − ( −1) 17108 17 324
Re m = 1 ⇒ Re m =1
By remainder theorem, remainder = (−1)500 = 1 Ans: (1) 109 109

8. N = (767)1009 17 325
⇒ Re m = 17.
Rem  N  = Rem
[Re m [N / 100 ]] 109
25 Choice (B)
 25 
Triumphant Institute of Management Education Pvt. Ltd. (T.I.M.E.) HO: 95B, 2nd Floor, Siddamsetty Complex, Secunderabad – 500 003.
Tel : 040–27898195 Fax : 040–27847334 email : info@time4education.com website : www.time4education.com SM1001962/17
15.
12 433
=
(
12 12 432 )
=
2(1272 )6 24. 2924 = 2920 . 24 = (210)92 . 24 = (1024)92 . 16.
Its last two digits are that of 2492 . 16.
438 6(73) 73 2492 = (242)46 = (576)46.
12 433 2(1272 ) 6 ∴ Last two digits of 2924are those of 7646 . 16.
∴ Re m = 6. Re m = 12. Last two digits of 76N where N is any natural number are 76.
438 73
Last two digits of 7646 are 76.
Choice (A)
∴Last two digits of 2924 = those of (76) (16) i.e 16.

10 2000 Alternate method:


16. We need R = Re m . Now 2000 = 18 (111) + 2
19
Last two digits of 2N, where N ≥ 2, show a cyclic pattern,
(1018 )111 10 2 10 2
∴ Re m = Re m = 5. with cycle length 20.
19 19 ∴ Last two digits of 2924, 2924-20, 2224-20(2), …..2924-20(46) i.e 24
Choice (B) are all the same.
∴ Last two digits of 2924 are 16. Choice (A)
17. By Wilson’s Theorem, 18! = 19k + 18
∴ 19! = 361k + 18(19) = 361k + 342. Ans: (342) 25. P2 + 7Q = 26. 56 = 106.
Q is odd. ∴ It has the form 4k + 1 or 4k + 3.
18. By Wilson’s Theorem, 27!(28) = 29k + 28 If it has the form 4k + 1, 7Q ends with 7. If it has the form
where k is an integer. It follows from this equation that k 4k + 3, 7Q ends with 3.
must be a multiple of 28. 7Q ends with 7 or 3. Also RHS of the given equation ends
∴ Dividing by 28 both sides, we get 27! = 29 (an int) + 1. with 0.
27! ∴ P2 ends with 3 or 7. But P is an integer i.e. P2 is a perfect
∴ Re m =1 Choice (C)
29 square.∴ P2 cannot end with 3 or 7.
∴ P has no possible value. Ans: (0)
19. 7900 = (8 – 1)900
(8 – 1)2 = (8 – 1) (8 – 1) = 82 – 2(8) + 1 = 8k2 + 1
(8 - 1)3 = (8k2 + 1) (8 – 1) = 8k3 – 1 26. 105 = 3 (5) (7)
(8 – 1)4 = (8k3 – 1) (8 – 1) = 8k4 + 1  2168   (3 − 1)168 
We can see that (8 – 1)N where N is any positive integer Rem   = Rem  
 3   3 
has the form 8k + (–1)N ∴ 7900 = 8k + 1    
Also 908 = 8(113) + 4  ( −1)168 
∴7900 – 908 = (8k + 1) – (8m + 4) = 8(k – 1 – m) + 5 = Rem   = 1 -----(1)
 3 
∴ The remainder is 5 Choice (C)  

20. a3 + b3 = (a + b) (a2 + b2 – ab). Also 25 + 31 = 27 + 29  2168   484   (5 − 1)84 


Rem   = Rem   = Rem   = 1 − (2)
= 56 and 112 = 56 (2)  5   5   5 
     
253 + 313 = 56 [252 + 312 – 25 × (31)] = 56 (an odd number)
Similarly 273 + 293 = 56 (an odd number)
Remainders of 2N divided by 7 have a cycle of 3.
253 + 313 + 273 + 293 = 56 (an even number)
= A number divisible by 112. ∴ Remainder is 0.  2168   56 ×3 
∴ Rem   = Rem  2 
Choice (D)  7  3 
   
21. A zero at the end of any number will result from the product  3
of a 2 and a 5. As A has only 1 multiple of 2(i.e.2 itself) and = Rem  2  = 1 ------(3)
 7 
1 multiple of 5 (i.e. 5), B will have only 1 zero at its end.  
Ans: (1) From (1), (2), (3), 2168 − 1 is divisible by 3, 5, 7 and hence
by their L.C.M i.e., 105.
22. Units digit of the factorial of any natural number which is
 168 
5 or more is 0. Required units digit = units digit of 1! + 2! + ∴ Rem  2  = 1 Choice (D)
3! + 4! = 1 + 2 + 6 + 4 = 3. Ans: (3)  105 
 
23. 422 = 2(211)
If p is any prime number, (p − 1)! + 1 is a multiple of p. (40 ) − (39 ) =
37 2 37 2 40 37 − 3937
(40 + 39 )(40 + 39 )
(Wilson’s Theorem) 27. x =
211 is prime.
36 36 37
4036 + 39 36
37

N N N–1 N–2
∴ 210! + 1 is a multiple of 211. a – b = (a – b) (a +a b + …..abN – 2 + bN – 1)
210! + 1 = 211k (say) ∴ 4037 – 3937 = (1) [4036 + 4035 (39) + ….40(39)35 + 3936)
210! = 211k − 1 -------- (1)
LHS is even and ∴ RHS must be even. ∴ k is odd. RHS is more than 4036 + 3936. ∴ LHS is also more than
Let k = 2k1 + 1 40 + 39 . ∴ x > 1.
36 36
Choice (D)
210! = 422k1 + 210
LHS is divisible by 210. 28. Each term in the dividend has the form (x + 1)2 x!
∴ RHS must be divisible by 210. (x + 1)2 x! = (x + 1) (x + 1) ! = (x + 2 – 1) (x + 1) !
∴ 422k1 must be divisible by 210. = (x + 2)! – (x + 1)!
∴ k1 must be divisible by 105. Required remainder is that of (19! – 18! + 18! – 17! + …. +
Letk1 = 105k2 3! – 2!) divided by 19 i.e. of (19! – 2) divided by 19. This is
210! = 422 (105k2) + 210 equivalent to – 2. (It is equal to 19 – 2 = 17) Ans: (17)
Dividing both sides by 210, 209! = 211k2 + 1
k2 must be odd. Let k2 = 2k3 + 1 29. (7020)118 + (8040)59 = 702360 + 802360 = 102360 (72360 + 82360)
209 ! = 211 (2k3 + 1) + 1 = 422 k3 + 212. = (72360 + 82360) followed by 2360 zeros
Alternately, 210! +1 is a multiple of 211 (Wilson’s Thorem). The rightmost non zero digit comes from 72360 + 82360
Subtracting 211 from 210! +1 ⇒ 210! – 210 is also a Units digit of 72360 = that of 74 = 1(cycle of 4)
multiple of 211. As 210 is co-prime to 211, 209! – 1 is Units digit of 82360 = that of 84 = 6(cycle of 4)
Units digit of 72360 + 82360 = 7
divisible by 211 ⇒ 209! leaves 212 as remainder.
Rightmost nonzero digit is 7. Choice (C)
Choice (A)
Triumphant Institute of Management Education Pvt. Ltd. (T.I.M.E.) HO: 95B, 2nd Floor, Siddamsetty Complex, Secunderabad – 500 003.
Tel : 040–27898195 Fax : 040–27847334 email : info@time4education.com website : www.time4education.com SM1001962/18
30. The least factorial divisible by 18 is 6! i.e., 720. (b) The cube of any prime number greater than 3 leaves
∴ All higher factorials are divisible by 18. a remainder of 1 or 5. 23 when divided by 6 leaves
∴ The remainder of the sum divided by 18 is the remainder a remainder of 2. 33 when divided by 6 leaves
of (1! + 2! + 3! + 4! + 5!) divided by 18. This equals 9. a remainder of 3.
Choice (A) ∴ The remainder cannot be 4. Choice (D)

31. Rem  7777777777  = Re m 7777  (The last 4 digits of the


Chapter – 3
 16   16  (Number Systems)
given number) = 1 Concept Review Questions
 7 262   2(131)   2 131 
Re m  = Re m 7  = Re m (7 )  Solutions for questions 1 to 20:
 16   16   16 
     
 (48 + 1)131  1. To express a number in binary, we use the digits 0 and 1.
 131 
= Re m  = Re m 1  = 1 Choice (C)
 16   16 
   
2. In octal system we have eight digits. Ans: (8)
[∵ (48 + 1)131 of the form 48k + 1131]
3. In duodecimal system, B is 11. Ans: (11)
1+1
Required remainder is Re m =2 Choice (D)
16 4. In hexadecimal system we use 16 digits. Ans: (16)
5. 2 12
32. Given, 2 6–0
α = 1883 + 2003 + 2113 + 2993, which is an even number. 2 3–0
When an even number is divided by an even number, the 1–1
remainder would always be even. It can be neither 23, nor
37, nor 9. Choice (D) ∴12 = (1100)2 Choice (B)

33. N = 757677 ………99100101…….119120 6. 12 1221


The remainder of any large number divided by 9 can be 12 101 –9
found easily by the following procedure. We break up the 8–5
number into a number of parts and add the numbers in
each part. We then find the remainder of the sum divided ∴ 1221 = (859)12
by 9. This remainder R (say) equals the remainder of the ∴ n = 859 Ans: (859)
large number divided by 9. (This also means that the value
of R is independent of the breaking points). 7. In septeunary system we use the digits 0, 1, 2, 3, 4, 5, 6.
We see that 75 + 120 = 78 + 119 = ….= 97 + 98 = 195 ∴ highest digit is 6. Choice (A)
∴ We would find it convenient to break after 75, 76 …..120
N  (195 )(x )  8. 16 2346
Re m  = Re m  where x is the number of pairs 16 146 – A
9  9  9–2
that have been formed.
X = 23. ∴ (2346)10 = (92A)16
N  (195 )(23 )   (189 + 6)(23 )  ∴ x = 92A Choice (B)
Re m  = Re m  = Re m 
9  9   9  9. 8 13
6(23) 1–5
= Re m =3
9 (13)10 = (15)8 Choice (A)

Alternative method: 10. (121)8 = (1 × 8 + 2 × 8 + 1 × 8 )10 = (81)10


2 0

Let N = 757677 …..99100101 ……..120. 2 81


Let M = 75 + 76 +……+ 120 2 40 – 1
We can think of M rather than N. (They have the same 9’s 2 20 – 0
remainder) M is the sum of 46 consecutive numbers. 2 10 – 0
If n is an odd number, the sum of n consecutive integers is 2 5–0
divisible by n (Also if n is an even number, say n = 2m, the 2 2–1
sum of n consecutive integers leaves a remainder of m) 2 1–0
∴The sum of 9 (and also 45) consecutive integers is
divisible by 9. ∴In M, which has 46 consecutive integers, ∴ (121)8 = (81)10 = (1010001)2
we can leave out the first 45 and think of only 120, or leave Choice (C)
out all the numbers from 76 to 120 and think of only 75.
M 75 120 11. (3AB)12 = (3 × 122 + A × 12 +B × 120)
i.e, Rem = Rem = Rem =3 = (3 × 144 + 10 × 12 + 11 × 1)10 = (563)10
9 9 9
∴ x = 563 Ans: (563)
Ans: (3)
12. (ACD)16 = (A × 162 + C × 16 + D × 160)
34. Successive powers of 7 leave the following remainders,
= (10 × 256 + 12 × 16 + 13 × 1)10
when divided by 5.
= (2765)10 Choice (A)
2, 4, 3, 1 ; 2, 4, 3, 1 ; and so on.
1000
7 13. (121)10 is a perfect square.
As 1000 = 250(4), Rem = 1. Choice (A)
5 (171)8 = (1 × 82 + 7 × 8 + 1 × 80) = (121)10 is a perfect square
(A1)12 = (A × 12 + 1 × 120)10 = 120 + 1 = 121 is a perfect
35. (a) The square of any prime number greater than 3 when square Choice (D)
divided by 6 leaves a remainder of 1. 22 when divided
by 6 leaves a remainder of 4. 32 when divided by 6 14. The numerical value of (1.001)2 = (1 × 20 + 0 × 2–1 + 0 × 2–2
leaves a remainder of 3. + 1 × 2–3)10 = (1 + 0 + 0 + 1/8)
∴ The remainder cannot be 5. Choice (D) = 1 + 0.125 = (1.125)10 Ans: (1.125)

Triumphant Institute of Management Education Pvt. Ltd. (T.I.M.E.) HO: 95B, 2nd Floor, Siddamsetty Complex, Secunderabad – 500 003.
Tel : 040–27898195 Fax : 040–27847334 email : info@time4education.com website : www.time4education.com SM1001962/19
15. 11011 3. The given number is (523)10
101
1101 16 523
111111 16 32 – 11
-------------- 2–0
1010001
-------------- Choice (B) ∴(523)10 = (20B)
since 11 = B in the hexadecimal system Choice (B)
16. (34)7 + (25)7 = (62)7 Choice (C)
17. 34 4. The given number is (2776)10
– 25
--------- 12 2776
5 12 231 – 4
--------- 12 19 – 3
∴ (34)6 – (25)6 = (5)7 Choice (D) 1–7

18. The largest digit in hexadecimal system is F. ∴ (2776)8 = (1734)12 Choice (B)
∴ Three digit largest number is (FFF)16 Choice (C)
5. The given octal number is (7464)8.
19. Every even number is divisible by 2. The remainder will be We write each digit as a block of 3 binary digits.
zero, which is the end digit of the binary number. Accordingly we get (111100110100)8. We now group the
∴ The binary representation of even numbers always end digits as blocks of 4 from right to left.
with zero. Choice (B) = (1111 0011 0100)16 = ((1111)2 (0011)2 (0100)2)16
20. = (F34)16. Choice (A)
2 247
6. The given number is (110001110)2.
2 123 1 We group the digits in blocks of 3 and find the octal
2 61 1 equivalent for each block of 3 digits
2 30 1 (110 001 110)2 = ((110)2 (001)2 (110)2)8
= (616)8 Choice (D)
2 15 0
2 7 1 7. The given number is (1100111011011)2.
2 3 1 We group the digits in groups of 4 and find the
hexa-decimal equivalent for each group of 4 digits.
1 1 (0001 1001 1101 1011)2
= ((0001)2 (1001)2 (1101)2 (1011)2)16
∴ (247)10 = (11110111)2 = (19DB)16 Choice (D)
∴ Option (A) is true
8. The given binary number is (1101.0101)2
8 247 Integer part
1 × 20 + 0 × 21 + 1 × 22 + 1 × 23 = 1 + 4 + 8 = 13
8 30 7 Fraction part
0 × 2–1 + 1 × 2–2 + 0 × 2–3 + 1 × 2–4
3 6
= 0.25 + 0.0625 = 0.3125
∴ The decimal number is 13.3125 Ans: (13.3125)
∴ (247)10 = (367)8
Option (B) is true 9. The given number is (BAD)16
12 247 We know that B = 11, A = 10 and D = 13 in the hexa-
decimal system. ∴The decimal number is
12 20 7 = 11 × 162 + 10 × 16 + 13 × 1 = 2816 + 160 + 13 = (2989)10
Choice (B)
1 8

∴ (247)10 = (187)12 10. (1101)2 + (46)8 + (97)10


Option (C) is true. Choice (D) Now, (1101)2 = 1 × 20 + 0 × 21 + 1 × 22 + 1 × 23
= 1 + 4 + 8 = (13)10
Exercise – 3(a) (46)8 = 4 × 81 + 6 × 80 = 32 + 6 = (38)10 ∴We get (13)10 +
(38)10 + (97)10= (148)10
Solutions for questions 1 to 25:
Choice (A)
1. The given number is (176)10
11. The given decimal number is 256
2 176 The minimum number of bits required to represent 256 in
2 88 – 0 the binary system is the number of digits in the binary
2 44 – 0 representation of 256.
2 22 – 0
2 11 – 0 2 256
2 5–1 2 128 – 0
2 2–1 2 64 – 0
1–0 2 32 – 0
∴ (176)10 = (10110000)2 Choice (C) 2 16 – 0
2 8–0
2. The given number is (472)10 2 4–0
2 2–0
8 472 1–0
8 59 – 0
7–3 ∴ (256)10 = (100000000)2
∴ (472)10 = (730)8 Ans: (730) ∴ 9 bits are required Ans: (9)

Triumphant Institute of Management Education Pvt. Ltd. (T.I.M.E.) HO: 95B, 2nd Floor, Siddamsetty Complex, Secunderabad – 500 003.
Tel : 040–27898195 Fax : 040–27847334 email : info@time4education.com website : www.time4education.com SM1001962/20
12. (256)16 – (256)8 22. The L.C.M. of 2, 3, 4 and 5 is 60. The number 60 – 1 = 59
We convert both numbers to a common base i.e., in base 10 leaves a remainder of 1, 2, 3, 4 and 5 respectively. Hence,
(256)16 = 2 × 162 + 5 × 16 + 6 × 1 1, 2, 3, 4 would be the last digits in the bases 2, 3, 4 and 5.
= 512 + 80 + 6 = (598)10 Ans: (59)
(256)8 = 2 × 82 + 5 × 8 + 6 x 1
= 128 + 40 + 6 = (174)10 23. To find 3-digit numbers of this form we look at the numbers
∴ (598)10 – (174)10 = (424)10 Choice (C) of the form (60n − 1) between 100 and 1000. These are
15 in number. The numbers being (119, 179, ….. 959).
13. The given equation is Ans: (15)
(n)n + 2 + (n – 1)n + 1 + (n – 2)n + ……+ (1)3
Since the number is smaller than the given radix in each case, 24. If we look at the choices,
the number will not change even if the radix is changed to 10 choice (A):
For example (4)7 = (4)10 6 × 80 + 1 × 81 + 0 × 82 + 5 × 83 = 2560 + 8 + 6
Hence each term of the expression can be written as = 2574, which is not a perfect cube.
(n)10 + (n – 1)10 + (n – 2)10 + ……(1)10 choice (B): 6 × 90 + 1 × 91 + 0 × 92 + 5 × 93 = 3660, which is
= (n + (n – 1) + (n – 2) + ……+ 1)10 not a perfect cube
choice (C): 6 × 110 + 1 × 111 + 0 × 112 + 5 × 113
=  n(n + 1)  Choice (C) = 6672, which is not a perfect cube
 2 10 choice (D): 6 × 70 + 1 × 71 + 0 × 72 + 5 × 73 = 1728
as, 1728 = (12)3, 5016 is a perfect cube in base 7.
14. The number 378 when expressed as the sum of powers of Choice (D)
2 is 256 + 64 + 32 + 16 + 8 + 2. Thus we require 6 of these
weights. Ans: (6) 25. A.M of (12)6 and (33)7 is (10)n
(12)6 = 1 × 61 × 2 × 60 = (8)10
15. Since there exists only one way of converting the number (33)7 = 3 × 71 + 3 × 70 = 21 + 3 = (24)10
(378)10 in base 2, i.e., (378)10 = (101111010)2, only one A.M of (8)10 and (24)10 = (16)10
such combination exists. Ans: (1) (16)10 = (10)n
here n = 16 Ans: (16)
16. f((25)8, (25)10, (25)16)
(25)8 = 2 × 81 + 5 × 80 = (21)10 Exercise – 3(b)
(25)16 = 2 × 161 + 5 × 160 = 32 + 5 = (37)10
∴ f((21)10, (25)10, (37)10) Solutions for questions 1 to 30:
= 21 × 25 + 25 × 37 + 37 × 21 = (2227)10 Choice (B)
1.
17. Let the original binary number be (a1 a2 ….an)2 where a1, 2 108
a2,…….an = 0 or 1 depending upon the value of n. 2 54 – 0
∴ (a1 a2…….an)2 = an × 20 + an – 1 × 21 + …..a1 × 2n – 1 2 27 – 0
If we now concatenate 1 to the end we get, (a1 a2 …an 1)2 2 13 –1
= 1 × 20 + an × 21 + an – 1 × 22 + ….a1 . 2n 2 6–1
= 1 + 2(an × 20 + an – 1 × 21 + …..+ a1 × 2n – 1) 2 3–0
= 1 + 2((a1 a2…….an)2) 1–1
∴ The new number is 1 more than double the original
number. Choice (D)
(108)10 = (1101100)2 Choice (C)
18. The given number is (1161)8
2.
(1161)8 = 1 × 80 + 6 × 81 + 1 × 82 + 1 × 83 8 567
= 1 + 48 + 64 + 512 = (625)10 8 70 – 7
The square root of (625)10 is (25)10 8 8–6
(25)10 = (31)8 1–0
∴ (1161)8 = (31)8 Choice (D)
(567)10 = (1067)8 Choice (D)
19. (234)6 =
3. 12 1896
2 × 62 + 3 × 6 + 4 × 1 = 72 + 18 + 4 = (94)10
((94)10)2 = 94 × 94 = (8836)10 12 158 – 0
(8836)10 in base 6 form: 12 13 – 2
1–1
6 8836 (1896)10 = (1120)12 Choice (B)
6 1472 – 4
6 245 – 2 4.
6 40 – 5 16 894
6 6–4 16 55 – E
1–0
3–7
∴ (8836)10 = (104524)6 Choice (C) (894)10 = (37E)16 Choice (A)

20. (1000111) = 71 5. (7640)8 = 7 × 83 + 6 × 82 + 4 × 81 + 0 × 80


(101)2 = 5 = 3584 + 384 + 32
The remainder when 71 is divided by 5 is 1. = (4000)10
Hence, the remainder is 1 when 1000111 is divided by 101
Choice (B) 16 4000
16 250 – 0
21. (120)8 = 1 × 8 + 2 × 8 = 64 + 16 = (80)10
2 1
16 15 – A
(24)8 = 2 x 81 + 4 × 80 = (20)10 0–E
L.C.M. of (80)10 and (20)10 is (80)10 = (120)8
Choice (D) ∴ (7640)8 = (EA0)16 Choice (D)
nd
Triumphant Institute of Management Education Pvt. Ltd. (T.I.M.E.) HO: 95B, 2 Floor, Siddamsetty Complex, Secunderabad – 500 003.
Tel : 040–27898195 Fax : 040–27847334 email : info@time4education.com website : www.time4education.com SM1001962/21
6. Given (10101101011)2 16. (23232)4 in decimal system is 750. (232)4 in decimal
= (010101101011)2 system is 46.
= [(010)2 (101)2 (101)2 (011)2] ∴ The remainder, when 750 is divided with 46, is 14.
= (2553)8 Choice (B) (14)10 = (32)4 Choice (B)
7. (ABC)16 17. (210)6 = 0 + 6 × 1 + 62 × 2
= C × 160 + B ×16 + A × 162 = 6 + 72 = 78
= 12 × 1 + 11 × 16 + 10 × 256 (30)6 = 0 + 3 × 6 = 18
= 12 + 176 + 2560 = (2748)10 Ans: (2748) ∴ The L. C. M of 18, 78 is 234.
(234)10 = (1030)6 Choice (C)
8. Given (6555)x − (777)x = (5556)x
⇒ (6555)x = (5556)x + (777)x 18. (11)7 = 1 + 7 = 8
Consider unit digit we know 6 + 7 = 13 but we have 5 in unit (55)7 = 5 + 35 = 40
digit, k, 13 – 8 = 5, ∴ x should be 8. (404)7 = 4 + 0 × 7 + 4 × 72 = 200
∴ (5666)8 + (457)8 = (6345)8 Choice (D) ∴ 8, 40 and 200 are in G. P Choice (B)
9. (423)9 = 4 × 9 + 2 × 9 + 3 × 9
2 0
19. Given, (24)6 = 4 + 2 × 6 = 16
= 324 + 18 + 3 = 345
(34)7 = 4 + 3 × 7 = 25
(423)6 = 4 × 62 + 2 × 61 + 3 × 60
= 144 + 12 + 3 = 159 The geometric mean of 16 and 25 is 16 × 25 = 20
(423)9 − (423)6 = 345 − 159 Given, geometric mean = (24)n
= 186 = (136)12 Choice (C) (20)10 = (24)n
20 = 4 + 2n ⇒ 2n = 16 ⇒ n = 8 Ans: (8)
10. The smallest three-digit number in the base 12 system is
100. The largest three digit number in the base 12 system 20. From options (2454)11 = 3205; (2454)12 = 4096; (2454)9
is BBB. = 1831, (2454)6 = 610 and (2454)7 = 921
(100)12 = 122 = 144. ∴ 4096 is a perfect cube Choice (B)
(BBB)= 123 − 1 = 1727.
The number of three digit numbers 21.
= (1727 − 144) + 1 = 1584 Ans: (1584) 2 456
2 228 – 0
11. The smallest three-digit number in base 5 system is 100. 2 114 – 0
The largest three-digit number in base 5 system is (444)
(100)5 = 52 = 25 2 57 – 0
(444)5 = 53 − 1 = 125 = 1 = 124 2 28 – 1
The number of three-digit numbers in the base 5 system is 2 14 – 0
124 − 25 = 99 2 7– 0
Out of these 99 numbers, 24 numbers are also three-digit 2 3– 1
numbers in the base 10 system. 1– 1
∴ The number of three-digit numbers which are actually (456)10 = (11001000)2
two-digit numbers in the base 10 system is 99 − 24 = 75 ∴ 4 weights can be used. Ans: (4)
Ans: (75)
22. The minimum weight is 8 kg Ans: (8)
12. (111)2 + (222)3 + - - - - - + (666)7
9
= (111)2 = 23 − 1, (222)3 = 33 − 1, …………. 23. 512 = 2
(666)7 = 73 − 1 ∴ The minimum number of bits required is 10
∴ The given sum = 23 − 1 + 33 − 1 + 43 − 1 + - - - - + 73 − 1 Choice (A)
13 + 23 + 33 + - - - - 73 − 1 − 1 − 1 + – - - - -− 1 for 7 times 24. L.C. M of 2, 3, 4, 5 and 6 is 60
2
 7 (7 + 1)  ∴ The required number is
  − 7 = 784 − 7 = 777 = (777)10 Choice (C)
 2  L.C.M (2, 3, 4, 5, 6) − 1 = 60 − 1 = 59. Ans: (59)
25. The smallest three digit number is 60 + 59 = 119.
13. (13)5 = 3 + 5 = 8
All these numbers differ by 60.
(13)8 = 3 + 8 = 11
tn = a + (n − 1) d
(13)12 = 3 + 12 = 15
f ( (13)5, (13)8, (13)12 ) 119 + (n − 1) 60 < 1000
= f (8, 11, 15) = 82 + 112 + 152 60n < 1000 – 59
= 64 + 121 + 225 = 410 Choice (D) 60n < 941
n < 941
14. (310)16 = 0 + 1 × 16 + 3 × 162 60
= 0 + 16 + 768 = 784 n < 15.6
The square root of 784 is 28. ∴ The value of n is 15.
∴ The number of three digit numbers = 15 Ans: (15)
16 28
1 – 12 26. 0.375 × 2 = 0.750 → 0
0.75 × 2 = 1.50 →1
28 = (1C)16 0.5 × 2 = 1 →1
∴ The square root of (310)16 = (1C)16 Choice (C) ∴ (0.375)10 = (0.011)2 Choice (D)
15. (43)8 = 3 + 4 × 8 = 35 27. (13.34375)10
The square of 35 = 1225 Consider the integer part, i.e. 13.
8 1225
2 13
8 153 – 1 2 6–1
8 19 – 1
2 3–0
2–3 1–1
∴ The square of (43)8 = (2311)8 Choice (A) (13)10 = (1101)2

Triumphant Institute of Management Education Pvt. Ltd. (T.I.M.E.) HO: 95B, 2nd Floor, Siddamsetty Complex, Secunderabad – 500 003.
Tel : 040–27898195 Fax : 040–27847334 email : info@time4education.com website : www.time4education.com SM1001962/22
Consider the fractional part 1
(ii) Circumradius of an equilateral triangle is (side of
0.34375× 2 = 0.68750 → 0 3
0. 68750 × 2 = 1.3750 → 1 the triangle)
1. 3750 × 2 = 1.75000 → 0
0.75 × 2 = 1.5 → 1 ∴ Circumradius = 3 3 . Choice (A)
0.5 × 2 = 1.0 → 1
(13.34375)10 = (1101. 01011)2 Choice (A) 7. As I is the incentre,
∠A
28. Let the two digit number be (xy) ∠BIC = 90 + = 130º Ans: (130)
2
(xy)7 = y + 7x
(yx)7 = x + 7y 8. (i) In the given triangle, the square of each side is less
Given (xy)7 = 3 (yx)7 than the sum of the squares of the other 2 sides. Such
y + 7x = 3 (x + 7y) a triangle is acute angled.
7x − 3x = 21 y − y ∴ its circumcentre lies inside the triangle.
4x = 20 y Choice (A)
x = 5y
∴ y = 1 and x = 5 (ii) In the given triangle, the square of its greatest side
∴ The number is (51)7 exceeds the sum of the squares of the other 2 sides.
= 1 + 35 = (36)10 Choice (B) Such a triangle is obtuse angled.
∴ its orthocenter lies outside the triangle.
29. (11)2 + (11)3 + (11)4 + - - - - (11)n Choice (B)
3+4+5+---+n+1
=1+2+3+---+n+1−3 9. Incentre Choice (B)
(n + 1) (n + 2)
= −3 10. As XY || QR,
2
PX PY
n + 3n + 2 − 6 n + 3n − 4
2 2 = (Basic proportion ality theorem )
= = Choice (D) XQ YR
2 2 4 PY 16
= ; PY = cm Ans: (16)
6 8 3
30. Let the number be (ab)n = b + na
∴ (ab0)n = 0 + nb + n2 a = n (b + na)
11. Since triangle PQR is right angled at Q,
∴ n times the original number Choice (C)
QS = (PS )(SR ) = 8 cm Ans: (8)
Chapter – 4
(Geometry) 12. The medians in the triangle divide it into 6 triangles of equal
1
area. ∴ Area of Δ BGF = (18) = 3 sq cm Choice (B)
Concept Review Questions 6
Solutions for questions 1 to 45:
13. As AD bisects ∠BAC
1. The given triangle is isosceles BD AB
= ⇒ ∴ BD =
8
(4 )= 3.2 cm Choice (D)
The area of an isosceles triangle whose base is b cm and DC AC 10

b 4a2 − b2 14. ∠B = 2∠C


which has each of its equal sides as a cm =
4 ∠C = 180 − 130 = 50° ; ∴ ∠B = 100°
∠A = 180 − (∠B + ∠C) = 30° Choice (C)
12 4(10 )2 − 122
∴ Area of the triangle = = 48 sq cm
4 15. AD being the angular bisector,
Ans: (48) AB BD
AB : AC = BD : CD ⇒ =
AC CD
2. 152 + 202 = 252
∴ The triangle is right angled at the point of the intersection AC . BD 3 × 1.5
∴ CD = = = 2.25 cm Ans: (2.25)
of the sides 15 cm and 20 cm. AB 2
∴ required distance = 15 + 0 + 20 = 35 cm. Choice (D)
AD DE 1 AE
3. The given triangle is isosceles. In an isosceles triangle, the 16. = = = A
AB BC 2 AC
centroid, the orthocentre, the circumcentre and the incentre
As AC = 12,
all lie on the median to the base.
∴ The area of the required quadrilateral is 0. Choice (A) ∴ AE =
1
(AC) = 6 cm D E
2
4. In an equilateral triangle, the centroid and the orthocentre
coincide and the centroid divides each median in the ratio
2 : 1 or 1 : 2. B F C
As x > y, x : y = 2 : 1 Choice (C)
Choice (B)
5. In an equilateral triangle, the incentre, the centroid and the
circumcentre coincide. 17. AB is a right angled Triangle A
∴ the area of the required triangle is 0. Choice (D) ∴ BD = 12.5 cm
25
D
6. (i) Inradius of an equilateral triangle is 1 (side of the GD =
1
(BD) = 12.5 G
2 3 3 3
triangle)
= 4.17 cm B C
∴ inradius = 1 3 3 cm. 24
(9) = Choice (B)
2 3 2 Choice (B)
nd
Triumphant Institute of Management Education Pvt. Ltd. (T.I.M.E.) HO: 95B, 2 Floor, Siddamsetty Complex, Secunderabad – 500 003.
Tel : 040–27898195 Fax : 040–27847334 email : info@time4education.com website : www.time4education.com SM1001962/23
18. Since they have the same base, 360
b1 = b2 Each of its exterior angles =
n
h1 : h2 = 3 : 5
 180(n − 2)   360 
1 1
∴ Areas are in the ratio = b1 h1 : b2 h2   = 2 n 
 n   
2 2
180 n(n − 6) = 0 ; As n ≠ 0, n = 6
= b1 h1 = b2 h2 ⇒ h1 : h2 (∵b1 = b2)
∴ The polygon is a hexagon.
∴ Ratio = h1 : h2 = 3 : 5 Choice (C) Choice (B)

19. E 360° 180°


30. Exterior angle of a polygon = ⇒E= = 45°
n 8
Also as I + E = 180° ⇒ I = 180 – 45 = 135°.
Choice (C)

31. Let the angle be θ


F G Given 180° – θ = 3(90° – θ°)
H
From the figure 180° – θ° = 270° – 3 θ°
2 2
EH = EF – FH 2 2θ° = 90°
Also EH2 = EG2 – GH2 θ° = 45° Choice (A)
∴ EF2 – FH2 = EG2 – GH2 Choice (C)
32. Let the number of sides of the polygon be N.
20. ∠P + ∠Q + ∠R = 180° N (N − 3 )
Number of diagonals it has = = 20
∠R = 180° – ( ∠P + ∠Q) = 180° – (90° + 30°) = 60° 2
In a right angles triangle, the ratio of the sides opposite to N2 – 3N – 40 = 0
(N – 8) (N + 5) = 0
the angels 90°, 60° and 30° are in the ratio = 2 : 3 :1
N > 0; ∴ N = 8
In ∆ PQR, QR, PQ and PR are the sides opposite to the
 360 
90°, 60° and 30° angles respectively. Each exterior angle =  ° = 45°
 8 
∴ PQ : QR : PR = 3 : 2 : 1 Choice (B)
Choice (C)
21. As BD and CE are angular bisectors of ∠B & ∠C 33.
∠OBC = ∠OCB = 22.5° ( ∠B = 45°, ∠B = ∠C)
∠COD = ∠OBC + ∠OCB = 22.5 + 22.5 = 45°.
Ans: (45)
O
22. ∠BOC = 130° A .15
1 9
But ∠BOC = 90° + ∠A, A
2 M B
1 O
∴ 130° = 90° + ∠A,
2 Let AB be the chord with midpoint M.
∠A = 80° 130°
C
B AB = 2MB = 2 OB2 − OM2
Choice (B)
= 2 152 − 92 = 24 cm Ans: (24)
23. The centroid of a triangle is the point of concurrence of its
medians. Choice (D) 34. ∠POQ = 2∠PRQ
∴ ∠PRQ = 50° Choice (A)
24. Area of parallelogram = 2 (Area of triangle ABC)
= 48 cm2 Ans: (48) 35. The maximum number of tangents that can be drawn
is 3 (2 direct, 1 transverse). Ans: (3)
25. EF = 2
(12) + 1 (24) = 16 cm Choice (B)
36. The maximum number of tangents that can drawn is
1+ 2 1+ 2
4(2 direct, 2 transverse). Ans: (4)
26. A quadrilateral which is formed by joining the midpoints of 37. Only an isosceles trapezium is a cyclic quadrilateral among
another quadrilateral must be a parallelogram whose area the first 3 choices. Choice (C)
is half that of the original quadrilateral.
∴ the quadrilateral formed must be a parallelogram of area 38. As ∠BAC and ∠BDC are in the same segment,
40 sq cm. Choice (C)
∠BAC = ∠BDC = 30º
In Δ BAC
27. Sum of its interior angles = 180(n − 2)
∠BAC + ∠ABC + ∠ACB = 180º
Sum of its exterior angles = 360º
∠ABC = 180º − (30º + 60º) = 90º. Choice (C)
180(n − 2) ≤ 360
n ≤ 4 ; ∴ n = 3 or 4 Ans: (2) 39. As AOBC is a quadrilateral,
∠AOB + ∠OAC + ∠OBC + ∠ACB = 360º
28. Suppose the polygon has n sides
As AC and BC are tangents,
n(n − 3 ) ∠OAC = ∠OBC = 90º
Given, = 3n ⇒ n(n − 9) = 0
2 ∴ ∠AOB = 360º − (50º + 2(90º) = 130º.
As n ≠ 0, n = 9 Ans: (9) Choice (D)

29. Suppose the polygon has n sides 40. As ABCD is a cyclic quadrilateral,
( )
Each of its interior angles = 180 n − 2 ∠ABC + ∠ADC = 180º
n ∠ADC = 180º − 85º = 95º Choice (D)
nd
Triumphant Institute of Management Education Pvt. Ltd. (T.I.M.E.) HO: 95B, 2 Floor, Siddamsetty Complex, Secunderabad – 500 003.
Tel : 040–27898195 Fax : 040–27847334 email : info@time4education.com website : www.time4education.com SM1001962/24
41. PQR forms a semi circle As AE = EC, ∠EAC = ∠ECA = 30°
∴ ∠PRQ = 90º ∠DAE = 180° − [(80° + 20°) + (30° + 30°)] = 20°
∴ PR2 + RQ2 = PQ2 Choice (B)
PR = 30 2 − 18 2 = 24 cm Ans: (24)
5.
A
42. As AB and CD are parallel
∠HIC = ∠GAH = 130º
As GH and JL are parallel,
∠HIC = ∠JKL = 130º
As CD and EF are parallel,
∠JKT = ∠JLE = 130º Choice (B)
D B C
43. l // m ⇒ ∠6 = ∠3
Since D divides BC in the ratio 1 : 3 externally, if DB = x,
∴ ∠1 + ∠3 = 120°
BC = 2x.
∠1 = ∠3
AB2 − DB2 = AD²
∴ ∠1 = ∠3 = 60°
= AC2 − DC2 102 − x2 = 202 − (3x)2
∠4 = 180° − ∠3 = 120° Ans: (120)
100 − x2 = 400 − 9x2
44. Let AD be the median drawn from A to BC 300 10 3
8x2 = 300 ⇒ x = =
By Apollonius theorem 8 2 2
2(AD2 + BD2) = AB2 + AC2
2 10 3
BC = 2x = = 5 6 cm.
AD2 + 
BC  1
 = (AB2 + AC2) 2
 2  2
Choice (D)

( )
2
1 2 8
AD = 12 + 16 2 −   = 2 46 cm. 6. a (a + b + c) = d2 ––––– (1)
2 2 b (a + b + c) = e2 ––––– (2)
Choice (D) c (a + b + c) = f2 ––––– (3)
Adding (1) and (2) and subtracting (3),
45. (PQ) (PR) = (PS) (PT) a(a + b + c) + b (a + b + c) − c (a + b + c)
(4) (18) = 3(3 + ST) = (a + b − c) (a + b + c) = d2 + e2 − f2
ST = 21 cm Ans: (21) As a + b > c, d2 + e2 > f2
Similarly e2 + f2 > d2 and d2 + f2 > e2
Exercise – 4(a) ∴ triangle DEF is acute angled.
Choice (A)
Solutions for questions 1 to 40:
7.
1. Given ∠8 = 2∠1 B
∠8 + ∠1 = 180°
⇒ 2∠1 + ∠1= 180°
⇒ ∠1 = 60° and ∠8 = 120° D F
⇒ ∠8 and ∠4 are corresponding angles,
∠8 = ∠4 = 120° Ans: (120)

2. As PQ || RS, QAB + ABS = 180° E A C


As AD and ∠BD bisect ∠QAB and ∠SBA,
2 ∠BAD + 2 ∠DBA = 180°
∠BAD + ∠DBA = 90° Let DF and DE be perpendicular from D to AC and AB
Hence the other angle ∠ADB = 90° respectively.
D
AB2 = AD2 + BD2 = 162 + 122 = 400
⇒ AB = 20 cm. Ans: (20)

3. ∠ECD = ∠BCD − 30° 10 3 cm 10 3 cm


∠BCD = ∠ABC = 80° (alternative angles)
Hence ∠ECD = 80° − 30° = 50°
As ∠ECD + ∠CEF = 50° + 130° = 180°, EF and CD are
parallel. B
A F
As AB || CD and EF || CD, it follows EF is also parallel to 5 3 cm 5 3 cm
AB.
Hence ∠BHF + ∠EFH = 180°
∠EFH = 180° – 40° = 140°. Height of ∆ABD = DF = AD2 − AF2
Note: ∠BCI = 100° is redundant. Choice (A)
= (10 3 ) − (5 3 )
2 2
= 300 − 75 = 15cm
4. A DF = EA.
(DC2 = DE2 + EC2) and DE = AF
20° ⇒ DC = (5 3 ) + (15 + 10)
2 2

= 75 + 625 = 10 7 cm

30° Note: The vertex D of the equilateral triangle can be shown


B D E C on the side of C. But, then, CD becomes minimum. Hence,
As AB = AD and as ∠BAD = 20°, D is shown as in diagram above.
∠ADB = ∠ABD = 80° Choice (A)

Triumphant Institute of Management Education Pvt. Ltd. (T.I.M.E.) HO: 95B, 2nd Floor, Siddamsetty Complex, Secunderabad – 500 003.
Tel : 040–27898195 Fax : 040–27847334 email : info@time4education.com website : www.time4education.com SM1001962/25
8. A In ∆ABC, BO is the median to AC; because the diagonals
of a parallelogram bisect.
AO = OC = (1/2)AC = (1/2)(24) = 12 --------- (1)
Applying Apollonius theorem to ∆ABC,
q AB² + BC² = 2(AO² + BO²) ⇒ 30² + 18² = 2(12² + BO²)
b ⇒ 6² (5² + 3²) = 2 x 6² x 2² + 2BO²
c
p ⇒ 6² (25 + 9 – 8) = 2BO² ⇒ (18) (26) = BO²;
r ⇒ BO = 6 13 BD = 2BO = 12 13 Choice (D)

B a C 12. As ABCD is a parallelogram, AB ǁ CD. ⇒ ABF ǁ CD


If a, b and c are the sides of the triangle and p, q and r are the Hence ∠EFB = ∠EDC
lengths of the medians as shown, then, by Apollonius theorem, ∠EBF = ∠DCE , ∠BEF = ∠DEC
 b 
2 (vertically opposite angles)
2  p2 +    = a2 + c2 –––––– (1) Hence ∆FEB is similar to ∆DEC
  2  
 EB FE FB
Thus = =
 a 
2 EC ED CD
2  q2 +    = b2 + 2
–––––– (2) As E is the midpoint of BC, EB = EC
  2  
 FB EC
Hence = = 1 ⇒ FB = CD = AB
 c 
2 CD EC
2  r 2 +    = b2 + a2 –––––– (3) AF = AB + BF = 2AB.
  2  

(1) + (2) + (3) = Alternate method:
2 2 2
2b 2a 2c In ∆EFB, DC is parallel to FB.
2 (p2 + q2 + r2) + + + = 2a2 + 2b2 + 2c2
4 4 4 BE FE
3
Hence, = .
2 (p2 + q2 + r2) = (a2 + b2 + c2) EC ED
2 As E is the midpoint of BC, BE = EC, hence,
Hence p2 + q2 + r2 FE = ED; i.e., E is the midpoint of FD. In ∆FDA, E is
3
= (a2 + b2 + c2) =
3
(72) = 54 sq.cm. midpoint of FD and EB ǁ DA..
4 4 Hence, B is midpoint of AF; FA = 2.AB Choice (B)
Note: The relation : 3(a2 + b2 + c2) = 4(p2 + q2 + r2) can be
remembered as a property of triangles. 13.
Ans: (54) D 18 cm C
AD DE AE 1
9. = = =
AB BC AC 3 16 cm
1 20 cm
Area of triangle ADE = (∆ ABC area)
9
(∵ ABC and ADE are similar triangles)
A F B G
1 10 cm
= (54) = 6 sq.cm. Ans: (6)
9 For the given set of measurements of the sides, the
diagram will be as shown above.
10. AB2 = AC2 − BC2 = 102 − 62 = 64
Let DF and CG be perpendiculars to AB, and let AF = x.
⇒ AB = 8 cm
In ∆DAF, DF² = 16² – x² = (256 – x²) ------- (1)
1 DFGC is a rectangle, hence, CG = DF and FG = DC = 18
FB = AB = 4 cm
2 In ∆CBG, CB² = CG² + GB² = DF² + (18 – FB)²
As FB || DE, BFC = CDE (corresponding angles) ⇒ (20)² = (256 – x²) + [18 – (10 – x)]²
⇒ 400 = 256 – x² + (8 + x)² = 256 – x² + 64 + x² + 16x
ABC = DEC = 90° ⇒ 80 = 16x; ⇒ x = 5 -------- (2)
DCE = FCB In ∆ACG, AC² = AG² + (GC)² = (x + 18)² + (CG)²
⇒ AC² = (23)² + (256 – 25) = 529 + 231 = 760
Hence ∆ FBC and ∆ DEC are similar.
AC = 760 and this is the longer diagonal.
BF BC
= Choice (C)
ED EC
BC 6
EC =
BF
× ED = × 20 = 30 units
4 14. EF =
2
(10) + 3 (20) = 16cm . Choice (C)
3+2 3+2
EB = EC − BC = 30 − 6 = 24 units. Choice (A)
11. 15. (AX) (XB) = (CX) (XD)
(8) (14 −8) = (XD+ 8) (XD)
D 30 cm C Hence XD2 + 8XD – 48 = 0
(XD + 12)(XD − 4) = 0
XD = 4 cm
x cm
18 cm O CD = CX + XD = XD + 8 + XD = 16 cm. Ans: (16)
12 cm
16. AB is the common radius of both circles
x cm AC = AD = AB = BC = BD
As AC = BC = AB, ∠CBA = ∠CAB = 60°
A B As AD = BD = AB, ∠DAB = ∠DBA = 60°
12 cm ∴ ∠CAD + ∠CBD = 240°. Choice (C)
nd
Triumphant Institute of Management Education Pvt. Ltd. (T.I.M.E.) HO: 95B, 2 Floor, Siddamsetty Complex, Secunderabad – 500 003.
Tel : 040–27898195 Fax : 040–27847334 email : info@time4education.com website : www.time4education.com SM1001962/26
17. As AB is the diameter of the circle, ∠AQB = 90° 22. Reflex angle ∠TOR = 2 ∠TSR = 240°
As ∠PQB is 40°, ∠AQP = ∠AQB − ∠PQB = 50° ∠TOR = 360° − 240° = 120°
As PT and PR are tangents,
∠BXQ is the external angle of ∆ ∠AQX .
∠PTO = ∠PRO = 90°
Hence, ∠BXQ = ∠XAQ + ∠XQA = 30° + 50° = 80°
∠TPR = 360° − (90° + 90° + 120°) = 60°. Choice (A)
Choice (A)
23. (XP) (XQ) = (XR) (XS)
18. Let ∠XZO be x°
As OX = ZO = radii, XS = ( XP)( XQ) = (6)(6 + 4) = 12cm.
( XR) 5
∠OXZ = ∠OZX = x°
∠XOZ = 180° − (∠OXZ + ∠OZX) = 180° − 2x° RS = XS − XR = 7 cm Ans: (7)
∠XYZ =1/2∠XOZ (Angle subtended at the centre of the
24. When the two tangents drawn
circle is twice the angle subtended at the circumference) Q
from a point outside the circle are P
Hence in quadrilateral OXYZ,
perpendicular, a square is formed
20° + 40° + [360° − (180° − 2x°)] + [1/2 (180° − 2x°)] =360°
by the outside point, the two
60° − 180° + 2x + 90° − x = 0 ⇒ x = 30. points of contact and the center of
the circle. O R
Alternate method: Hence, PQOR is a square.
It is given that PO = 20 cm
Join O to Y. In ∆OXY, ∠OXY = 20° (given data) and
∠OXY = ∠OYX . ⇒ PQ = 20/ 2 cm.
In a square, the diagonals are equal; ⇒ QR = OP = 20 cm
Hence, ∠XZY = 1 ∠XOY = 1 (140°) = 70°---- (1)
2 2 Hence, perimeter of ∆PQR = (20 / 2 ) + (20 / 2 ) + 20
(Angle in the segment is half the angle at the center).
= 20 [(2 / 2 ) + 1] = 20( 2 + 1) = 20 2 + 20 sq. cm
Given that ∠OZY = 40° --------- (2)
Choice (D)
Subtracting (2) from (1),
∠XZY − ∠OZY = 70° – 40°
25.
⇒ ∠XZO = 30° Choice (A)
A 8 cm 8 cm
B
19. ∠TSR = ∠TPQ = 50°
(Angles in the same segment are equal) x cm
5 cm O•
∠PRX = 180° − (30° + 50°) = 100° = ∠STR + ∠TSR
∠STR = 100° − 50° = 50° (5 – x) cm
7 cm 7 cm
Choice (C) D C
20. Given that AD : DC = 3 : 2.
Let E and F be the points of A
contact of the incircle with From the above diagram, OA2 = x2 + 82
sides. AB and BC respectively. D OD2 = (5 – x)2 + 72.
AD = AE, (tangents to the As OA2 = OD2, we have x2 + 64 = x2 − 10x + 25 + 49
circle are equal), E O ⇒ 10x = 10 ⇒ x = 1
EB = BF, FC = CD
As AD : DC : 3 : 2, let AD :
Radius of the circle = OA = 12 + 82 = 65 cm.
DC : BF = 3 : 2 : x B F C Choice (D)
Hence, perimeter of ∆ABC
= (3k + 2k) + (2k + xk) + (xk + 3k) 26. Length of the transverse common tangent is
= 10k + 2kx = 2k (5 + x) and this is given equal to 36. d2 − (r1 + r2 )² = 13 2 − (8 + 4 )2 = 169 − 144 = 5 cm
Hence, 2k (5 + x) = 36, ⇒ k(5 + x) = 18 -------- (1)
By Pythagoras theorem, AB² + BC² = AC² Choice (C)
⇒ (3k + xk)² + (xk + 2k)² = (3k + 2k)²
⇒ 9 + x² + 6x + x² + 4 + 4x = 25 27. ∠ABE < ∠AEB = (As AB > AE)
⇒ 2x² + 10x – 12 = 0; 2(x + 6) (x – 1) = 0 ∠BAE =∠ECD (Angles in the same segment are equal)
⇒ x = 1; while x = –6 is not acceptable ------ (2) ∠ABE =∠EDC (Angles in the same segment are equal)
Substituting in (1), k (5 + 1) = 18,⇒ k = 3 ∠ABE =∠ECD (AB || CD)
Hence BF = BE = kx = 3 x 1 = 3 ------- (3) ∴ ∠BAE = ∠ABE
As can be seen from the diagram, OEBF is a square; and In ∆ABE, ∠BAE + ∠ABE +∠AEB = 180
the inradius = BF = BE 180° > 3 ∠ABE, 60° > ∠ABE
∴ Inradius = 3 cm ∴ Maximum value of ∠ABE = 59°. Ans: (59)
Ans: (3)
28. In order that the circumcentre A12
21. Let its inradius and circumradius be lies on one of the sides, the A11 A1
A
r cm and R cm respectively. triangle must be right angled. A10 A2
CF = CE and AF = AD (Tangents to If A6A12 is one of the sides,
the same circles from the enternal there can be 10 other vertices A9
points A and C.) F •O A3
to form 10 distinct right-angled
AF + CF = AC = 2R (where R is the D r 0 triangles. Similarly, if A1A7 is
circumradius) A8 A4
r r one of the sides, 10 other
AB = r + AD, BC = r + CE vertices can be taken to form A7 A 5
B r E C
Perimeter of ∆ABC = AB + BC + AC 10 distinct right-angled A6
= (r + AD) + (r + CE) + 2R triangles.
= 2r + 2R + (AD + CE) = 2r + 2R + (AF + CF) There are 6 distinct diameters that can be drawn with the
= 2r + 2R + AC = 2r + 2R + 2R given set of twelve points.
∴ Perimeter of the triangle ABC = (2r + 4R)cm Thus the number of right angled triangles formed
2r + 4R = 24 ⇒ r + 2R = 12 cm. Ans: (12) = 6 × 10 = 60. Ans: (60)

Triumphant Institute of Management Education Pvt. Ltd. (T.I.M.E.) HO: 95B, 2nd Floor, Siddamsetty Complex, Secunderabad – 500 003.
Tel : 040–27898195 Fax : 040–27847334 email : info@time4education.com website : www.time4education.com SM1001962/27
29. VZ : ZW = PX : XQ = 3 : 8. Also BE = ED.
3 3 ∴ Difference of the perimeters of the triangles is that of a
∴ VZ = VW and PX = PQ and b. ∴ a − b = 40
11 11
Also VZ and PX are diameters. a + b = 60 and a − b = 40 ∴ a = 50 and b = 10.
AC2 = AB2 + BC2 − 2 (AB) (BC) cos ∠ABC
3
∴ VZ = PX = (4.4) = 1.2 BD2 = BC2 + CD2 − 2 (BC) (CD) cos ∠BCD
11 ∠ABC = 180° − ∠BCD
Radius = 2.2. ∴ OZ = OV – VZ = 1. Similarly OX = 1 ∴ cos ∠ABC = −cos ∠BCD.
∴ XOZY is a square (XOZY is a rectangle)}
{∴ AC2 + BD2 = 2(a2 + b2) − 2ab cos ∠ABC + 2ab cos ∠ABC
XOZY is a rectangle. ∴ OZ ⊥ RS. ∴ Z is the midpoint of = 2(a2 + b2) = 2(502 + 102) = 5200. Choice (B)
RS. ∴ RS = 2RZ = 2 (2.2)2 − 12 = 1.6 6
34.
P
Similarly TU = 1.6 6 . ∴ Sum of the lengths = 3.2 6 .
Ans: (3.2)
30. Let the first two sides be 3x and 4x
Let G be the point of intersection of
the two medians. T
In any triangle all the medians are 2x 2a
concurrent. b G
∴ The third median of the triangle 2b Q S U R
2x a
must also pass through G. 1.5x
∴ G must be the centroid. PQR is equilateral
As G is the centroid, it divides each 1.5x ∠R = 60°, ∠TSR = 30°
median in the ratio 2 : 1. STU and TUR are 30° – 60° – 90° triangles.
b2 + (2a)2 = (2x)2 and a2 + (2b)2 = (1.5x)2 ∴ Ratio of the sides of each is 1 : 3 :2
Adding these 5(a2 + b2) = 6.25x2
a2 + b2 = 1.25x2 TU = x, SU = 3 x and UR = x . SR = 4 x
3 3
Third side = ( 2a)2 + ( 2b )2 = x 5
S is the midpoint of QR ( Q PQR is equilateral)
= 12 5 (given) ; ⇒ x = 12 8x
QR = 2SR =
Smaller of the first two sides = 3x = 36. Choice (A)
3
31. Triangle AΙB is equilateral Perimeter of PQR = 3QR = 24 x Choice (A)
A B
∴ Each of its angles is 60°. 3
∴ ∠ΙAD = ∠ΙBC = 30°
ABCD is a square and triangle 35. A
AΙB is equilateral.
∴ AD = AΙ and BΙ = BC 60°
180 ° − 30° 75° Ι 75°
∴ ∠ADI = ∠AID = 75° 75°
2 D C
= 75° and ∠BΙC = ∠BCΙ = 75°. D B
∠ΙDC = ∠ΙCD = 15°. E
∴ ∠DIC = 150°. Choice (C)

32. ∠PRQ is an angle in a


R
semicircle. Let the rhombus be ABCD. C
∴ It is a right angle. Let ∠A = 120°
PR2 + RQ2 = PQ2 = 412
Let E be the point of intersection of the diagonals
∆DAE and ∆BAE are congruent (sss)
PR and RQ are integers.
∠A
∴ only possible (PR, RQ) ∠DAE = ∠BAE = = 60°
values are (9, 40) and (40, 9). P 41 Q 2
The inradius of any right angled triangle whose sides are a, DAE and BAE are both 30° – 60° – 90° triangles.
a+ b−c DE = 3 EA
b, c and c is the hypotenuse is
2 BD = 2DE, AC = 2EA
9 + 40 − 41 ∴ BD = 3 AC.
∴ The inradius of triangle PQR is =4
2 Required ratio = 3 :1 Choice (D)
Choice (A)
33. 36. Let the sides be 4a, 4b, 4c where a, b, c are positive
A a B integers.
Let 4a ≥ 4b ≥ 4c.
4a + 4b + 4c = 44
b E a + b + c = 11. Also a ≥ b ≥ c
If a = 6, a > b + c. This is not possible.
a ≤ 5.
C If a = 5, b + c = 6. (b, c) = (5, 1), (4, 2), (3, 3)
D a If a = 4, b +c = 7, (b, c) = (4, 3)
Let AB = a and BC = b, let AB > BC a cannot be less than 4 ( Q a + b + c ≤ 3a i.e. 11≤ 3a
Perimeter of ABCD = 2(AB + BC) = 2(a + b)
i.e. a ≥ 3 2 )
2(a + b) = 120 3
a + b = 60. A total of 4 triangles can be formed. Ans: (4)
Triangles BCE and CDE have CE in common.

Triumphant Institute of Management Education Pvt. Ltd. (T.I.M.E.) HO: 95B, 2nd Floor, Siddamsetty Complex, Secunderabad – 500 003.
Tel : 040–27898195 Fax : 040–27847334 email : info@time4education.com website : www.time4education.com SM1001962/28
37. P BD = DF (Q BO = OE)
DF = FA (Q CE = EA)
∴ AD : DB = 2 : 1. Ans: (2)

16 2 Exercise – 4(b)
16
Solutions for questions 1 to 55:
1. ∠5 + ∠7 = 180 ––––– (1)
T 7
S Q As per data ∠5 = ∠7 –––––– (2)
5
Hence from (1) and (2), ∠7 = 75
20 ∠2 = ∠6 (corresponding angles)
14 ∠2 + ∠6 = 2∠6 = 2∠7 (∠6 and ∠7 are vertically opposite
angles) = 150°. Ans: (150)
R
Let us say the diagonals meet at T. 2. ∠TRS + ∠RTU = 180° (TU || RS)
PQ2 = PT2 + TQ2 and QR2 = QT2 + TR2 ∴ ∠TRS = 60° ⇒ ∠QRS = ∠QRT + ∠TRS = 10°+ 360°= 70°
PQ2 – QR2 = PT2 – QT2 As the alternate angles ∠PQR = ∠QRA, PW || RS
SP2 – RS2 = PT2 – QT2 As TU || RS and PW || RS, we have TU || PW
PQ2 – QR2 = SP2 – RS2 ∴ ∠PWU + ∠WUT = 180°
162 – 122 = SP2 – 202 ∴∠WUT = 130°. Choice (C)
SP = 16 2 . Ans: (16)
3. ∠BEF + ∠EFD = 180° (AB || CD)
38. A 2∠GEF + 2∠GFE = 180°
(EG bisects ∠BEF and FG bisects ∠EFD)
∠GEF + ∠GFE = 90° ––––– (1)
In ∆ EFG, ∠EGF + ∠GEF + ∠GFE = 180°
From (1), ∠EGF = 90°
∴ FG2 = EF2 − EG2
18
FG = 25 2 − 15 2 = 20 cm. Choice (D)

4. ∠EAG + ∠ADF = 180° (As BC ǁ DF)


B 36 D 9 C ∠EAG = 180° − 80° = 100°
∠AGB = ∠EAG − ∠ABG = 100° − 30° = 70°
AB2 = AD2 + BD2 = 1620 ⇒ AB = 18 5
Choice (A)
AC2 = AD2 + DC2 = 405 ⇒ AC = 9 5
AB2 + AC2 = BC2 5.
X
∴ ∠A = 90°. P
Note: In ∆PQR, if ∠Q = 90 and QS is the altitude to PR, 138°
then QS2 = (PS)(SR). Conversely, if QS is the altitude to
PR and QS2 = (PS)(SR), ∠Q = 90° Ans: (90) Y R

39. AB : BC : Altitude to AC = p : q : r. 142°


Let AB = pk, BC = qk, Altitude to AC = rk.
1 1 Z Q
Area of ABC = (AB)(BC) = (AC)(Altitude to AC)
2 2 Draw YR parallel to XP and ZQ.
( AB )( BC ) ∠XYR = 180° − ∠YXP = 180° − 138° = 42°
Altitude to AC = ∠ZYR = 180° − 142° = 38°
AC
Hence ∠XYZ = ∠XYR + ∠ZYR = 42° + 38° = 80°
( pk )( qk ) kpq Ans: (80)
rk = =
( pk ) + ( qk )
2 2
p +q
2 2
6. Let the angles of triangle be 2x, 3x and 5x.
2 2 2x + 3x + 5x = 180°
pq p q
r= ⇒ r2 = 10x = 180° ⇒ x = 18°
p +q
2 2 p2 + q2 2x = 36°, 3x = 54° and 5x = 90°.
As we have one of the angles is 90°, the triangle i s a right
1 1 1 angled triangle, and is also scalene as the other two angles
= + Choice (C) are not equal. Choice (C)
2
r 2
p q2
7. Let the perpendicular sides be a and b.
40. a and b are integers.
A
a2 + b2 = 412
F Only possibilities of (a, b) are (40, 9) and (9, 40). In either
case, a + b = 49.
E ∴ Perimeter = 90 Ans: (90)
D
O
8. Let the angles be 3x°, 4x° and 5x°
3x° + 4x° +5x° = 180°
B C x = 15
3x° = 45°, 4x°
BE in the median, ∴ AE = EC = 60° and 5x° = 75°
CD bisects BE. ∴ BO = OE. ∴ The triangle is acute angled whose largest angle
Let F be the point on AB such that EF II CD. exceeds 70°. Choice (B)

Triumphant Institute of Management Education Pvt. Ltd. (T.I.M.E.) HO: 95B, 2nd Floor, Siddamsetty Complex, Secunderabad – 500 003.
Tel : 040–27898195 Fax : 040–27847334 email : info@time4education.com website : www.time4education.com SM1001962/29
9. 15. Let the lengths of the hypotenuse, the shortest side and the
A third side be c cm, b cm and a cm
c + b = 64 ⇒ b = 64 − c
c = 2 (c − b) = 2 (2c − 64)
20° c2 = a2 + b2 = [4 (c − 32)]2 + [64 − c]2
16c2 – 1152c + 0480 = 0
50° c + 2 – 72c + 1280 = 0 ⇒ c = 40 or 32.
C But if c = 32, b = 32 i.e., b = c
B D This is not possible.
∴ c = 40. Ans: (40)
∠BDA = ∠DAC + ∠DCA = 20° + 50° = 70°
As AB = BD, ∠BAD = ∠BDA = 70° 16.
∠ABD = 180° – ( ∠BAD + ∠BDA ) B
= 180° – (70° + 70°) = 40° = ∠CBA Ans: (40)

10. Let ∠ABC be x and ∠CBD be y.


A
Since AC = BC,
∠ABC = ∠CAB = x. A C
Since BC = CD D
∠CBD = ∠CDB = y C BC − BD = AB + (2AD) − (AB + AD2)
2 2 2 2 2

Hence, ∠ABC + ∠CBD = AB2 + 4AD2 − AB2 − AD2 = 3AD2 Choice (C)
= ∠CAB + ∠CBD
D 17. As triangles ABC and PQR are similar,
⇒ ∠ABD = ∠CAB + ∠CDB B
2
As the sum of these angles is 180° Area of ∆ABC =  perimeter of ∆ABC 
⇒ x + x + y + y = 180° Area of ∆PQR  perimeter of ∆PQR 
 
⇒ x + y = 90° = the required sum. 2
Ans: (90) 50 1  40 
⇒ = = 
200 4  x 
11. ∠ABC = ∠AMB + ∠MAB and ∠ACB
1 40 40
= ∠ANC + ∠CAN; as the exterior angle is equal to the sum = = ⇒x= = 80 cm. Choice (D)
of the two interior opposite angles. 4 x 1/ 2
But ∠ABC + ∠ACB = p + q
= ∠AMB + ∠MAB + ∠ANC + ∠CAN = m° + x° + n° + z° 18. Perimeter of ACP = AC + CP + PA = AC + a + a
⇒ x° + m° + z° = p° + q° − n° Choice (D) where a = (1/2) of PQ or QR or RP.
AC = (1/2). QR, because A and C are the mid points of
PQ and PR.
12. BC2 = AC2 − AB2 = 52 − 32 = 16 ⇒ BC = 4 units
Let AD be x units. Hence, the perimeter of ∆ACP = a + a + a = 3a
AB2 − AD2 = BC2 − DC2 Perimeter of ∆PQR = (2a)3 = 6a.
Ratio = 3a : 6a = 1 : 2
32 − x2 = 42 − (5 − x)2
Solving the above equation we obtain x = 1.8 units. Note: Side of ∆PQR is 8 cm. This information is redundant.
Choice (D)
Alternate method:
19. ∠QPR + ∠PQR + ∠PRQ P
= 180° –––––– (1)
As per the diagram, ∠ABC = 90° and BD is perpendicular
∠QOR + ∠OQR + ∠ORQ
to the hypotenuse. Hence, AB² = AD. AC = 180°
⇒ 3² = 5.AD, ⇒ AD = 1.8 Ans: (1.8) O
∠PQR ∠PRQ
∠QOR + +
13. Let AB = a cm, BC = b cm and 2 2
B
AC = c cm = 180° –––––– (2) Q R
By Pythagoras theorem ∠PQR + ∠PRQ
2
= 180° − QOR
a 2
  + c 2 = CY 2 --------- (1) Y = 180° − 140° = 40°
2
c
2
  + a2 = BX 2 --------- (2)
1
2
( 
=  180 0 − ( QPR )  (from (1)

)
2 A X C
⇒ 180° − ∠QPR = 80° ⇒ ∠QPR = 100°
3a2 3c 2 Choice (A)
On subtraction,
4
− = BX 2 − CY 2
4
20. ∠CAB = 80°
= (
3 2
a − c2 =) (
3
AB2 − AC2 ) ∴ ∠ACB + ∠CBA = 100°
4 4 ∠ACB ∠ABC
∴ ∠DCB + ∠OBC = + = 50°.
Hence AB − AC = AB − AC
2 2 2 2
4 2 2
=
BX − CY
2 2 3
4
AB − AC
2 2
(
3
) ∴ ∠BOC = 180° – ( ∠OCB + ∠OBC) = 130°.
Ans: (130)
Choice (A)
21. ∠EAB = ∠EAD +∠DAB = 150°
14. n = 6 In ∆EAB, EA = EB
 2n − 4  180° − ∠EAB
8 ∴ ∠AEB = ∠ABE = = 15°
Interior angle =   × 90 = 6 × 90 = 120° 2
 n 
∠BED = ∠AED − ∠AEB = 60° − 15° = 45°
360 In ∆EFD, ∠FED + ∠EFD + ∠FDE = 180°
Exterior angle = = 60°
6 45° + ∠EFD + 60° = 180
Choice (A) ∠EFD = 75° Choice (D)
nd
Triumphant Institute of Management Education Pvt. Ltd. (T.I.M.E.) HO: 95B, 2 Floor, Siddamsetty Complex, Secunderabad – 500 003.
Tel : 040–27898195 Fax : 040–27847334 email : info@time4education.com website : www.time4education.com SM1001962/30
22. As AD = BC, ABCD is an isosceles trapezium. 31. Given that AP = 8 cm, BP
B
Any isosceles trapezium has the property of being cyclic. = 12 cm and CD = 22 cm
∴ ∠BCD + ∠DAB = 180° Let CP = x cm then DP
12
∠BCD + 100° = 180° = (22 – x) cm D P
∠BCD = 80°. Choice (A) According to the “chords’
8
segments” theorem, AP x C
23. ∠P + ∠Q = 140° + 40° = 180° BP = CP x DP A
Hence PL is parallel to QN. ⇒ (22 – x) x = 8 x 12
∠LNM = 180° − ∠PLN = 180° − 130° = 50° ⇒ x² – 22x + 96 = 0 ⇒ x = 6 or 16
As LM = LN, ∠LMN = ∠LNM = 50° So, the point P divides the chord CD into segments of
∠MLN = 180° − 2 ∠LMN = 180° − (100°) = 80° 6 cm and 16 cm.
Choice (B) ∴ Their difference = 16 – 6 = 10 cm Choice (D)

24. 32. ∠ABE + ∠AEB + ∠EAB = 180°


A 21cm C ∠EAB = ∠EDC (Angles in the same segment are equal)
∠EDC + 20° + 110° + ∠EDC = 180°
105 cm ∠EDC = 25°. Choice (A)
x cm
13 cm D 33. PQ2 = (QB) (QA)
122 = (BQ) (QB + BA) = (BQ) (QB + 10)
x cm QB2 + 10QB − 144 = 0
(QB + 18) (QB − 8) = 0 QB = 8 cm.
B Choice (A)
By Appollonius theorem,
AB2 + AC2 = 2 (AD2 + BD2) 34. The distance between the chords exceeds the radius of the
circle
132 + 212 = 2 (105 + x2) ⇒ 10 2 = x ∴ Both chords cannot be on the same side of the centre.
∴ The length of the other diagonal = 2x = 20 2 cm . Half of the length of the chord whose length is 144 cm.
Choice (D) = 72 cm. Distance of the chord from the centre
= 722 − 722 = 21 cm
25. In ∆UTQ and ∆USP, ∠U is common
Distance of the other chord from the centre = 81 − 21 = 60 cm
∠UTQ = ∠USP (RQ || SP)
x
∴ Triangles UTQ and USP are similar = 752 − 602 = 45, x = 90 Ans: (90)
TQ UQ 2
TQ UQ 1
∴ = ;∴ = = (as PS = PQ)
PS UP RQ UP 3 35. Exterior angle of the regular polygon = 180° − 162° = 18°
1
∴ UQ = UP Ans: (3) n(number of sides) =
360°
3 exterior angle
360°
26. Let the radii of the three circles be = = 20 Ans: (20)
a cm, b cm and c cm. a b 18°
a + b = 21–––––– (1)
b + c = 22 –––––– (2) a b 36. Sum of the interior angles of the polygon = (2n − 4)90°
a + c = 23 –––––– (3) = [2 (20) − 4] 90° = 3240° Ans: (3240)
Hence 2 (a + b + c) = 66 ⇒ a + b c c
+ c = 33 37. Let the angle be x°; then the supplement = (180° – x°)
Hence c = 12, a = 11 and b = 10. 2
Thus the radius of the smallest circle is 10 cm. x° = (180° − x°)
3
Choice (A)
3x° = 360° − 2x°
27. As triangle OAB is an equilateral triangle and AB = 4 cm, 5x° = 360° ⇒ x = 72°
OA = OB = (radii of circle) = 4cm Choice (D)
∴ Circumference of the circle = 2πr = 8π cm Ans: (8)
360°
38. Exterior angle of a polygon =
28. x = 36°; as the angle between the tangent and the chord n
Interior angle = 180(n − 2) – 90°.
through the point of contact is equal to the angle in the
alternate segment. 2
∠BOP = 2 ∠PAB = 2 (36°) = 72° ∴ n = 8.
y + z + (360° − ∠BOP )+ 36° = 360° ∴ Exterior angle = 45° and interior angle = 135°
⇒ y + z = 36° Choice (C)
Hence x + y + z = 36° + 36° = 72° Choice (C)
39. There are two possibilities to be considered
1 (1) h is the greatest. The eight angles, in degrees, are
29. ∠BOC = 90 + ∠BAC d – 3, d – 2, d – 1, d, d + 1, d + 2, d + 3,
2
and d + 8 (i.e. h = d + 4)
(0 is the incentre of triangle ABC)
∴ 8d + 8 = 360 ⇒ d = 44. The angles are
∠BOC = 2∠BAC (0 is the centre of the circle)
41, 42, 43, 44, 45, 46, 47 and 52
1
90° + ∠BAC = 2∠BAC
2 (2) h is the least. The eight angles, in degrees, are
60° = ∠BAC d – 3, d – 2, d – 1, d, d + 1, d + 2, d + 3 and d
∠BOC = 120°. Ans: (120) (i.e., h = d – 4)
∴ 8d = 360 ⇒ d = 45.
30. AD2 = (AB) (AC) = (4) (4 + AD + 4) The angles are 42,43, 44, 45, 46, 47, 48 and 45.
AD2 − 4AD − 32 = 0 In either case, exactly 3 angles are greater than 45°.
As AD > 0, AD = 8 Choice (A) Ans: (3)

Triumphant Institute of Management Education Pvt. Ltd. (T.I.M.E.) HO: 95B, 2nd Floor, Siddamsetty Complex, Secunderabad – 500 003.
Tel : 040–27898195 Fax : 040–27847334 email : info@time4education.com website : www.time4education.com SM1001962/31
40. Let AB and CD be the The longest side of a scalene triangle must be more than
two walls opposite to A one-third of the perimeter and less than half of the
each other which are C perimeter of the triangle.
separated by a distance 8 < c < 12.
of 22 feet. 24 20 ∴ c is 9, 10 or 11.
Let BP = x feet where P If c is 9, a + b = 15. ∴ (a, b) = (7, 8)
is the foot of the ladder. If c is 10, a + b = 14. ∴ (a, b) = (5, 9) or (6, 8)
Length of the ladder = B D If c is 11, a + b = 13.
x P (22 – x)
AP = CP ∴ (a, b) = (3, 10), (4, 9), (5, 8) or (6, 7).
⇒ AP² = CP² ⇒ 24² + (a, b, c) has 7 possible values. Ans: (7)
x² = 20² + (22 – x)²
⇒ 44x = 308 ⇒ x = 7 46. Ι. Let the area of the triangle be A.
∴ AP = 24 ² + 7² = 25 Ans: (25) 2 A 2A
The sides of the triangle are , and 2A .
4 6 9
41. The distance between a pair The sum of any two sides is more than the third side.
of points, one on each circle, ∴ Ι is correct.
is minimum, when the two
ΙΙ. Let the area of the triangle be B.
points are the points of S
intersection of the line of 2B 2B 2B
C •D • •A The sides of the triangle are , , .
centres with the circles. B 6 8 15
Hence when A and B are the 2B 2B 2B
centres of the outer and + > .
8 15 6
inner circles respectively,
∴ ΙΙ is also correct.
distance between centres = AB = AC − BC
Both Ι and ΙΙ are correct. Choice (C)
= r1 − (s + r2) = r1 − r2 − s. Choice (B)
47. Let the length and the
42. As ABCD is a quadrilateral, A B
2(∠a + ∠b + ∠c + ∠d) = 360°.
D
d
C breadth of ABCD be ℓ and
P
a + b + c + d = 180° c b respectively.
Also as ABP is a triangle, a + b + P R S 1
= 180°……….(1) 2(ℓ + b) = 34 and ℓb 5
a Q b 2 E
Also as DQC is a triangle, c + d + Q = 30
A B
= 180°………(2)
(1) + (2) ⇒ a + b + c + d + P + Q = 360° ℓ + b = 17 and ℓb = 60 D C
12 P
∴ P + Q = 180°( ∵ a + b + c + d = 180°) ℓ > b ∴(ℓ, b) = (12, 5). (Let
∵ PQRS is a quadrilateral, ∠PRQ + ∠PSQ AB = 12, BC = 5)
CP : PD = 1 : 3
= 360° – ( ∠P + ∠Q) =180°. Choice (C) ∴ CP = 3 and PD = 9
43. Let the triangle be ABC. Let Required distance = EP.
A
D be the foot of the Let EQ be the perpendicular drawn from E to CD.
perpendicular from A to BC P Q BC DC
EQ = = 2.5 and QP = – CP = 3.
(D is not shown in the figure). 2 2
Let s be the side of the
61
square. EP = EQ 2 + QP 2 = Choice (D)
PQ  BC 2
B S 25 R C
∴ Triangles APQ and ABC
are similar 48. A 1
B
PQ AD − s s AD − s
∴ = ⇒ =
BC AD 25 AD 13
E
The triangle is right-angled at A ( Q 152 + 202 = 252).
1 1
∴Its area = (15) (20). This is also equal to (25) (AD).
2 2
D 3 F 1 C
∴ AD = 12 ⇒ s = 12 − s ⇒ s =
300
25 12 37 AB ||CD
1200 ∴ ∠BAE = ∠ECD and ∠EBA = ∠EDC
∴ Perimeter of PQRS = 4s = Choice (B)
37 Triangles AEB and CED are similar. Their corresponding
EB AB 1
44. Let O be the centre of the circle. B Q 6 sides are proportional, i.e., AE = = =
The sides of the quadrilateral form C CE ED CD 4
tangents to the circle. ∴∠OPA = Let AE = x and BE = y ∴ EC = 4x and ED = 4y.
6 Let F be the foot of the perpendicular from A to CD.
∠OSA = 90°. Also ∠A = 90° and
OP = OS = radius P • R ∴ CF = 1, FD = 3
O
∴ APOS is a square. . . . . . . (1) 8 8 and AF = AD 2 − DF 2 = 13 − 9 = 2.
RC = QC = 6 (given)
∴ RD = CD – RC = 14 – 6 = 8 A 8 S 8 D ∴ AC = 5x = AF 2 + FC2 = 4 + 1 = 5
∴DS = 8 and
SA = DA – SD = 16 – 8 = 8. ∴ radius is 8 and BD = 5y = BC 2 + CD 2 = 4 + 16 = 20
( Q From (1)) Ans: (8)
AE = x = 5 and ED = (4/5) BD = 4y = 8 5
45. Let a, b, c be the sides of each triangle satisfying the given 5 5
conditions. 8
DE 4 y 5
The triangles are scalene. ∴ = = =8 Ans: (8)
∴ Let a < b < c. AE x 1
a + b + c = 24 where a, b, c are integers. 5

Triumphant Institute of Management Education Pvt. Ltd. (T.I.M.E.) HO: 95B, 2nd Floor, Siddamsetty Complex, Secunderabad – 500 003.
Tel : 040–27898195 Fax : 040–27847334 email : info@time4education.com website : www.time4education.com SM1001962/32
49. 2
D C ∴ Area of the circle = π AC  = π  5625 
 2   4 
6x + 1
x
Area of the region inside the circle which is outside the
E quadrilateral = 5625 π − 2106 Choice (A)
4
6x – 3
30x + 5
53. Let height of the pole be ‘h’ and
radius of the circle be ‘a’
A B i.e. PL = h and OQ = OP = a PQ is
also equal to a
AB || CD and the diagonals meet at E let us draw a line ∴ ∆QPL is a 30°, 90°, 60° triangle O
through E which is parallel to AB. This line will also be L
parallel to CD. and PL = QP . 60°
Ratio of the intercepts AE and EC will be the same as that 3
30°
of BE and ED. P
∴ PL = PL = 1 Q
= BE . ⇒ Let ED = x ⇒ 30 x + 5 = 6x − 3
AE OP QP 3
EC ED 6x + 1 x Choice (B)
⇒ 6x2 − 17x − 3 = 0
54. In ∆ABC, AB = 16, BC = 20
⇒ (6x + 1) (x − 3) = 0 ⇒ x = 3.(∵ x > 0) A F
and CA = 18
i.e., ED = 3 D and E are points on AB 18
Choice (A) and AC respectively much 16 E
that
50. Let ABCD be the rhombus AD + AE = 25 and area of
Let E and F be the midpoints of A
∆ ADE = (1/2) Area of D,B
AB and AD respectively. 20 C
∆ABC
The line joining the midpoints of We notice that it we take D = B and E as the midpoint of
two of the sides of a triangle is E F AC, both the conditions are satisfied. (They would also be
parallel to the third side of the 6
satisfied if we take AD = 9 and AE = 16. But even in that
triangle. ∴ EF ||BD case, we would get the same value of DE as the first case.)
∴ Triangles AEF and ABD are
B D
similar ∴ DE = BE = m (say)
∴ BD = 2EF = 12. Let F be a point in BE extended such that BF = 2 m.
Let P be the point of intersection
(∆AEB ≅ ∆CEF. ∴ CF = AB = C (say). Let BC = a)
12
of the diagonals BP = PD = = ∴ In ∆BCF, (2m)2 = a2 + c2 – 2ac cos ∠BCF
2
= a2 + c2 + 2ac cos ∠ABC
6 and BP2 + PA2 = BA2
Each side of the rhombus is 10. C (∵ AB ∥ FC and ∠ABC, ∠FCB are supplementary)
In ∆ABC, b2 = a2 + c2 – 2ac cos∠ABC
∴ PA = 10 − 6 = 8.
2 2
∴ 4m2 = a2 + c2 + (a2 + c2– b2) = 2a2 + 2c2 – b2
∴ The diagonals of the rhombus are 16 and 12. = 2(202 + 162) – 182 = 988 and DE2 = BE2 = M2 = 247
1 Choice (B)
∴ Its area = (product of the diagonals) = 96
2
Choice(C) 55. A

51. The smallest triangle that can circumscribe a given circle is


equilateral triangle whose side is 2r
C
2 3 times the radius of the circle.

Here, a side (AB = 6) is given


along with the radius D B πr X
•O Point A reaches point X, when the wheel makers half a
(OD = 3 ) of the circle. rotation; i.e. BX is equal to half the perimeter of the wheel;
A B i.e. BX = 1/2(2πr) = πr
Here the ratio of the side and the radius AX = 4r 2 + π2r 2 = 4(100) + π2 (100) = 10 4 + π2
= AB = 6 = 2 3 Also AB = 2r
OD 3
AX 10 4 + π2 π2 + 4
∴The ∆ABC can be equilateral ∴ Required ratio = =
Hence the perimeter of ∆ABC is the least when it is AB 20 2
equilateral.
Alternate method:
∴The least perimeter of ∆ABC = 3 × 6 = 18. Ans: (18)
The problem can be solved, even if the value of the radius
52. AB2 + BC2 = AC2. ∴ ∠B = 90o (∴
∴∠D = 90°, as ∠B + ∠D = 180°) is not given. From the figure, it is clear that,
∴ Area of the quadrilateral is the total area of two right AX² = AB² + BX² = (2r)² + (πr) ² = r² (π² + 4)
angled triangles. ∴ Area of the quadrilateral AX² r ² ( π² + 4 ) ( π² + 4 )
Hence, = =
= ( AB )(B) + ( AD(DC) AB² 4r ² 4
2 2 AX 1
 
⇒ = ( π² + 4 )
(72) 75 2 − 722  AB 2
= ( 45)(60) +   = 2106 Note: Radius = 10 cm is redundant information.
2 2 Choice (B)

Triumphant Institute of Management Education Pvt. Ltd. (T.I.M.E.) HO: 95B, 2nd Floor, Siddamsetty Complex, Secunderabad – 500 003.
Tel : 040–27898195 Fax : 040–27847334 email : info@time4education.com website : www.time4education.com SM1001962/33
Solutions for questions 56 to 65: statement ΙΙ, the longest side is 5
Using both the statements, in the right angled triangle
56. Statement Ι: The diagonals can be equal even in a non- the hypotenuses is known and the two angles are also
rectangle. For example, diagonals are equal in an isosceles known. So we can find the remaining two sides and thereby
trapezium. the area. Choice (C)
(If the figure is a rectangle, it must be a square).
Ι is not sufficient. 63. From statement Ι,
Statement ΙΙ alone is not sufficient as it is not given whether ∠BAC + ∠BCA + ∠ABC = 180°
the diagonals are equal and bisect each other. 50° + ∠BCA + 30° + ∠BCA = 180°
Using both the statements also we can’t answer the ∠BCA = 50°
question as the four sided figure may be as given below. So the given triangle is an isosceles triangle.
Statement Ι alone is sufficient.
A B A
From statement ΙΙ, the altitude bisects the base. Hence it is
isosceles. Choice (D)

64. From statement Ι, AB + AC = 2BC and AC = BC


D B So AB = AC = BC, so triangle ABC is an equilateral
triangle.
C In an equilateral triangle the perpendicular line drawn from
D
A bisects BC. So statement Ι alone is sufficient.
Figure (1)
From statement ΙΙ, AB = AC. So triangle ABC is an
C isosceles triangle. ∴ The perpendicular line drawn from
Figure (2) A bisects BC. So statement ΙΙ alone is also sufficient.
Figure (1) is a rectangle but figure (2) is not a rectangle. Choice (B)
Choice (D)
65. From statement Ι,
57. If circumcircle passes through A then we can’t say whether We don’t know whether the diameter of circle B is more or
∠A is 90° or not. So statement Ι alone is not sufficient. less than the diameter of circle A. So we can’t answer the
From statement ΙΙ, ∠B = 90° ⇒ ∠A is not equal to 90°. question.
∴ Statement ΙΙ alone is sufficient. Choice (B) Using both the statements,
If the circle with centre A is bigger than the other circle the
58. From statement Ι, two circles will not touch each other. If the circle with centre
B is bigger than the other circle they will intersect each
∠A = 60° Q
other, but the two circles never touch each other externally.
As OP = OQ = radii X
Choice (C)
∠P = ∠Q = ∠X and ∠POQ P A
= 180° – 60° = 120° O •
B Chapter – 5
∴ ∠X + ∠X + 120° = 180° C
⇒ ∠X can be determined. (Mensuration)
∴ Statement Ι alone is
Concept Review Questions
sufficient.
From statement ΙΙ, ∠B = 30° = ∠C
Solutions for questions 1 to 50:
∴ ∠A = 60°; ∴ ∠X = 30°
∴ Statement ΙΙ alone is sufficient. Choice (B)
1
1. Area of triangle = (4)(6) sin30º = 6 sq.cm. Choice (B)
59. From statement Ι, AD = 2. 2
⇒ BD = 2, because ∠DAB = ∠ ABD = 45°.
2. Area = rs. Choice (A)
∴ AB = 2 2
 BD 2 1  abc
BD = 2 ⇒ DC = 2 Q tan 30° = = =  3. Area = . Choice (D)
4R
 DC DC 3 
= BC = 4 + 12 = 4
3 2
4. Area of the equilateral triangle of side a cm = a .
∴Perimeter = 2 2 + 4 + 2 + 2 3 . 4
∴ Statement Ι alone is sufficient.
But statement ΙΙ alone is not sufficient as nothing is known ∴ Area of the given equilateral triangle =
4
( )
3 2
6
about the point E. Choice (A)
= 9 3 sq.cm. Choice (D)
60. Statement Ι is always true for any regular polygon.
From statement ΙΙ, one of its exterior angles = 60° 5.
360°
∴ The number of sides = =6
60
∴ Statement ΙΙ alone is sufficient. Choice (B) 13 13

61. From statement Ι, AB2 > BC2 + AC2 so ∠C is the largest, 5


but we can’t say which is the smallest angle. So statement Ι h= 132 − 52 = 12
alone is not sufficient. 10
From statement ΙΙ, BC2 < AB2 + AC2 so we can say that ∠A 1
Area = × 12 × 10 = 60 cm². Ans: (60)
is not obtuse, but we can’t say which is the smallest angle. 2
Using both statements also we can’t say which is the
smallest angle in triangle ABC. Choice (D) 6. a = 14cm, b = 48cm, c = 50 cm
This is a right angled triangle
62. From statement Ι, the angles are 30°, 60° & 90° and hence As a2 + b2 = c2
the ratio of the sides can found but not the area. From ∴ Area = ½ (a) (b) = 336 cm 2. Choice (B)

nd
Triumphant Institute of Management Education Pvt. Ltd. (T.I.M.E.) HO: 95B, 2 Floor, Siddamsetty Complex, Secunderabad – 500 003.
Tel : 040–27898195 Fax : 040–27847334 email : info@time4education.com website : www.time4education.com SM1001962/34
7. The triangle formed by joining the midpoints of the sides of
another triangle must have an area which is a quarter of the ∴ Area = 6 3 ( ) = 108cm²
2
or
area of the outer triangle.
d2 (6 3 )2
T2 is formed by joining the midpoints of the sides of T1 Area = = = 108 cm². Ans: (108)
1 2 2
∴ T2’s area = (T1’s area)
4 18. Area = ½ (14 + 8) × h ⇒ 154 = 11 h
∴ Required ratio = 1 : 4 Choice (C) h = 14 cm Choice (B)

8. Let the length and the breadth of the rectangular sheet be 19. l = 12cm, b = 5 cm
l m and b m respectively
lb = 1680 . . . (1) D= l2 + b 2
2(l + b) = 164 ⇒ l + b = 82 . . . . (2)
length of diagonal = 12 2 + 5 2 = 13 cm
From (1) and (2), ℓ = 42 and b = 40. Ans: (42)
Area = l × b = 12 × 5 = 60 sq.cm. Choice (A)
9. l = 81m, b = 25m 20. Field = 25 m × 15 m
Area of rect. = 81 × 25m2 Path width = 3 m
Area of square = a2 = 81 × 25 × 2 ⇒ 81 × 50 Total Area = 2 × 25 × 3
⇒ 2a2 = 81 × 100 = 8100 + 2 × 15 × 3 or 31 × 21 – 25 x 15 = 276 sq.m
+4×3×3
⇒ 2a 2 = 8100 ⇒ a 2 = 90 m Choice (A)
= 240 + 36 = 276 sq.m. Choice (A)

3 1
10. Area of triangle = (2a)2 = a2 3 21. Area of a trapezium = (Height) (sum of the lengths of the
4 2
Area of square = (2a)2/2 = 2a2 parallel sides)
∴ ratio = a2 3 : 2a2 = 3 :2 Choice (C) ∴ Required area (in sq.cm.) =
1
(12) (21 + 3) = 144 cm2.
2
1 Ans: (144)
11. Required area = (80) (18) = 720 sq.cm. Ans: (720)
2
21
22. 2πr = 21π ⇒ r =
2
1
12. Required Area = (4 + 20) (5) = 60 sq.cm. Choice (C)
2 Area = πr 2 = 27 × 27 × 21 = 346.5 cm2. Choice (D)
7 2 2
13. Area of quadrilateral PQRS = Area of PQR + Area of PSR
23. r = 7cm ⇒ 2r = 14 cm = a 2
= (6)(12) + (4)(12) = 60 sq cm
1 1
Choice (A)
2 2 (a is side of square)
⇒ a = 14/ 2 = 7 2
14. A
Perimeter of the square = 4a = 4 × 7 2

13 cm = 28 2 cm. Choice (C)


Perimeter = 52cm = 4a
∴ a = 13cm E
d = 10 cm = AC B C 24. Distance covered by the wheel is one revolution = 2π r
∴ AE = 5 cm 175 × 2π r = 1100
⇒ 350 × 22/7 × r = 1100 ⇒ r = 1m
∴ ED = 13 2 − 5 2 = 12cm ∴ d = 2r = 2m Ans: (2)
∴ Area of ABCD = 4 × ½ (AE) (ED) D 1 3
=4×
1
× 5 × 12 = 120 sq.cm. Choice (C) 25. Radius of incircle = 1/3 × altitude = × a
2 3 2
2 3
15. Let the length and the breadth of the rectangular sheet be Radius of the circumcircle = 2/3 × altitude = × a
3 2
l m and b m respectively
lb = 420 ∴ r1 : r0 = 1 : 2
2(l + b) = 82 ⇒ l + b = 41 ratio of areas = 1 : 4 Choice (B)
26. Area of the path = π (21²) – π (14)²
Diagonal length (in m)= l2 + b 2 = (l + b)2 − 2lb
=
22
(21−14 )(21+ 14 )
= 412 − 2( 420) = 1681 − 840 7
22
= 841 = 29 Ans: (29) = × 7 × 35 = 770 m² Ans: (770)
7
16. Let the length and the breadth of the rectangle be 8x and 27. Perimeter of the semi-circle = π r + 2r
3x cm respectively. 22
Length of the wire = Perimeter of the rectangle. = (14 ) + 28 = 72 cm. Ans: (72)
7
264 = 2(8x + 3x)
264 = 22x
x = 12 28. Area of the sector =
72
360
π (7)2 = 1 22 72
5 7
( )
Required area (in sq.cm.) = (8x) (3x) = 24x2
= 30.8 sq.cm. Choice (C)
= 24(122) = (24) (144) = 3456 Choice (D)
29. Semi Perimeter of the quadrilateral = 15 cm.
17. Diagonal = a 2 = 6 6
Area = (15 − 6)(15 − 7 )(15 − 8 )(15 − 9) = 12 21 sq.cm.
± a = 6 3 cm Choice (D)
nd
Triumphant Institute of Management Education Pvt. Ltd. (T.I.M.E.) HO: 95B, 2 Floor, Siddamsetty Complex, Secunderabad – 500 003.
Tel : 040–27898195 Fax : 040–27847334 email : info@time4education.com website : www.time4education.com SM1001962/35
30. Lateral Surface area = (Perimeter of the base) (Height) = 70π + π(82 + 62) = 170π sq.cm. Choice (D)
= 2(6 + 4) (6) = 120 sq.cm. Ans: (120)
3 3
31. Total surface area = Lateral surface area + 2(Area of the 47. Area of a regular hexagon = (side)2
base) = (4) (6) (10) + 2(62) = 312 sq.cm. Choice (B) 2

32. Volume = (Area of the base) (Height)


∴ Required area =
3 3 2
2
( )
4 = 24 3 sq.cm.

=
4
( )
6 (20 ) = 180 3 sq.cm.
3 2
Choice (A)
Choice (A)

48. Let DE denote the radius of A


33. Volume = (5) (3) (2) = 30 cubic cm. Ans: (30) smaller cone. Let BC denote the
radius of the bigger cone.
Triangles ADE and ABC are
34. Longest diagonal = Body diagonal = l2 + b2 + h2 cm. similar.
D E
Choice (D) ∴
AD DE AE
= =
AB BC AC
35. Face diagonal of a cube = 2 (side) B C
1
∴ DE = BC
∴ Face diagonal = 6 2 cm. Ans: (6) 2
2
1  1
π BC  AD
Volume of the smaller cone
36. Body diagonal of a cube = 3 (side) ∴ = 2  2 =
1

∴ Body diagonal = 8 3 cm. Choice (C)


Volume of the bigger cone π BC AD
2 8
∴ Required ratio = 7 : 8 Choice (B)
37. (i) Lateral surface area = 2(4) (6 + 5) = 88 sq.cm.
Choice (A) 1 
π BC  AE
49. Required ratio = 2 
(ii) Total surface area = Lateral surface area + 2(Area of π (BC) ( AC)
the base) = 88 + 2(6)(5) = 148 sq.cm. Choice (D)
AE 1
As = , required ratio = 1 : 4 Choice (A)
38. (i) Lateral surface area = 4(102) = 400 sq.cm. AC 2
Choice (A)
50. Similar to a method shown in the previous solution it can be
(ii) Total Surface area = 6(102) = 600 sq.cm. shown that the ratio of the sides of the bases of the smaller
Choice (B) and larger pyramids is 1 : 2. Let s be the side of the larger
pyramid and let h be its height.
39. Total Surface area = 3π(62) = 108 π sq.cm. Ans: (108) 1  s  s  h
 2   2  2
Required ratio = 3   
4
40. (i) Volume = π (123) = 2304π cubic cm. Choice (C) 2 2
3 1 hs s 2
  + s2 +   s
3 2  2  2
(ii) Surface Area of the sphere = 4π(122) = 576π sq.cm.
Choice (A) 1  s2h 
3  8  1
41. Curved Surface Area of the hemisphere = 2π(62) = = Choice (B)
17 2 7
= 72π sq.cm. Ans: (72) s h
38

42. Volume of the hemisphere =


2
3
( )
π 63 = 144 π cubic cm. Exercise – 5(a)

Choice (B) Solutions for questions 1 to 35:

43. If a prism and a pyramid have the same base as well as 1. Let each equal side be a cm. Let the base be b cm.
height, the volume of the prism will be thrice that of the 2a + b = 72 ––––– (1)
pyramid. a=b+6 ––––– (2)
Solving (1) and (2), a = 26 and b = 20
∴ Required ratio = 3 : 1 Choice (B)
Area of an isosceles triangle whose each equal side is
a cm and whose base is b cm is given by
44. Area of parallelogram which has adjacent sides of lengths
b
a cm and b cm with the angle between them being 4a2 − b2 sq. cm .
θ = ab sinθ = (8) (10) sin 30° = 40 sq.cm. Ans: (40) 4
a = 26, b = 20
45. (i) Lateral Surface area = 1/2 (16) (8) = 64 sq.cm. ∴ Area = 240 sq. cm. Choice (A)
Choice (D)
2. Let the sides be a and b. Let the angle between them be θ.
a2 + b2 + ≤ 4  1 ab sin θ 
(ii) Total surface area = Lateral Surface area + Area of
the base = 64 + 16 = 80 sq.cm. Choice (B) 2 
46. (i) The Lateral Surface Area of a frustum whose top subtracting 2 ab both sides,
radius, bottom radius and slant height are r cm, R cm 0 ≤ (a − b)2 ≤ 2(ab) (sin θ − 1)
and l cm is given by πl (R + r). ∴ sin θ ≥ 1
Lateral Surface Area = π(5) (6 + 8) = 70π sq.cm. As − 1 ≤ sin θ ≤ 1, sin θ = 1
Choice (D) ∴ θ = 90°
1 1
Area of the triangle = ab sinθ = (12) (sin 90°)
(ii) Total Surface Area = Lateral Surface Area + Base 2 2
Area + Top Area = 70π + π(R2 + r2) = 6 sq.units. Ans: (6)

Triumphant Institute of Management Education Pvt. Ltd. (T.I.M.E.) HO: 95B, 2nd Floor, Siddamsetty Complex, Secunderabad – 500 003.
Tel : 040–27898195 Fax : 040–27847334 email : info@time4education.com website : www.time4education.com SM1001962/36
3. Rhombus ABCD can be divided Radius = 42
B
into two congruent triangles ∆ Hence, l = 108 – 84 = 24
ABC and ∆ ACD of perimeter 36 1
cm each. a a ∴ Area = × 24 × 42 = 504 sq.cm.
y 2
2a + 2x = 36 ⇒ a + x = 18____ (1)
Rhombus ABCD can be divided x x Alternate method:
into four congruent triangles A C
∆ABE, ∆BCE, ∆ADE, and ∆ ECD E Let the central angle of the sector be θ. (inradians)
of perimeter 24 cm each. y (42) θ + 42 + 42 = 108
Hence, a + x + y = 24___ (2) a a (42) θ = 108 – 84 = 24 ; θ = 24
From equation (1), 42
a + x = 18. From this information  θ 2 (24)
and equation (2), D Area of the sector =  r  = (42)(42)
 2  (42)(2)
y = 24 − (a + x) = 24 − 18 = 6 cm.
Triangle ABE is right-angled, a2 = x2 + 62 = (12) (42) = 504 cm2. Ans: (504)
a2 = (18 − a)2 + 62
9. The sum of the diameters of the two circles is equal to the
a2 = 324 − 36a + a2 + 36
side of the square.
36a = 360 ⇒ a = 10 cm. Choice (A)
If radius of each circle is r, then 2r + 2r = 4 ⇒ r = 1
4 x π(1)² 4π π
4. Let the shorter parallel side of the trapezium be a cm. ∴ required ratio is = =
1 4² − 4 π(1)² 4[ 4 − π] 4−π
Area of the trapezium = h (b1 + b2), where b1 = a and Choice (D)
2
1
h = a; b2 = 21 ⇒ a (a + 21) = 98 10.
2
S R
a + 21a − 196 = 0
2

(a + 28) (a − 7) = 0
a = 7 cm. A B C
O
Hence the height of the trapezium = a = 7 cm. Ans: (7)

5.
120° P Q
E D
P, Q, R, S are the points of intersection of the circles with
the middle circle.
120° In sector ASBP (with centre A)
F C AS = AB = AP = r
O
∠SAP = 120° ( ∵ ASB is an equilateral triangle)
 120  2 πr 2
A B Area of the sector ASBP =   πr = 3
 360 
The hexagon shown has 6 congruent triangles of which
three of than are inside the triangle BFD. (without overlap) πr 2
Similarly area of the sector BSAP (with centre B) =
3 3
∴ Area of the triangle = × (Area of the hexagon)
6 2πr 2
∴ Area of each shaded region = [ – Area of rhombus
1 3
∴K= Choice (D) ASBP]
2
 2πr 2 3r 2   2π 3 2
6. Area of a parallelogram = (product of two adjacent sides) × =  − ( 2)  =  − r
 3 4 
  3 2 

sin (Angle between them) = 20 × 10 × sin 45°
= 20 × 10 ×
1
= 100 2 sq. cm. Choice (D)  2π 3
∴ Area of the required region = 2r2  − 
2  3 2 

7. Let the radius of the circular wire be r. Let the sides of the Choice (C)
triangular wire and the other wire be a and s respectively.
Given, the resulting figures enclose the same area. 11. A
3 2
∴ πr =
2
a = s2
4
The lengths of the wires are 2πr, 3a, 4s respectively. D• •C

4π 2r 2 π  πr 2  π
2
 2πr 
  = = = (1) < 1 B
 4s  16 s 2 4  s 2  4
(a)
∴ (2πr)2 < 4s2 . ∴ 2πr < 4s
A A
16  s 2  16  3 
2
 4s  4
  = = = <1
 3a  9  a 2  9  4  3 3
∴ 4s < 3a D C D C
2πr < 4s <3a. Both (Ι) and (ΙΙ) follow. Choice (C)

1 B B
8. Area of the sector = lr
2
(b) (c)
Perimeter = l + 2r = 108

Triumphant Institute of Management Education Pvt. Ltd. (T.I.M.E.) HO: 95B, 2nd Floor, Siddamsetty Complex, Secunderabad – 500 003.
Tel : 040–27898195 Fax : 040–27847334 email : info@time4education.com website : www.time4education.com SM1001962/37
∆ADC and ∆ DBC are equilateral triangles. identical equilateral triangles. Area of the inscribed polygon
Required Area = Area of sector DACB (fig (b)) + Area of = (6 3 . r²)/4 ---------- (2)
sector CADB (fig (c)) – Area of the rhombus ACDB (fig (a))
From (1) and (2) required ratio
1 1 3 128 π
= x π x 64 + π x 64 − 2 x x 64 = − 32 3 = [6 3 .r²/4]/[6 3 r²/3] = 3 : 4
3 3 4 3
Choice (A) Alternate method:
12. Let the central angle of the smallest sector be x°.
Ratio of the areas of two regular hexagons is the ratio of
The central angles of the 2nd, 3rd, 4th and 5th sectors are
the squares of their sides. If a regular hexagon is inscribed
2x°, 4x°, 8x° and 16x° respectively.
in a circle and another regular hexagon is circumscribed on
Given that
x° + 2x° + 4x° + 8x° + 16x° = 360° the same circle, then their sides are in the ratio ( 3 : 2).
360° Hence, ratio of areas is 3 : 4. Choice (C)
⇒ 31x° = 360° ⇒ x° =
31
360° 16. Area of the remaining plot over which earth dug out is
uniformly spread = (70 × 40) – (10 × 5) = 2750 sq. ft.
∴ Area of the smallest sector = 31 π 2 2 =
360
( )4π
31
sq.cm Volume of the earth dug out = 10 × 5 × 27.5 = 1375 cu.ft
∴ Rise in the level = Volume of the pit
Choice (D)
Re maining area of the plot
13. The semi-perimeter of the cyclic quadrilateral 1375 1
= = ft. Ans: (0.5)
2+ 4+6+8 2750 2
= = 10 cm.
2
Area of the cyclic quadrilateral 17. Let the number of lead shots be N.
= (10 − 2)(10 − 4)(10 − 6)(10 − 8) (N) (volume of the lead shot) = Increase in the volume of water
4
π (40)2 [(8.5 − 6) 10] = N × π (2)3
= 8 × 6 × 4 × 2 = 8 6 cm2 . Choice (D) 3
(all measurements are converted into mm)
14. Let the side of the equilateral triangle ABC be a cm long 3
and radius of the circle circumscribing it be r cm. Area of N = 1600 × 2.5 × 10 × × 1 = 3750. Choice (B)
4 8
3 2
the triangle ABC can be expressed as, a as well as
4 18. Let the radius of the base of the cone be r and the slant
a3 height of the cone be l.
3 πr (l + r) = 200π and l + r = 25
4r
200 π
a3 r= =8
∴ 3 a2 = ⇒ r = a . . . . (1) π(25)
4 4r 3 Curved surface area of the cone
Let the circumradius of triangle DEF be R cm. = π r l = π (8) (25 − 8) = 136π cm2. Choice (C)
For any equilateral triangle, its circumradius is twice its
2a 19. There are three possible cases in which a cone can be cut
inradius = R = --------- (2)
3 out of a cuboid. Given l = 56, b = 21, h = 14; where l, b, h
Ratio of the areas of the outer circle and the inner are expressed in cms.
2
Case (i): when the base of the cone is in the plane of lb.
  4 2 3 2
equilateral triangle = π  2a  : 3 a2 = πa : a The Radius of the cone =
21
= 10.5 cm.
 3 4 3 4
  2
Height of the cone = 14 cm
16π : 3 3 . Choice (A)
1
V1 = volume of the cone = π (10.5)2 (14)
3
15. When a regular hexagon is
circumscribed around a circle, A G B = π × 7 ² 1. 5 ² × 7 × 2
the hexagon gets divided into 3
six identical equilateral π
triangles, when each of the = (7²) (31.5)
3
vertices is joined to the centre F C
O Case (ii): when base of the cone is in the plane of bh
of the circle.
The radius (r) of the circle is 14
The Radius of the cone = = 7 cm
equal to the altitude of any of 2
the triangles formed. If a is the E D Height of the cone = 56 cm
side of the hexagon, then 1
V2 = volume of the cone = π (7)2 (56)
( 3 ⋅ a) / 2 = r; a = (2r/ 3 ) and the area of polygon 3
= (6. 3 · a²)/4 = (6 3 r²)/3 ---- (1) Case (iii): when the base of the cone is in the plane of h, l
Q 14
P The Radius of the cone = = 7 cm
When a regular hexagon is 2
inscribed in a circle, radius of Height of the cone = 21 cm
the circle is equal to the side 1
of the polygon. Hence r = the U R V3 = volume of the cone = π (7)2 (21)
O 3
side of the polygon; and area
of the polygon Hence, the maximum possible volume occurs in case (ii)
= [6. 3 . (side)²/4], as the S V2 = 1  22 (49 )(56 ) = 2874.67 cm3.
T 3 7 
polygon gets divided into six
Choice (D)

nd
Triumphant Institute of Management Education Pvt. Ltd. (T.I.M.E.) HO: 95B, 2 Floor, Siddamsetty Complex, Secunderabad – 500 003.
Tel : 040–27898195 Fax : 040–27847334 email : info@time4education.com website : www.time4education.com SM1001962/38
20. Height of the cylinder = overall height − height of the The diagram shows the large cone from which a small cone
hemisphere = h. was cut to form the bucket.
14 Ratio of the radii of the cones = AD : BE = 21 : 7 = 3 : 1
h = 21 − = 14 cm. Because of the similar triangles formed, ratio of the heights
2
of the cone = (H : h2) is equal to the ratio of the radii; = 3 : 1
Curved surface area of the cylinder = 2πrh
(here H = h1 + h2)
22
=2× × 7 × 14 = 616 cm² Ratio of the volumes of the two cones is
7 2
1 1 R2 H 3 3
Curved surface area of the hemisphere = 2πr2 πR ²H : π r ²h2 = ⋅ =   ⋅   = 27 : 1 Hence
2 3 3 r² h2  1  1
22  14  22
=2× ×  = 2 × × 72 = 308. the volume of the bucket = (27 – 1) parts of the ratio
7  2  7 26 parts of the ratio = 2548π (given data)
The difference between curved surface areas of the 1
⇒ 26 × × π × 7² × h2 = 2548π
cylinder and the hemisphere = 616 − 308 = 308 cm2. 3
Ans: (308)
⇒ h2 = 6 and h1 = H – h2 = 3 x 6 – 6 = 12 cm.
21. If the height is decreased by x cm, decrease in the volume Choice (B)
= (1/3) [πr2h − πr2 (h − x) = πr2x].
It the radius decreases by x cm, decrease in volume 25. E
= (1/3) [πr2h − π (r − x)2h] = (1/3)π [r2h − (r2 − 2xr + x2)h] ΙΙ 4m
= (1/3)π [2xrh − x2h] C 28 m
D
Combining the two results, πr2x = π [2xrh − x2h]. F 4m
Cancelling π and x both sides, 8m
8m
− r 2 + 2rh
r = 2rh − xh; x =
2
. Choice (A) B G 12 m A Ι
h
0⋅8
22. External radius of the pipe = = 0⋅4 cm = 4 mm.
2
Internal radius of the pipe
= external radius of the pipe − thickness = 4 − 2 = 2 mm.
Volume of the material of pipe = [π (42 − 22) 280] mm3
From the given information, the area of he field that the cow
12 x 280 x π
= cm3 can graze is represented by the sectors Ι and ΙΙ.
1000 (The cow is tied to corner A of the plot)
22 th
15 × × 12 × 280 Area grazed by the cow = 3 of the area of the circle with
Weight of the pipe = 7 = 158⋅4 gm. 4
1000
radius 12 m + Area of the sector with D as center with
Ans: (158.4)
3 90
radius (12 m – 8 m) i.e. 4 m = π(12)2 + x π ( 4)2
23. Capacity of the reservoir = 50 × 30 × 20 = 30,000 m3. 4 360
Time taken to fill the reservoir = 108π + 4π = 112π = 352 sq. m Ans: (352)
capacity of the reservoir
= (cross − sec tional area of the pipe ) 26. Let the side of each cube be s cm.
(speed of flow through the pipe ) s3
Volume of each cube into which C1 is cut = .
(30,000)m3 N
= = 1200 hr. Ans: (1200)
 25  2 10 × 1000 m s3
 m × Side of each of these cubes = 3
 100 × 100  1 hr N

24. Capacity of the bucket = πh [(21)2 + (7)2 + (21) × (7)] Diameter of sphere inscribed in any of these cubes = 3 s3
3 N
= 2548π. 3
 3 
2548 × 3 7644 3 s 
h= = = 12
( 441 + 49 + 147 )   πs3
637 ∴ Volume of that sphere = 4 π  N  =
3  2  6N
Alternate method:
 
 
A D Total volume of the spheres inscribed in
 πs3 
 (N) = πs
3
(h1) C1 = V1 = 
 6N  6
 
Volume of the sphere inscribed in C2
3
4 s πs3
= V2 = π  =
(H) 3  2  6
B E Required ratio = 1 : 1. Choice (A)

27. Let the length and the breadth of R, (as well as R2) be
(h2) l and b respectively.
Let the side of S be a. A cylinder which is formed by folding
along on edge of a rectangle will have that edge as the
circumference of its base. The edge adjacent to the edge
would be its height. C1 will have a height of b and a
C circumference of l.

Triumphant Institute of Management Education Pvt. Ltd. (T.I.M.E.) HO: 95B, 2nd Floor, Siddamsetty Complex, Secunderabad – 500 003.
Tel : 040–27898195 Fax : 040–27847334 email : info@time4education.com website : www.time4education.com SM1001962/39
2 32.
Its volume = π  l  b = lb  l  A 45 B
 2π   2π 
 b 
C2’s volume = lb   15
 2π  15
2
 a   a 
C3’s volume = π   a = a2  
 2π   2π 
a2 = lb. As l > b, b < a < l D C
9 E 63 F
∴ Ca > Cc > Cb. Choice (C)
Let the feet of the perpendiculars drawn from A and B to
28. The dimensions of the room are 4x, 5x and 7x. Volume of CD be E and F respectively.
the room = (4x) (5x) (7x) In ∆DAE, DA = 15, DE = 9
140x3 = 30240, x = 6 cm. ∴ AE = 12
Difference in the costs for covering the walls with papers of 63 − 45
different prices DE = FC = =9
2
= (5.50 − 5) 2 [(7x) (5x) + (5x) (4x)]
= 55x2 = 55(6)2 = 55 (6) (6) = `1980. Ans: (1980) 1
Area of the quadrilateral = (AE)(AB + CD)
2
29. Let the radius of the sphere be r cm. The height and the 1
radius of cylinder would be r cm each. = (12) (45 + 63) = 648. Ans: (648)
2
Ratio of the curved surface area of the cylinder and the
volume of the sphere
33.
= 2π(r )(h) = 1 ; 2πr = 1 ⇒ r =
2 9
4 3 3 4 3 3 2 c
πr πr a
3 3
a
Volume of the sphere
3
= 4 π 9  = 4 π × 729 = 243π cm3. Choice (A)
3 2 3 8 2
b b
30. Let the side of the square base be a.
Let the height of each tank be h. a
Let the radius of the hemispherical tank be r.
The height of the hemispherical tank is also its radius. a
∴ h = r. c
2a
4a = 2πr ⇒ r =
π
2a3 The perimeter of the figure = c + b + a + b + a, where c is
Volume of the cuboidal tank = a2 h = a2r =
π the arc length of the bottom as well as the top quadrant.
2 3 πa
Volume of the hemispherical tank = πr c=
3 2
2 2 
3
16 3 Perimeter of the figure = 2 
π a  + 2b + 2a = πa + 2b + 2a
= π  a = a 
3  π  3 π2  2 
The ratio of the volumes of the cuboidal tank to the πa + 2b + 2a = 100
hemispherical tank is 3π : 8. 100 − π a − 2a πa
b= = 80 – – a.
Difference of the volumes as a fraction of the volume of the 2 2
cuboidal tank = 3π − 8 ≈ 66 − 56 = 10 ≈ 15 %  90 
3π 66 66 Area of the figure (A) = 2  π a 2  + ab
 360 
Choice (B)
πa
= π a + a (80 –
2
– a) = 80a – a2
31. 2 2
P T Q
∴ A + a = 80
2
Ans: (80)
a

34. V

U S R
S R
O
Let TQ = UR = 1.
∴ PQ = QR = RS = ST = 4 and Ar PQRS = 16 P M Q
Ar of ∆PTS = (1/2) (4) (3) = 6
Ar of ∆TQU = (1/2) (1) (3) = 1.5 Let the base of the pyramid be the square PQRS. Let V be
Ar of ∆URS = (1/2) (4) (1) = 2 the Vertex, O the centre of the base and M the mid point of
Ar ∆ TUS 13 PQ. Let OM = 3.
Ar of ∆TUS = 16 – 9.5 = 6.5 and = ∴ MQ = MP =3, i.e, PQ = 6.
Ar PQRS 32 Semiperimeter of PQRS = 12. Height of pyramid OV = 4.
Choice (B) ∴ VM = 5

Triumphant Institute of Management Education Pvt. Ltd. (T.I.M.E.) HO: 95B, 2nd Floor, Siddamsetty Complex, Secunderabad – 500 003.
Tel : 040–27898195 Fax : 040–27847334 email : info@time4education.com website : www.time4education.com SM1001962/40
Area of ∆VPQ = S = 1 (6) (5) = 15. 5.
A
2
If S = 15, A = 36
In general A = 36 S = 2.4S. Ans: (2.4)
15
∠A ∠A
35. Let the length, breadth and the height of the rectangular 2 2
box be ℓ, b and h respectively.
The box is inscribed in a sphere. ∴ The diameter of the B D C
sphere is the longest diagonal of the box.
Diameter of the sphere = l2 + b2 + h2 Area of ∆ABC = 1/2 × AB × AC × sin 60° = Area of ∆ABD +
Area of ∆DAC
l + b + h ⇒ ℓ + b + h = 5000
50 2 = 2 2 2 2 2 2
= 1/2 × AB × AD × sin 30° + 1/2 × AD × AC × sin 30°
Total surface area of the box = 9400. ⇒ 1/2 .(10)(15)(sin60°)
2(ℓb + bh + ℓh) = 94000
Sum of the lengths of all the edges of the box =  1 × 10 × AD × 1 
= 4(ℓ + b + h) 2 2
(ℓ + b + h)2 = ℓ2 + b2 + h2 + 2(ℓb + bh + ℓb)
+  1 × AD × 15 × 1 
= 5000 + 9400 = 14400 2 2
⇒ ℓ + b + h ℓ = 120° ⇒ 4(ℓ + b + h) = 480 Ans: (480)
150 3 = AD (10 + 15) ⇒ AD = 6 3 cm. Choice (D)
Exercise – 5(b)
Solutions for questions 1 to 45: 6.
A E 2 B
1. Let the hypotenuse be c cm and the other two sides be 2 2
a cm and b cm. a + b + c = a + b + a2 + b2 = 90 2

1 ab = 270 ⇒ ab = 540 ⇒ a + b = 90 − c H F
6
2
(Squaring the above equation)
(a + b)2 = (90 − c)2 D G C
a2 + b2 + 2ab = 8100 − 180c + c2
180c = 8100 + c2 − 2 × 540 – c2
Let ABCD be the square and EFGH be the quadrilateral
8100 − 2 × 540 7020
c= = = 39 cm. obtained by joining the midpoints of the sides of ABCD.
180 180 side of ABCD
AE = AH = = 2m.
Alternate method: 2
∴ EH = 2 2 m.
Among the ratios of the sides of a right-angled triangle,
5 : 12 : 13 is one ratio. The sum of the numbers is 5 + 12 + Similarly EF = FG = GH = 2 2 m.
13 = 30; and 30 is a factor of 90 which is perimeter. EFGH is also a square and each side is 2 2 m.
Hence, the sides could be 3 × 5, 3 × 12 and 3 × 13 i.e., 15, A circle is inscribed in EFGH
36 and 39. If these are the sides then, area is ∴ Side of EFGH = Diameter of the circle,
15 × 36 =270, which satisfies the given condition. Hence
∴ Radius of the circle = 2 m.
2 An equilateral triangle was inscribed in the circle.
hypotenuse = 39 cm Choice (A)
∴ circumradius of the triangle = 2 m.
2. Semi-perimeter of the triangle ∴ side of the triangle = 2 . 3 = 6 m
34 + 50 + 52 136
= = = 68. (∵ circumradius of an equilateral triangle of side a =
a
).
2 2
3
Area of the triangle = ( 68 )( 68 − 34 )( 68 − 50 )( 68 − 52 )
3 3
Area of the triangle = sq.m.
= 68 × 34 × 18 × 16 = 816 cm2. Ans: (816) 2

3. Let the side of the equilateral triangle be a cm and the (∵


∵ Area = 3 a 2) Choice (C)
4
height be h cm; then, ⇒ h = 3
a=6 3
2 7. Length of the line joining the midpoints of the non-parallel
Thus, a = 12 cm. sides of an isosceles trapezium be x
Area of the equilateral triangle = ( 3 ⋅ a² )/ 4 a cm
A B
= 3 (12)2 = 36 3 cm2. Choice (A)
4
c cm
4. Let the inradius of the triangle be r cm. 10 cm
7+4+9 E F
Semi-perimeter of the triangle s = = 10 cm. c cm
2
Area of the triangle = 10(10 − 7 )(10 − 4 )(10 − 9 )
C D
= 10 × 3 × 6 × 1 = 180 b cm
Area of the triangle = rs, where r is the inradius. 1/2 × 10 × (a + b) = 150; a + b = 30
180 6 5 3 5 30
r= = = . Choice (C) x = length of DE = = 15 cm. Choice (D)
10 10 5 2

Triumphant Institute of Management Education Pvt. Ltd. (T.I.M.E.) HO: 95B, 2nd Floor, Siddamsetty Complex, Secunderabad – 500 003.
Tel : 040–27898195 Fax : 040–27847334 email : info@time4education.com website : www.time4education.com SM1001962/41
8. D 12. Area of the shaded region = Area of the square ABCD −
90
Total area of 4 sectors = 142 − 4 × × π (7)2
360
A 1 22
= 196 − 4 × × × 49= 196 − 154 = 42 sq. cm.
36 4 7
cm Choice (A)
25
24 3 + 6 + 9 + 12
cm 13. Semi – perimeter = = 15cm
cm 2
Area = (15 − 3)(15 − 6)(15 − 9)(15 − 12) = 18 6 sq. cm
Choice (B)
B 7 cm C
14. Area swept by the hour hand between 11:20 a.m. and
As AB2 + BC2 = 242 + 72 = 252 = AC2 ⇒ ∠ABC = 90° 11:55 a.m. = area of sector through which the hour hand
Area of the quadrilateral ABCD = ∆ABC + ∆ACD swept. Angle of the sector formed for the interval
11:20 a.m. and 11:55 a.m. is the angle of rotation of the
= 1 × 24 × 7 + ∆ACD = 84 + Area of the triangle ACD = 309 hour hand in an interval of 35 minutes = 35 × (3/2) degrees.
2
1 1 22
⇒ ∆ACD = 225 cm². Ans: (225) 35 × × π x 62 35 × × × 36
Hence required area = 2 = 2 7
9. 360 360
A B = (5 × 11 × 36) / 360 = 5.5 cm2 Ans: (5.5)
F 15. Let the side of the equilateral triangle be a and side of
square be s.
4s
E 3a = 4s ⇒ a = .
3
Diagonal of the square = 2 s
D C
4s
Side of the equilateral triangle
Let the length of the perpendiculars AE and CF be = 3 = 4 = 4:3 2 .
x and (5 + x) respectively. Diagonal of the square 2s 3 2
Area of the trapezium = 1/2 [(x + 5) + x] 30 = 210 Choice (D)
Hence the sum of the lengths of the perpendiculars to the
diagonal BD = 14 cm. 16. Let the radius of the circle be r. Let the sides of the square
Note (1): The data that the altitudes differ by 5 cm is and the triangle be s and a respectively.
redundant. 3 2
(2): Even if AD and BC are shown on the parallel sides, πr2 = s2 = a ⇒ C = 2πr, S = 4s, T = 3a
4
the solution does not differ in anyway. Choice (C)
 2
C2 = 4π2 r2 = 4π2  S  = 4πs2
10. Area of the floor  π 
 
= Total cos t of paving the floor with square tiles
Rate of paving the floor with square tiles  
 2 
T = 9a = 9 S 
 = 12 3 s
2240 2 2 2 and S2 = 16s2
= = 320 m² 
7  3 
 
If the breadth of the floor is ‘b’ and the length of the floor is ‘l’  4 
l = 2b ⇒ C < S < T2
2 2
320 = (2b) (b) ∴ C < S < T. Choice (C)
320
b2 = = 160 ⇒ b = 160 = 4 10 m
2 17. Side of the hexagon = 24 = 6cm
4
l = 2b = 8 10 m
perimeter of the floor = 2 (l + b)
2
( )
Area of the hexagon = 3 3 6 2 = 54 3 sq.cm .Choice (A)
= 2 (8 10 + 4 10 ) = 24 10 m . Choice (D)
18. Let the radius of each cylinder be r cm. Let the heights of
11. cylinder A and B be a cm and b cm respectively.
2πrb = 1 +
300 
 (2πra) ⇒ b = 4a
 100 
4a − a
r1 ∴ required percentage = × 100 = 75% Ans: (75)
4a

19. Let the radius of the sphere be r cm. The height and the
radius of cylinder would be r cm each.
r2 Ratio of the curved surface area of the cylinder and the
volume of the sphere

= 2π(r )(h) = 1 ; 2πr = 1 ⇒ r = 9


Let the radius of the inner circle be r1 cm and the radius of 2
outer circle be r2 cm. 4 3 3 4 3 3 2
r2 = r1 + 7 πr πr
3 3
πr22 − πr12 = 1078
Volume of the sphere
= π (r1 + r2) (r1 – r2) = 1078 3
= 4 π 9  =
⇒ π(7) (r1 + r2) = 1078 4 729
π× = 243π cm3 Choice (A)
r1 + r2 = 49 cm Ans: (49) 3  2 3 8 2

Triumphant Institute of Management Education Pvt. Ltd. (T.I.M.E.) HO: 95B, 2nd Floor, Siddamsetty Complex, Secunderabad – 500 003.
Tel : 040–27898195 Fax : 040–27847334 email : info@time4education.com website : www.time4education.com SM1001962/42
20. 4 cm The radii and the central angles of the 3 sectors, A, B and
4 cm
4 C are tabulated below.
4 cm
cm A B C
40 32 Radius 7 14 21
cm cm Central Angle 90° 90° 270°

Total area = Area (A) + Area (B) + Area (C)


π 2 π 3π π 2
4 cm 4 cm = (7 ) + (14)2 + (212) = [7 + 142 + 3(21)2]
4 cm 52 cm 4 cm 4 4 4 4
60 cm 22 1
= × [49 + 196 + 1323] = 1232 Choice (D)
7 4
When squares of side 4 cm are cut from a rectangle of the
length 60 cm and breadth 40 cm, we get a piece which 27. Let the radius and the height of the room be r m and h m
looks like shaded figure above.
respectively. The length of the longest rod that can be
Volume of the cuboid formed when the resulting piece is
placed in the room is 29 m.
made into a cuboid = 52 × 32 × 4 = 6656 cm3
∴ 4r2 + h2 = 841 –––– (1)
Ans: (6656)
The curved surface area is 1320 m2
 22 
∴ (2)
21. Let the length, breadth and height of the cuboid be l cm,
b cm and h cm respectively. Given that r h = 2640 ⇒ hr = 210 –––– (2)
 7 
l2 + b 2 + h 2
(1), (2) ⇒ (2r + h)2 = 841 + 840 = 1681 = 412 and (2r – h)2 = 1
= 1 [2(lb + lh + bh)] ⇒
2 2
1
[ ]
(l − b)2 + (b − h)2 + (l − h)2 = 0 ∴ 2r + h = 41 or 2r + h = –1
or 2r – h = 1 or 2r – h = –1
This is possible only if l = b = h ∴ r = 10.5, h = 20 OR r = 10, h = 21
∴ The cuboid is a cube its volume = l3 = 729 ⇒ l = 9 ∴ The height could be 20 m or 21 m. Choice (D)
∴ Its lateral surface area = 4l2 = 324 sq. cm.
Choice (C) 28. Let the length breadth and height of the cuboid be l cm,
b cm and h cm respectively.
22. Let the original radius of the balloon be r cm. Increase in its Area of the third of the mutually adjacent faces) face
surface area = 4π (3r)2 − 4πr2
− (96 + 48 ) = 72sq. cm
432
= 4π (9r2 − r2) = 8 (4πr2) = (8) (original surface area). =
Ans: (8) 2
lb, lh and bh must be 72, 96 and 48 in any order
23. Quantity of brick work required (lh) (bh) (lb) = (72) (96) (48)
 2 2 ⇒ lbh = (72)(96 )(48 ) = (24 )(3)(24 )(4 )(24 )(2)
= π  10 + 6  −  10   × 30
 2 12   2   = 242 = 576 Choice (B)
 
= π [5⋅52 − 52] × 30 =π [30⋅25 − 25] × 30 29. As the increase in the volume in both cases must be the
= π [5⋅25] × 30 = 157.5π cubic feet. Choice (B) same, final volumes must be the same in both cases.
Final volume = π π(5 + x)2 (5) = π (5 + 3x) (52)
24. Area of the field = (Curved surface area of the roller) × ⇒ 25 + 10x + x2 = 25 + 15x ⇒ x (x − 5) = 0
22 As x ≠ 0, x = 5.
Number of revolutions = (2 × × 49 × 160 × 600) cm2 Ans: (5)
7
30. Given ℓbh = 140 cm3; ℓb = 28 cm2; bh = 20 cm2;
= ( 44 x 7 x 160 x 600 ) m2 = 2956.8 sq.m Choice (A)
= 28 × 20 = 4 cm; b = 5 cm; ℓ = 7cm.
(100 )2 (lb) (bh)
B=
lbh 140
25. Let the radius of the cone be r and height of the cone be h. ∴ Sum of the edges = 4(& + 4 + 5) = 64 cm.
Choice (D)
r = 1 h ⇒ h = 3r, Total surface area = πr(r + l)
3
31. Let the length of the rectangle be l cm. and the breadth of
As h ∝ r and l2 = h2 + r2; r + l ∝ r; ⇒ r(r + l) ∝ r2.
the rectangle be b cm.
∴ Total surface area α r2.
lb = 247 and l − 3 = b + 3 ⇒ l = b + 6.
⇒ Total surface area = kr2 [k is a constant of proportion]
(b + 6) b = 247 b2 + 6b − 247 = 0
Percentage error in Surface area
(b + 19) (b − 13) = 0, b = 13 cm. l = 13 + 6 = 19 cm.
(error in calculatin g total surface area )
= x 100 Perimeter of the original rectangle
Actual total surface area = 2 (l + b) = 2 (19 + 13) = 64 cm. Ans: (64)

= k (1.01r ) − kr × 100 = 0.0201 × 100 = 2.01


2 2

2
32. 5b
kr
Ans: (2.01)

26.

C A1 11a

7 14
9b
A 7
7
14
A2
5a
B 14

Triumphant Institute of Management Education Pvt. Ltd. (T.I.M.E.) HO: 95B, 2nd Floor, Siddamsetty Complex, Secunderabad – 500 003.
Tel : 040–27898195 Fax : 040–27847334 email : info@time4education.com website : www.time4education.com SM1001962/43
Perimeter = 5b + 11a + 9b + 5a + 14b + 16a = 28b + 32a (∵ WUR is also a 45°, 90°, 45° triangle)
28b + 32a = 136 ⇒ 7b + 8a = 34
b
b=
34 − 8a VW = PS – (TV + WU) = a – 2 =a- 2 b
7 2
Area (A) = A1 + A2 = 55ab + 70ab = 125ab Triangles XVW and XQR are similar (∵ VW||QR and ∠X is
 34 − 8 a  common)
= 125a   ∴ 7A = 4250a – 1000a2
 
 7  Ratio of their corresponding sides = VW = a − 2b .
∴ p = 4250, q = 1000 and p + q = 5250. Ans: (5250) QR a
2
 a − 2b 
33. Let the number of times he would go round before ∴ Ratio of the areas of XVW and XQR =  
completely mowing half of the lawn be x.  a 
 
Area of the lawn which would remain after mowing
Choice (A)
= (40)(30) = 600sq. m ∴ (40 − 2x) (30 − 2x) = 600
2 39. Volume of the bottle (V) = Volume of the upper cylinder +
⇒ 4x2 − 140x + 600 = 0 ⇒ x = 30 or 5 Volume of the frustum + Volume of the lower cylinder.
When x = 30, 40 − 2x is negative The height of the frustum is 15 – (8 + 4) or 3 cm.
∴ x = 5. Ans: (5) π
∴ V = π (4)2(8) + 3[62 + 42 + (6) (4)] + π(6)2(4) cm3
3
1 = π (128 + 76 + 144) = 348π cm3. Ans: (348)
34. Area of the triangle EFG = × EG × AG (as height of ∆
2
1 40. Let the side of the square be a.
EFG =DE = AG) = × AD × 1 AB Area of the shaded region = Area of the outer circle – Area
2 3
of the square
1 × AD × 1 AB Area of the dotted region = Area of the square – Area of the
Area of the triangle EFG
=2 3 = 1 : 6. inner circle
Area of the rec tan gle ABCD AD × AB The side of the square is the diameter of the inner circle.
Choice (B) The diagonal of the square is the diameter of the outer circle.
2
  2
∆ECD 3 Required ratio = π 2a  − a 2 : a 2 − π a 
35. = (as heights are equal, areas are in the ratio of  2   
2
∆ECB 2  
π π
the bases). Similarly;
∆ECB
=
4 = − 1 : 1− = 2(π − 2 ) : 4 − π Choice (B)
∆EBA 3 2 4
∆ECD 3 4 2
Multiplying the two results, = x = 41. Let the side of the square be a.
∆EBA 2 3 1
Height of the pyramid = 3( 2 a) = 3 2 a
Choice (A)
Let the length of the perpendicular drawn from the vertex to
36. Let the length of the cuboid be l cm and breadth and height any of the bases of the triangular regions be h
of the cuboid be b cm and h cm. h2 = (3 2a )2 + ( a )2 ⇒ h = 73 a
As l > b 2 2
l h = 60 ____ (1)
Area of each of the triangular regions = 1 ah
bh = 40 ____ (2) 2
l bh = 480 ____ (3)
= 73a 2 = 73 P 2
( ) = 73 P2 Choice (D)
Multiplying (1) and (2), l b h2 = 2400,
4 4 4 64
2400
l b h (h) = 2400; h = 2400 = =5 42. Diagonal of the rectangle = Diameter of the circle
lb h 480
= 12 units.
l = 60 = 12 cm b = 40 = 8 cm For a rectangle whose diagonal is constant, the area as
5 5 well as its perimeter are maximum when it is a square.
Longest diagonal of the cuboid ∴ Ι is true, but ΙΙ is false Choice (A)
= l2 + b2 + h2 = 122 + 82 + 52 = 144 + 64 + 25 43. Let the radius and the height of the cylinder be r and h
= 208 + 25 = 233 cm . Choice (B) respectively.
2πr(r + h) = 440 and r + h = 10
2πr(10) = 440
37. Let the side of the equilateral triangle be a cm. Let the length
and the breadth of the rectangle be l cm and b cm respectively. ∴ r = 22 = 7
2 (l + b) = 3a 22 7
∴h=3
If l = a, b = a .
2 Volume = πr2h = 462 cm3. Ans: (462)
3 2 44. G
Required ratio = a : lb = 3 : 2 Choice (B)
4

38. PQRS is a square. P T Q E F


I
∴∠PQS = 45o
∴ ∆QTV is isosceles. V H
QV b
∴ TV = TQ = = X The points E and F to be centered
2 2
W The circles intersect at G and H.
Also WU = UR = b ∴ GH is a common chord of the two circles.
2 ∴ ∠GEH = 2 ∠GIH
S U R = 60°.----------(1)

Triumphant Institute of Management Education Pvt. Ltd. (T.I.M.E.) HO: 95B, 2nd Floor, Siddamsetty Complex, Secunderabad – 500 003.
Tel : 040–27898195 Fax : 040–27847334 email : info@time4education.com website : www.time4education.com SM1001962/44
As the circles are congruent, ∠GFH = 60° -----(2) hA 1 rA 1
= ; =
EGFH is a rhombus (∵ Each side of EGFH is a radius of hB 2 rB 4
one of the circles). Also (1), (2) imply that triangles EGH v = π r²h so
and FGH are equilateral
v A r 2 A hA 1
Let the radius of each circle be r = 2 =
vB r B hB 32
 3 2
Area of EGFH = 2  r  So, both are required to answer the question.
 4 
  Choice (C)
Area of the region common to the two circles
= 2 (Area of the sector EGH – Area of the triangle EGH) Chapter – 6
  π 2 3 2 (Coordinate Geometry)
= 2  60 .πr 2 − 3 r 2  = r – r
 360 4  3 2
  Concept Review Questions
3 π 3
Ratio = : – : 2π = 3 3 : 2 π – 3 3 . Solutions for questions 1 to 40:
2 3 2
Choice (C) 1. (a) The equation of the x - axis is y = 0 Choice (B)

45. Ratio of the densities of the materials is 1 : 2. Also, the (b) The equation of the y - axis is x = 0 Choice (A)
pipes have the same weight. ∴ The ratio of their volumes is
2 : 1 (Let the lengths be L1 and L2) 2. The slope of the line parallel to the x - axis is 0.
Let the outer diameters be 5x and 4x. Ans: (0)
Let the inner diameters be a and b
Let the thicknesses be 5y and 4y 3. (a) The equation of the line parallel to the x - axis is y = k
a 5 ( x − 2y ) 5 where k is a real number.
a = 5x – 2(5y) and b = 4x – 2(4y) ( ∴ = = ) As this line passes through the point (5, 9), 9 = k
b 4 ( x − 2y ) 4
( )
⇒ y – 9 = 0 is the required line. Choice (B)
π (5 x )2 − a 2
L1 2 (b) The equation of the line parallel to the y-axis is x = h
4 =
( )
Ratio of volumes =
π ( 4 x )2 − b 2 L2 1 where h is a real number.
As this line passes through the point (5, 9), 5 = h
4
⇒ x – 5 = 0 is the required line. Choice (A)
2
L1 2  4  32
∴ =   = Choice (A)
L2 1 5 25 4. We know that the coordinate axes intersect at the origin.
So, (0, 0) is their point of intersection. Choice (D)
Solutions for questions 46 to 50: 5. The lines x = 2 and y = 3 intersect at (2, 3). Choice (A)

46. Neither of the statements alone is sufficient as each has 6. Every point lying on a line satisfies the equation of the line
only partial information. ⇒ 2(3) + 3(2) + k = 0 ⇒ k = –12 Ans: (–12)
Combining statements Ι and ΙΙ,
(1/2)d1d2 = area of rhombus. 1/2 × 6 × d2 = 24 ⇒ d2 = 8 7. In the given point x < 0, y < 0
2
 d1   d2 
2 ⇒ it belongs to Q3. Choice (C)
we have the side of the rhombus =  2  +  2  so
    8. In Q2, x < 0, y > 0 and the distance of a point from Y-axis is
the x-coordinate and the distance from X-axis is the
s= 3 + 4 = 5 ; ∴ Perimeter = 4s = 20.
2 2
y-coordinate.
Choice (C) ∴ the required point is (–2, 3). Choice (B)

47. From statement Ι, 6a² = 96, so a can be found and the 9. The distance of the point from X-axis is its Y-coordinate i.e.
diagonal a 3 can be found. 3 units. Ans: (3)
From statement ΙΙ, a3 = 64 so a can be found and the 10. The distance between two points (x1, y1) and (x2, y2) is
diagonal a 3 can be found. Choice (B)
( x 2 − x1 ) 2 + ( y 2 − y1 ) 2

48. From statement Ι, 4 π r² = 120. Here, it is given that (x1, y1) = (2, −3) and (x2, y2) = (−1, 1)

⇒ r = (120 / 4π) ; v = 4 π r3. ∴ Required distance = (2 + 1) 2 + ( − 3 − 1)2


3 = 5 units Choice (A)
Hence statement Ι alone is sufficient.
Statement ΙΙ alone cannot be sufficient since no
dimensions of the parallelopiped are known. 11. The distance from the origin to (x1, y1) is x12 + y12
Choice (A) Here (x1, y1) = (− 5, − 12)

49. From statement Ι, h = r/2 = ( − 5) 2 + ( − 12) 2 = 13 units Choice (C)


From statement ΙΙ, r = 4 = 2
12. It is given that PQ is the perpendicular bisector of AB and Q
Combining statements Ι and ΙΙ, r = 2 and h = 1.
is a point on AB.
Volume = 1/3π r²h.
⇒ Q is midpoint of AB.
So we can find the volume of the cone.
Choice (C)  x + x 2 y1 + y 2 
The midpoint is given by  1 , 
 2 2 
50. Volume of a cylinder is πr²h. So either of the statements
alone is not sufficient as the relation between radii and i.e.  − 3 − 7 , 4 + 2  = (−5, 3)
heights is given in different statements. Combining  2 2 
statements Ι and ΙΙ Choice (C)
nd
Triumphant Institute of Management Education Pvt. Ltd. (T.I.M.E.) HO: 95B, 2 Floor, Siddamsetty Complex, Secunderabad – 500 003.
Tel : 040–27898195 Fax : 040–27847334 email : info@time4education.com website : www.time4education.com SM1001962/45
13. If θ is the angle made by the line with X-axis in the positive 23. Let the required point be P.
direction, then the slope of the line is tanθ P lies on the x-axis.
= tan 60° = Choice (D) ∴ P is (a, 0)
3
Given XP = YP i.e., XP2 = YP2
(a – 2)2 – (0 – 4)2 = (a – 6)2 + (0 – 10)2
14. (a) Equation of the line with slope m and y-intercept ‘C’ is a2 – 4a + 4 + 16 = a2 – 12a + 36 +100
given by y = mx + c, here m = − 1, c = 3 a = 29/2 Choice (D)
i.e., y = − x + 3 or x + y = 3 Choice (B)
(b) Let the equation be y = mx + c 24. We know that the centroid of a triangle is the same as the
4 centroid of the triangle formed by the mid points of its sides.
m= . When the line cuts the x-axis, y = 0.
3  −4 − 2 + 2 0 + 2 + 4 
∴ centroid of the triangle =  , 
4
∴ 0 = (6) + c  3 3 
3  −4 
i.e., c = –8. =  ,2  Choice (C)
 3 
4
Required equation is y = x−8
3 25. Let P = (–7, 8), Q = (–3, 9), R = (–5, 6) and S = fourth vertex
i.e., 3y = 4x – 24 Choice (A) P, Q, R, S form a parallelogram
∴ its diagonals bisect each other.
15. Intercepts made by the line ax + by + c = 0 on coordinate ∴ midpoint of PR = midpoint of QS.
axes are given by Let S = (a, b)
− c 12  −7 + ( −5) 8 + 6 
x-intercept = = =4 Midpoint of PR =  ,  = (–6, 7).
a 3
 2 2 
−c − ( − 12 )
y-intercept = = = −3 Ans: (4, –3)  −3 + a 9 + b 
b −4 Midpoint of QS =  , 
 2 2 

16. Slope of the line ax + by + c = 0 is given by −a  −3 + a 9 + b 


(–6, 7) =  , 
b
 2 2 
Given line is 3x − 4y + 7 = 0 a = –9 and b = 5
−3 3
Slope = = Choice (A) Alternate method:
−4 4
Fourth vertex of a parallelogram is given by
17. The equation of the line, with x-intercept and y-intercept b, (x1 + x3 – x2, y1 + y3 – y2)
x y = (–7 – 5 – (–3), 8 + 6 – 9) = (–9, 5) Choice (A)
is given by + =1
a b
26. ∴ slope of L1 ≡ 3x – 4y + 7 = 0 is 3/4
x y
i.e. + = 1 ⇒ 2x − 3y = 6 Choice (B) −a
3 −2 and slope of L2 ≡ ax + 8y – 6 = 0 is .
8
Two lines will not intersect only if their slopes are equal.
18. The equation of the line, with slope ‘m’ and passing through
3 −a
the point (x1, y1), is given by y − y1 = m (x − x1) = , i.e, a = –6 Ans: (–6)
4 8
Here m = −2 and (x1, y1) = (−1, 4) 27. Two lines are perpendicular, then product of their slopes is
3
−2 2 −3
i.e. y − 4 = (x + 1) –1. slope of L1 = and slope of L2 = .
3 3 b
3y − 12 = − 2x − 2  2  −3 
   = –1 i.e., b = 2 Ans: (2)
2x + 3y = 10 Choice (C)  3  b 
19. If two lines with slopes m1 and m2 are perpendicular then 28. Given equations of the lines are 8x – 3y = 13 and
m1 m2 = −1 2x + y = 5
1 Solving the two equations & we get x = 2; y = 1
⇒ m1 (−2) = − 1 ⇒ m1 = Ans: (0.5)
2 ∴ The point intersection (2, 1). Choice (B)

20. If two lines are parallel their slopes are equal 29. We know that of a1x + b1y + 4 = 0 and a2x + b2y + c2 = 0
⇒ Required slope = Slope of the given line a1 b1 c1
represent to same line then = =
−3 3 a 2 b2 c 2
= = Choice (B)
−4 4 6x + py + 18 = 0 and x + y + q = 0 represent the same line
6 p 18
⇒ = =
−2 1 1 q
21. Slope of the given line is =1
−2 ∴ ⇒ p = 6 and q = 3, p + q = 9 Ans: (9)
Slope in terms of θ is tanθ = 1
⇒ θ = 45° = π/4 Choice (A) 30. Let A = (4, 3), B = (0, 7) and C = (–4, 3)
7−3 7−3
slope of AB = = −1 , slope of BC = =1
22. Distance from the origin to the line ax + by + c = 0 is given 0−4 0 − ( −4)
c (slope of AB) (slope of BC) = –1
by
a2 + b 2 ∴ AB ⊥ BC
∴ triangle ABC is right angled at B.
∴ The required distance = 10 = 2 units ∴ orthocentre = (0, 7)
32 + 42 (∵ for any right angled triangle, orthocentre is the vertex
Choice (C) containing right angle). Choice (A)

Triumphant Institute of Management Education Pvt. Ltd. (T.I.M.E.) HO: 95B, 2nd Floor, Siddamsetty Complex, Secunderabad – 500 003.
Tel : 040–27898195 Fax : 040–27847334 email : info@time4education.com website : www.time4education.com SM1001962/46
⇒ k = 7/3
31. Diagonal of the square = AC = (7 − 3)2 + (13 − 5)2 ∴ Equation of line is x = 7/3 i.e. 3x – 7 = 0. Choice (D)
= 80 units.
4. 5x − y + 6 = 0 → (1)
1 → (2)
∵ area of a square = (diagonal)2 4x + 3y + 1 = 0
2 Solving (1) and (2), we get x = −1 and y = 1
Its area =
1
2
( 80 ) = 40 sq.units
2
Ans: (40)
∴ (−1, 1) lies in the 2nd quadrant. Choice (B)

 x + x 2 + x3 y1 + y 2 + y3 
5. Centroid =  1 , 
32. Since one of the two lines is parallel to Y-axis and other is  3 3 
parallel to X-axis
2+4+0 8−2+6
∴ The angle between the lines is 90°. Ans: (90) =  ,  = (2, 4). Choice (D)
 3 3 
33. The perpendicular distance from (x1, y1) to the line
ax 1 + by 1 + c 6. Let the x-axis divide the line segments in the ratio m : n, at (x, 0).
ax + by + c = 0 is
 m(1) + n( 4) m(1) + n(7)  7n + m
a2 + b2 ∴(x, 0) =  , ⇒ =0
 m+n m + n  m+n
3( 2) + 4(3 ) + 10 28
∴ required distance = = units m −7
5 ⇒ 7n + m = 0 ⇒ =
32 + 42 n 1
Choice (B) ∴ the required ratio is 7 : 1 externally. Choice (C)

34. Let A be (x, y). Given (X, Y) = (2, 4) and (h, k) = (–2, –3) 7. If a set of three points are collinear, then the area of
x=X+h,y=Y+k triangle formed by joining these points is equal to zero.
⇒ x = 2 –2 =0 and y = 4 – 3 = 1 1 1− 3 1− 7 1 −2 −6
∴ (x, y) = (0, 1) Choice (B) ∆= =
2 7−3 7−k 2 4 7−k
35. The equations relating the coordinates are x = X + h, ⇒ (−2) (7 − k) − 4 (−6) = 0
y=Y+k ⇒ − 14 + 2k = −24 ⇒ 2k = −10 ⇒ k = −5. Ans: (–5)
Here (x, y) = (4, –2) and (h, k), = (–7, 5)
4 = X –7and–2 = Y + 5 8. Any collinear point will lie on the line formed by the given
X = 11, Y = –7 and (X, Y) = (11, –7) Choice (D) points. The equation of the line joining (1, 3) and (3, 7) is
7−3
(y − 3) = (x − 1)
36. radius = g2 + f 2 − c = (− 4)2 + (3)2 + 11 = 6 3 −1
⇒ y − 3 = 2x − 2 ⇒ 2x − y + 1 = 0
circumference = 2  22  (6 ) = 12π The point (2, 5) satisfies the equation. Choice (D)
 7 
Ans: (12) 1 x1 − x 2 x1 − x 3 1 1 − 3 1 − ( −1)
9. Area = =
2 2 2 2 y1 − y 2 y1 − y 3 2 1− 5 1− 2
37. Centre and radius of the circle (x – a) + (y –b) = r is (a, b)
and r. Here the centre is (3, –2) and r = 6. 1 −2 2
Choice (C) =
2 − 4 −1
38. The centre of the circle x2 + y2 + 2gx + 2fy + c = 0 is (–g, –f) 1 1
= [2 − 2( −4)] = × 10 = 5 sq. units. Ans: (5)
which is (–4, 3). Choice (A) 2 2

39. The radius of the circle x2 + y2 + 2gx + 2fy + c = 0 10. Area formed by the line ax + by + c = 0 and co-ordinates
is g 2 + f 2 − c . Here, it is (3)2 + (− 2)2 + 23 = 6 axes is given by
1 c2
Area = π(6)2 = 36π Choice (D) 2 ab
Given line : 4x − 3y = 24, a = 4, b = –3 and c = –24
40. Diameter of the circle is 2 g 2 + f 2 − c 1 ( −24 )2 1 (24 )2
∴ Area = ⋅ = ⋅ = 24 sq. units.
2 ( 4) ( −3 ) 2 12
= 2 (− 3 ) + (4) + 56 = 18
2 2
Ans: (18)
Ans: (24)
Exercise – 6(a) 11. The distance between (–1, –1) and ( ( 3,− 3 )
Solutions for questions 1 to 19:
= (− 1− 3 ) + (− 1+ 3 )
2 2
= 2 (1+ 3) = 2 2
1. Centre of the circle x2 + y2 = a2 is (0, 0). The distance between ( 3 , – 3 ) and (1, 1)
24 2 + 72 = 576 + 49
( 3 − 1) + (− )
Distance = 2 2
= 3 −1 = 2 (3 + 1) = 2 2
= 625 = 25 units. Ans: (25)
The distance between (–1, –1) and (1, 1)

2. Slope =
y 2 − y1
=
a − (a + b ) (− 1 − 1 )2 + (− 1 − 1)2 = 4+4 =2 2
x 2 − x1 − b − (a − b) Since the distances between the points taken in pairs are
equal, they form an equilateral triangle. Choice (D)
a−a−b −b b
= = = Choice (A)
−b−a+b −a a 12. The vertex containing the right angle is the point of
intersection of perpendicular lines
3. Equation of a line parallel to y-axis is of form x = k i.e., 3x + y − 4 = 0 and x − 3y + 2 = 0.
7  Solving the above equations we get x = 1, y = 1.
As, the line passes through the point  , − 2  ;
3  Choice (B)

Triumphant Institute of Management Education Pvt. Ltd. (T.I.M.E.) HO: 95B, 2nd Floor, Siddamsetty Complex, Secunderabad – 500 003.
Tel : 040–27898195 Fax : 040–27847334 email : info@time4education.com website : www.time4education.com SM1001962/47
13. Roots of the quadratic equation x2 − 7x + 6 = 0 are 6, 1. Solutions for questions 21 to 35:
x y
∴If a = 6, b = 1, then the equation is + = 1 a1 a2 + b1b2 + 3+ 3
6 1 21. cosθ = =
x y a12 + b12 ⋅ a22 + b22 1+ 3 1+ 3
⇒ x + 6y = 6 and if a = 1, b = 6, then the equation is + = 1
1 6 +2 3 + 3
⇒ 6x + y = 6. Choice (D) = =
4 2
14. Let the fourth vertex be D(x, y)  3
∴θ = cos− 1   = 30°
Since the diagonals bisect each other in a parallelogram,  2 
the midpoint of AC coincides with the midpoint of BD.  

∴ −1 + x = 2 + 3 Alternative method:
2 2
Slope of x + 3 y + 6 3 = 0,
⇒ x = 6 and 3 + y = 4 − 2 ⇒ y = –1
2 2 m1 = −1 or θ1 = 150° and slope of y + 3 x + 2 = 0,
∴ D(x, y) ≡ (6, –1) Choice (D) 3
m2 = – 3 or θ2 = 120° .
15. The equation of a line making equal intercepts is of the
∴ The angle between the lines = 150 – 120 = 30°
form x + y = 1 (since a = b) Choice (B)
a a
⇒ x + y = a (a ≠ 0), hence x + y – 5 = 0 Choice (A) 22. 2x + y − k = 0 → (1)
4x + y = 13 → (2)
16. x − 2y = 4 ⇒ 3x − 6y = 12 → (1) x − 3y = 13 → (3)
− 3x + 6y = − 2 ⇒ 3x − 6y = 2 → (2) Solving (2) and (3), we get x = 4, y = − 3
Distance between (1) and (2) is Substituting in (1), 8 − 3 − k = 0
c1 − c 2 12 − 2 10 2 5 ⇒k=5 Ans: (5)
d= = = = units.
a 2 + b2 36 + 9 3 5 3
x1 − x 3 y1 − y 3
Choice (C) 23. Area of quadrilateral required =
x2 − x4 y2 − y4
ax1 + by1 + c 2.1 − 1.4 + 7 5 1 −2 − 2 −3 − 3 1 −4 −6 1
17. d = = = = 5 units. = = ((–4)(6) – (–4)(–6)
a 2 + b2 4 +1 5 2 − 2 − 2 3 − ( −3) 2 −4 6 2
Choice (B) = 24 sq.units. Ans: (24)

18. Since the lines are parallel their slopes should be equal. −2
24. ∴ slope of L1 ≡ 2x + 3y – 8 = 0 is and slope of
7 − 3 3
⇒ = ; ⇒ 7 k = −3
k k
3 L2 ≡ kx − 9y + 24 = 0 is
9
−3 7
⇒k = Choice (B) We know that two lines will not intersect only if their slopes
7 are equal.
−2 k
19. The given parallel lines represent the parallel sides of the = ⇒ k = –6. Choice (C)
3 9
squares hence the distance between them gives the length
of the side.
10 − 7
3+3 6 25. slope of first line = =3.
d= = 3−2
122 + 32 153 −1
slope of second line =
4
∴ Area = d2 = 36 = sq. units. Choice (D) 3
153 17 −1 y−6
Required equation is =
3 x − 18
Solutions for question 20:
i.e., 3y + x – 36 = 0. Choice (D)
20. (i) The given line passes through (0, 5) and (3, 0).
−1
∴ y − 5 = 0 − 5 (x − 0 )
26. The slope of 2y + x – 23 = 0 is
2
3−0
⇒ 3y − 15 = − 5x ⇒ 5x + 3y − 15 = 0 Choice (C) −1
∴ slope of XY = =2
−1
(ii) The line is passing through origin and (3, 1).
2
∴Hence the equation is y = 1 x The equation XY is y – 5 = 2(x – 3)
3 y = 2x – 1
⇒ x − 3y = 0 Choice (A) midpoint XY is the point of intersection of y = 2x – 1 and 2y
(iii) m = tanθ = tan 45° = 1 + x – 23 = 0
The line passes through (4, 0) and makes an angle of ∴ midpoint of XY is (5, 9).
45° with x-axis. 3+a 5+b
Let Y be (a, b),  ,  = (5, 9)
∴y = mx + c  2 2 
y = x + c (since m = 1)
(a, b) = (7, 13) Choice (A)
0=4+c⇒c=−4
∴ The equation is x − y = 4 Choice (B) 27. The equations relating the coordinates are x = X +1,
(iv) The line makes an intercept of 4 on either axis y=Y–1
x y Let f(x, y) = 2x - 3y + 7 then the transformed equation is
+ =1 given by f(X, Y)
a b = 2(X + 1) – 3(Y –1) + 7 = 0
⇒ x + y = 4. Choice (D) ⇒ 2X – 3Y + 12 = 0 Choice (A)

Triumphant Institute of Management Education Pvt. Ltd. (T.I.M.E.) HO: 95B, 2nd Floor, Siddamsetty Complex, Secunderabad – 500 003.
Tel : 040–27898195 Fax : 040–27847334 email : info@time4education.com website : www.time4education.com SM1001962/48
28. When the axes are rotated through an angle of 45° i n anti 32.
clockwise direction then the equations relating the R1 R3 R2
coordinates are
x = Xcos45° – Ysin45° and y = Xsin45° + Ycos45°
X−Y X+Y
x= , and y =
2 2 30
∴ The transformed equation of f(x, y) = 0, is
 
fx − y, x + y = 0 15
 2 2 

X−Y X+Y
i.e., − 2 +5 =0
 2 
2   10 20 30
⇒ X – Y – 2 (X +Y) + 5 2 = 0
⇒ –X – 3Y + 5 2 = 0 ⇒ X + 3Y – 5 2 = 0 R2 is parallel to R1. Let its equation 3x + 2y = c
Choice (D) R3 is exactly midway between R1 and R2 ∴ Its equation is
30 + c
3x + 2y =
29. Y 2
Given : equation of R3 is 3x + 2y = 45
Q 30 + c = 45
2
(3, 3) c = 60
(0, 3) • R1 is closer to the origin than R2
C
distance between R1 and the origin = 30 30
P• =
(2,2) 3 2 + 22 13
distance between R2 and the origin
• • X
0 = 60 60 Ans: 0.5
(3, 0) =
3 +22 2 13
The equation of the circle is (x-3)2 + (y-3)2 =9 the center is
(3,3) and radius =3 33. The point of intersection has integral coordinates. Let this
P(2,2) is inside the circle as(2-3)2+(2-3)2<9 point be (x0, y0)
∴The largest possible distance from P to any point on the 4x0 + 5y0 = 26 and y0 = kx0 + 2
circle is PQ, where Q is the end of the diameter passing 16
through P 4x0 + (kx0 + 2) = 26 ⇒ x0 =
4 + 5k
∴PQ = PC + CQ = (3 − 2)2 + (3 − 2)2 +3 (∵CQ = r = 3) x0 is an integer
∴PQ= 3 + 2 Choice (A) ∴ 5 + 5k is a factor (positive or negative) of 16. Also it is odd.
4 + 5k = ±1
30. The maximum distance between any point on one circle −3
k= or –1
C1 and any point on another circle C2 is that between the 5
points which are on the line joining the centers of the circles k has only one integer value. It can be verified that
(between A and B in the figure below). when k = –1, y0 is an integer. Ans: (1)
y 34. y

A 22
C x 20
–8 O 3y + x = 22
18
(x + 8)2 + y2 = 36
16
D x + (y + 15)2 = 64
–15 12
B 2
10
8 3y + x = 22
A
AB = AC + CD + BD i.e, 6 + CD + 8
6 I
CD = CO 2 + OB 2 = 17 C
∴ Maximum distance is 31. Ans: (31) 4
2
31. The points we have to consider must satisfy the condition
x2 + y2 < 9. x
If x2 = 0, x = 0 and y2 < 9. y = 0, ± 1, ± 2 O
2 4
D B
10 12 16 18
6 8 20 22
If x2 = 1, x = ± 1 and y2 < 8. y = 0, ± 1, ± 2 2
If x2 = 4, x = ± 2 and y2 < 5. y = 0, ± 1, ± 2 At I, x + 3y = 22 and 3x + y = 22
A total of 25 points satisfy the given condition. Let Ι be (x0, y0)
Choice (B) x0 + 3 y0 = 22 and 3x0 + y0 = 22

Triumphant Institute of Management Education Pvt. Ltd. (T.I.M.E.) HO: 95B, 2nd Floor, Siddamsetty Complex, Secunderabad – 500 003.
Tel : 040–27898195 Fax : 040–27847334 email : info@time4education.com website : www.time4education.com SM1001962/49
∴ Choice (A) can be the third vertex, and none of the other
Solving these (x0, y0) =  11 , 11 
 2 2 choices satisfy the properties of equilateral triangle.
  Choice (A)
The convex quadrilateral formed by the given lines and
the coordinate axes is OAIB = S 7. We know that centroid divides each median in the ratio
S = Area of AIO + Area of BIO 2 : 1 from vertex.
22  11  121 Given A = (8, 4), D(12, 8)
= 2Area of AIO = (AO) (IC) =   =
3  2  3  2(12) + 8 2(8) + 4   32 20 
∴ Centroid =  ,  =  , 
∴ 3S = 121 Ans: (121)  3 3   3 3 
Choice (B)
35. The distance from the origin to the line 8x – 15y + 140

= 0 is 8 (0) − 15 (0) + 140 i.e.


140 8. Let the third vertex be (x3, y3)
4
i.e. 8  5 + 7 + x3 6 + 9 + y3 
8 + 15
2 2 17 17 Centroid =  , 
 3 3 
The circle x2 + y2 = 64 is centered at the origin and has
a radius of 8  12 + x 3 15 + y 3 
(0, 0) =  , 
As the shortest distance from the origin to the line is  3 3 
more than the radius, the line does not meet the circle at x3 = –12, y3 = –15
even one point. Ans: (0) ∴ Required vertex is (–12, –15). Choice (D)
Exercise – 6(b)
−3
9. The slope of the given line 3x + 4y + 11 = 0 is
Solutions for questions 1 to 45: 4
−3
The slope of the line parallel to it = (∵ parallel lines
1. Centre of the given circle = (5, 4) 4
The distance between (8, 8) and (5, 4) = (8 − 5)2 + (8 − 4)2 have equal slopes)
−3
= 5 units Ans: (5) Required equation is y – 4 = (x – 3)
4
2. 4y – 16 = –3x + 9
Y
i.e., 4y + 3x – 25 = 0 Choice (C)
θ
x y
10. Equation of the line with equal intercepts is + =1
a a
θ 18 If it is passes through the point (5, 12) then 5 + 12 = a ⇒ a = 17
∴ Equation of the required line is x + y = 17 and intercepts
the line are 17, 17 Ans: (17, 17)
X
−9 2 O
11. When x + 4y – 16 = 0 intersects the X-axis, y = 0.
∴ x + 4(0) – 16 = 0 i.e., x = 16.
9 2   1  When it intersects the Y-axis, x = 0
  –1  
Required angle = θ = Tan–1   = Tan  2  0 + 4y – 16 = 0 i.e., y = 4
 18   
Y
Choice (C)
(0, 4)
3. It is given that the parallel lines x + 3y + 7 = 0 A
⇒ 4x + 6y + 14 = 0 and 4x + 6y + 15 = 0.
4
14 − 15 1 (16, 0)
Distance between the lines = = X
2 13 O 16 B
4 +6
2 2

Choice (B)
1 1
Area of triangle AOB = (OA)(OB) = ( 4)(16)
4. Given, the lines 3x + 4y + 8 = 0 and 12x – 5y + 9 = 0 2 2
Angle between the lines = 32 sq.units Choice (C)

 (3)(12) + (4)(− 5)   16 
 = Cos–1   12. y2 – 9y + 18 = 0
= Cos–1 
 ( )(
 32 + 42 122 + (− 5)2 
 )  65  ⇒ y2 – 6y – 3y +18 = 0
i.e., (y – 6) (y – 3) = 0
Choice (D) i.e., y = 6 or 3 by taking one root as slope and the other as
Y – intercept
5. AB = 3CD (∵ C and D trisect AB) ∴ the required equation of the lines are y = 6x + 3 or
y = 3x + 6 Choice (D)
AB = (3 − 6) 2 + (7 − 13) 2 = 3 5
13. The point of intersection of the lines 3x + 4y = 14 and
CD = 5 Choice (A) 2x + 3y = 10 is (2, 2)
Given the three lines are concurrent
6. Let P = (3, –4), Q = (–3, 4) and the third vertex be R. ⇒ point (2, 2) lies on the line 5x + ky = 6.
⇒ 5(2) + k(2) = 16 ⇒ k = 3
PQ = ( −3 − 3) 2 + ( 4 − ( −4)) 2 = 10 Ans: (3)
(
from Choice (A) if R 4 3 .3 3 then ) 14. The point of intersection of 2x + 3y + 4 = 0, 5x – 7y –19 = 0 is
(1, –2)
RP = (3 − 4 3 ) + ( −4 − 3 3 )
2 2
(1, –2) lies on the line 4x + ky + 6 = 0
4(1) – 2k + 6 = 0
= 9 − 24 3 + 48 + 16 + 24 3 + 27 = 10 k = 5. Choice (A)

Triumphant Institute of Management Education Pvt. Ltd. (T.I.M.E.) HO: 95B, 2nd Floor, Siddamsetty Complex, Secunderabad – 500 003.
Tel : 040–27898195 Fax : 040–27847334 email : info@time4education.com website : www.time4education.com SM1001962/50
1 x1 − x 2 x2 − x3 Given that (3, 2) lies on 4x + ay − 22 = 0.
15. Area of triangle = ⇒ 4(3) + a(2) – 22 = 0
2 y1 − y 2 y2 − y3
∴a=5 Ans: (5)
Let D, E, F be the midpoints = D(1, 5) E(3, 6), F(4, 8)
1
1 1− 3 3 − 4 1 −2 −1 1 24. Gradient of AB = tan30° =
Area = = = 4 −1 3
2 5−6 6−8 2 −1 − 2 2
= 3/2 sq.units. 1
equation of AB is y = x + c where c is the y−Intercept.
3 3
∴ Area of ∆le ABC = 4 × = 6 sq.units. Ans: (6) It is passing through (3, 4).
2
1
4= (3) + c ⇒ 4 – 3 = c
1 x1 − x 3 x2 − x4 3
16. The area of quadrilateral =
2 y1 − y 3 y2 − y4 1
∴y= x+4– 3
1 1− 5 3 − 3 1 3
= = 16 − 0 = 8 sq.units. Choice (C)
2 3 − 3 1− 5 2 At B, x = 0 ∴ y = 4 – 3

17. Given 2x + 3y + 6 + k (x – 4y + 8) = 0 ∴ AB = (3 − 0)2 +  4 − (4 −



)
2
3  =2 3

Choice (B)
⇒ (2 + k) x + (3 – 4k)y + 6 + 8k = 0 This line is parallel to
the Y-axis. 25. The ratio in which the X – axis divides the line joining the
The coefficient of y should be zero. points (x1, y1) and (x2, y2) is –y1 : y2.
3 – 4k = 0 ⇒ k =
3
Ans: (0.75) Here y1 = −1, y2 = 3
4 ∴ Required ratio is 1 : 3
∴ the X – axis divides in a 1 : 3 ratio internally.
18. Gradient of the line joining the points A (3, 6) and B (4, 9) is Choice (C)
9−6
=3 26. Let its x−intercept be a. Its y–intercept =14–a
4−3 then the equation of the line is
And slope of the line joining the points A (3, 6) and C (5, k) x y
k−6 + =1
= a 14 − a
5−3
Given A, B, C are collinear slope of AB = slope of AC (3, 4) lies on it. ⇒ 3 + 4 =1
k −6 a 14 − a
⇒ =3 42 – 3a + 4a = 14a – a2
2
∴ k = 12 Choice (A) a2 – 13a + 42 = 0
(a – 6) (a – 7) = 0
19. p2 – p – 12 = 0 If x – intercept is 6, then y – intercept is 8. So, the equation
(p – 4) (p + 3) = 0 of the line is x + y = 1 ⇒ 4x + 3y = 24
p = 4 or –3 6 8
If the slope is 4, and x - intercept is –3, then the equation of When x – intercept is 7 then y – intercept is also 7.
the line is y = 4x + c where c is the y–intercept. When y = 0, then the equation of the line is x + y = 7. Choice (D)
x = –3. ∴ c = 12. So, the line is y = 4x + 12, if the slope is
–3, and x – intercept is 4. It can be similarly shown above 27. Parallel lines have equal gradients. True.
that the equation of the line can be y = –3x + 12 Perpendicular lines have the product of their gradients as
Choice (C) –1. True.
A line parallel to the x – axis has its gradient as 0. So, it is
20. Gradient of the line joining the points (4, 7) and (6, 11) is also true.
11 − 7 ∴ None of the three statements is false. Choice (D)
= =2
6−4 28. Given the lines 4x – 3ky + 4 = 0 → (1) and
Let (x, y) be any point on the line. Then 2x – 5y + 1 = 0 → (2) which intersect at a point whose
y −7 x-coordinate is twice its y - coordinate.
2= ⇒ y = 2x – 1 Choice (D)
x−4 So, let the point of intersection be (2p, p).
As (2p, p) lies on (2), 4p – 5p + 1 = 0 ⇒ p = 1
21. Let A=(4, –5), B = (0, 0) and C = (5, 4) ∴ The point of intersection is (2, 1).
AB = (0 − 4)2 + (0 − (− 5)2 ) = 41
Now this point (2, 1) also lies on (1).
⇒ 4(2) – 3k (1) + 4 = 0 ⇒ k = 4 Ans: (4)
BC = (5 − 0) + (4 − 0)
2 2
= 41 29. At the intersection of the line with the X – axis,
y = 0. ∴ 10x = 30 Y
AC = (5 − 4) + (4 − (− 5))
2 2
= 82
∴ AB = BC and AB2 + BC2 = AC2 (0, 10)
∴ the triangle is right angled and isosceles. Choice (B)
10 10x + 3y = 30
22. Given the parallel lines (opposite sides of a square)
8x + 4y – 12 = 0 and 2x + y + 4 = 0 ⇒ 8x + 4y + 16 = 0.
X
− 12 − 16 O 3 (3, 0)
Distance between opposite sides =
8 +4
2 2
⇒x=3
7  
= perimeter = 4  7  = 28 Ans: (28) At the intersection of the line with the Y – axis,
5  5 x = 0. ⇒ 3y = 30
  5
∴ y = 10.
23. The point of intersection of the lines 2x + 3y – 12 = 0 and 1
∴ Required area = (10)(3) = 15 sq units Choice (C)
3x + 4y – 17 = 0 is 2
Triumphant Institute of Management Education Pvt. Ltd. (T.I.M.E.) HO: 95B, 2nd Floor, Siddamsetty Complex, Secunderabad – 500 003.
Tel : 040–27898195 Fax : 040–27847334 email : info@time4education.com website : www.time4education.com SM1001962/51
 (2)(2) − (3)(−1) (2)(6) − (3 )(4 )  37. Equation of the line passing through (1, 4) and (4, 1) is
30. Required point =  , 
2−3 2−3 1− 4
  y−4= (x − 1) (Two point form)
4 −1
= (−7,0) Choice (D)
⇒ y − 4 = − (x − 1)
31. Area of the triangle formed ⇒ x + y − 5 = 0. Choice (A)
1 [(a + 1) − (a − 1)] [(a − 1) − (a + 3 )]
= 38. Given points A (0, 0), B = (−2, 3) and C (6, −9)
2 [(a + 2) − (a + 1)] [(a + 1) − (a − 3 )]
−3
slope of (AB) =
1 2 −4 1
= = |2(4) – (–4)(1)| 2
2 1 4 2 −3
slope of (BC) = .
= 1 |8 + 4| = 6 sq units Ans: (6) 2
2 ∴The points are collinear and will form a straight line.
Choice (D)
32. Let the x-intercept be a, then the y – intercept is a + 12.
Then the equation of the line is x + y =1 39. The equation in the new system is f (X, Y) = aX + bY + C
a a + 12 ∴ The equation in the original system is f(x, y) = 0, where
(2, 12) is a point on it ⇒ 2 + 12 = 1 X = x– h and Y = y – k i.e., X = x +1 and Y = y – 2
a a + 12 ∴ f(x, y) = a(x + 1) + b(y – 2) + c = ax + by + a – 2b + c = 0
Choice (B)
i.e., 2a + 24 + 12a = 12a +a2
a2 − 2a − 24 = 0 ⇒ (a – 6) (a + 4) = 0 ⇒ a = 6 or a = −4
40. f(X, Y) =X2 +Y2 = 2
When a = 6 then b = 18. Then the slope of the line is
X = xcos60° + ysin60°, Y = –xsin 60° + y cos 60°
−b −18
= = −3, which is negative. x + 3y − 3x + y
a 6 X= , and Y =
2 2
When a = −4 then b = 8. Then the slope of the line is
2 2
−b −8  x + 3y   
= =2 ∴f   +  − 3x + y  = 2
a −4  2   2 
As the slope is positive, the slope of the line is 2. Ans: (2)    
4x 2 + 4y 2
33. ⇒ =2
Y 4
⇒x +y =2
2 2
Choice (B)
(0, k)
41.
Y
150°
X
O (h, 0)
(2, 2)
(0, 2) C

Gradient of the line = tan 150° = – 1
3
k −0 1 P
Gradient of the line = =– ⇒k= 1 h
0 −h 3 3 • • X
0
h (3, 0)
Given h + k = 3 ⇒ +h=3
3 The given circle is shown in the figure above.
3 3 ( 3 − 1) 3 3 ( 3 − 1)
The shortest distance from origin to the circle is OP = OC – PC
3 3
( 3 + 1)( 3 − 1)
h= = = Choice (A) Centre of the circle is C (2, 2)
3 +1 2
∴ OC = 2 2 + 2 2 = 2 2 and PC is radius = 2

34. Gradient of 6x – 3y – 3 = 0 i.e., y = 2x –1 is 2. Required ∴ The required distance OP = 2 2 − 2 = 2 2 − 1 ( )


1 Choice (D)
equation will have a gradient of – . Its equation is
2 42. Any secant of a circle must be closer to the circle's centre
1 y −1 than any tangent to the circle. In the given problem, the
– = ⇒ 2y + x = 3.
2 x −1 circle is centered at the origin. The origin must be closer to
Choice (B) the secant than the tangent.
The distance between the origin and the line
35. The point of intersection of 2x + 3y – 13 = 0 and 5(0) − 4(0 ) − 20
3x + 2y – 12 = 0 is (2, 3) 5x – 4y – 20 = 0 is
5 2 + (− 4)
2
Line 2x + 6y + k = 0 passes through (2, 3)
⇒ 2 (2) + 6 (3) + k = 0 20
k = –22 Choice (D) i.e, . The distance between the origin and the line
41
36. The point of intersection of 2x + 3y – 8 = 0 and 2y – 3x – 1 = 0 5(0 ) − 4(0 ) + 40 40
5x – 4y + 40 = 0 is i.e
is (1, 2).
4−2 5 2 + (− 4 )2 41
Gradient of the required line = =1
3 −1 The line 5x – 4y – 20 = 0 is closer to the origin than the
other line. ∴ This must be secant and the other line must
Required equation is 1 = y − 2 ⇒ y = x + 1 be tangent. Radius = Distance from the centre to the line
x −1
Choice (A) 5x – 4y + 40 = 0. ∴ Radius = 40 Choice (A)
41
nd
Triumphant Institute of Management Education Pvt. Ltd. (T.I.M.E.) HO: 95B, 2 Floor, Siddamsetty Complex, Secunderabad – 500 003.
Tel : 040–27898195 Fax : 040–27847334 email : info@time4education.com website : www.time4education.com SM1001962/52
43. The given circles are x2 + y2 – 8y + 12 = 0 and x2 + y2 + 6y = 0

–3

i.e., x2 + (y – 4)2 = 22 and x2 + (y + 3)2 = 32


centre C1(0, 4) and r1 = 2 and C2 (0, –3) and r2 = 3
As the centers lie on the y – axis, PQ is the shortest distance between the circles.
C1 C2 = 4 – (–3) = 7
PQ = C1 C2 – (r1 + r2) = 7 – (2 + 3) = 2 Ans: (2)

44. 3x + 4y = 10 ----- (1) and my – x + 4 = 0 ------- (2) meet at Y


only one point
Solving (1), (2), we have 3(my +4) + 4y = 10
−2 10m + 16 P (0, 2)
y= ,x= •
3m + 4 3m + 4

y is integer only when m = –1 and m = –2. P
For m = –1, y = –2, x = 6
For m = –2, y = 1, x = 2 •
(3, 0)
X
0
∴ Only two integral values of m are possible. Ans: (2) (4, 0)

45. The points we have the consider must satisfy the condition
x2 + 2y2 < 24.
x2 and 2y2 must both be less than 24 i.e
y2 < 12 and x2 < 24 Choice (D)
If y2 = 0, x2 < 24 ∴ y = 0 and x = 0, ± 1, ± 2 ± 3, ± 4
If y2 = 1, x2 < 22. ∴ y = ± 1 and x = 0, ± 1, ± 2 ± 3, ± 4 49. From statement Ι alone, the area cannot be found as the
If y2 = 4, x2 < 16. ∴ y = ± 2 and x = 0, ± 1, ± 2 ± 3 equation of only one side is known.
If y2 = 9, x2 < 6. ∴ y = ± 3 and x = 0, ± 1, ± 2 From statement ΙΙ alone, the area cannot be found as the
A total of 9 + 2(9 + 7 + 5), i.e., 51 points satisfy the given equations of only two sides are known.
condition. Ans: (51) From Ι and ΙΙ together, the area can be found as all the
three equations are known.
Solutions for questions 46 to 50: Choice (C)

46. 50. (1, 8)


D • C
A
(4, 7)

(1, 2) B C

Unless we know the coordinates of C, it is not possible to A (1, 4) B
find the centroid.
∴ using both the statements also we cannot solve From statement Ι,
The distance between the midpoints of the sides is length
Choice (D)
of the side of square the length of the square = 4.
47. Using both the statements together we can find the ∴ Area = 16 sq units. Statement Ι alone is sufficient
equation of the line using point slope form and then we can From statement ΙΙ
check whether (7, 2) lies on L or not. We can find the length of diagonal
Hence, both statements together are sufficient. From that we can find area also.
Choice (C) ∴ Statement ΙΙ alone is also sufficient
Hence, either of the statements is sufficient to answer the
48. Even after using both the statements, P can lie in the 1st or question.
in the 2nd quadrant. Choice (B)

Triumphant Institute of Management Education Pvt. Ltd. (T.I.M.E.) HO: 95B, 2nd Floor, Siddamsetty Complex, Secunderabad – 500 003.
Tel : 040–27898195 Fax : 040–27847334 email : info@time4education.com website : www.time4education.com SM1001962/53
Chapter – 7 Given secθ = 17 and θ ∉ Q1
(Trigonometry) 8
⇒ θ ∈ Q4 (secθ is positive in Q1 and Q4)
Concept Review Questions
8
As a cotθ is negative in Q4, cotθ = − Choice (B)
Solutions for questions 1 to 35: 15
c c
 2π   2π  180 ° 12. a = cos10° – sin10° and b = cos70° – sin70°
1.   =   × c = 120° Ans: (120) if 0° < θ < 45°, sin θ < cosθ
 3   3  π
if θ = 45°, sin θ = cosθ
if 45° < θ < 90°, sin θ > cosθ
 c  c
2. 300° = 300° ×  π  =  5π  Choice (B) ∴a > 0 and b < 0. Choice (C)
 180°   3 
 
13. (180 + θ) ∈ Q3 and cot is positive in Q3.
3. 1 revolution = 360° or 2πc Choice (C) ∴cot (180 + θ) = cotθ Choice (B)

4. Minute hand covers an angle of 6° per minute. 14. (1 + sinθ) (1 − sinθ) sec θ2

In 12 minutes it covers an angle of 12 × 6 = 72° (1 − sin2θ)sec2θ = (cos2θ) (sec2θ) = 1 Choice (D)

= 72 × π = 2π Choice (C) 15. (a) cosec(330)° = cosec(360° − 30°)


180 5 = − cosec30° as (360° − θ) ∈ Q4 = −2 Ans: (–2)
(b) sec(1020)° = sec(3.360° − 60°) = sec60° = 2
5. Hour hand covers an angle of 1 ° every minute. ((360° − θ) ∈ Q4) Ans: (2)
2

In 30 minutes it covers 30 × 1 = 15° = π Choice (A) 16. If the angles of a triangle are in 1 : 2 : 3 ratio then the
2 12 angles of the triangle are 30°, 60° and 90°.
The ratio of the sides of the triangle is 1: 3 : 2.
6. Let the right angled triangle be ABC. C
The given sides are AB = 8 and BC = Choice (D)
17 by Pythagoras theorem.
17. For any value of θ sinθ and cosθ lie between −1 and 1
We have AC2 = BC2 − AB2 = 172 − 82
17 whereas tanθ and cotθ vary from −∞ to ∞.
AC = 225 = 15 Cosecθ and secθ do not lie between −1 and 1
AC 15 So cosecθ = 1 is not possible.
sinθ = = Choice (D)
BC 17 2
θ
And tanθ = 15 A 8 B
8 18. We know that sec2θ − tan2θ = 1
Choice (C) (secθ + tanθ) (secθ − tanθ) = 1
ab = 1 Choice (B)
7. We know that cosθ is positive in Q1 and Q4 and tanθ is
negative in Q2 and Q4. ∴ θ is in Q4 Choice (D) 19. Given cosec4θ + cot4θ − 2cosec2θcot2θ
= (cosec2θ − cot2θ)2 = 12 = 1 Ans: (1)
8. 500° lies in Q 2 ∴ tan 500° < 0
–200° lies in Q 2 ∴ sin –200° (and cosec –200°) > 0 20. Given secθ = −2 and cotθ = − 1
–500° lies in Q 3 ∴ tan –500° (and cot –500°) > 0 3
–400° lies in Q 4 ∴ cos –400° (and sec –400°) > 0 sin θ = cosθ tanθ
∴The statement in choice (D) is false. Choice (D)
1 1 3
= = = Choice (B)
9. sec θ. cot θ  1  2
y ( −2) − 
 3 


21. cosec = cosec 135°
4
cosec (180 – 45°) = +cosec45° = 2 Choice (A)
x
0
1
22. Given cosecθ – cotθ = → (1)
2
∴ cosecθ + cotθ = 2 → (2)
We know that cosx is symmetrical to x-axis and meets the
1
X-axis at infinite number of points. Choice (D) (∵ cosecθ + cotθ = )
cos ecθ − cot θ
sin θ × cos θ 1 tan θ 5
10. = tanθ = Choice (C) solving (1) and (2) we get cosecθ =
cos θ sec θ
1 + tan 2 θ 4
4
sinθ = Ans: (0.8)
11. 5

59
23. Given, tan2θ + 2sec2θ =
16
17 59
17 2 − 8 2 tan2θ + 2(1 + tan2θ) = (∵ sec2θ = 1 + tan2θ)
16
θ 9 3
tan2θ = ⇒ tanθ = ± Ans: (0.75)
16 4
8
Triumphant Institute of Management Education Pvt. Ltd. (T.I.M.E.) HO: 95B, 2nd Floor, Siddamsetty Complex, Secunderabad – 500 003.
Tel : 040–27898195 Fax : 040–27847334 email : info@time4education.com website : www.time4education.com SM1001962/54
24. We know that –1 ≤ sinθ ≤ 1 and – 1≤ cosθ≤1 Let BC be the height of the tower.
From options, secθ = 2/5 ⇒ cosθ = 5/2 is not possible. AB = 50 cm and θ = 60°
Choice (D) BC
∴tanθ = BC ⇒ tan 60° = = 50 3 Choice (D)
AB 50
25. sec46° = sec (90 – 44)° = cosec44°
cosec46° = cosec(90 – 44) = sec44° 32. C
∴ sin44° sec46° + cos44° cosec46°
sin44°cosec44°+ cos44°sec44°= 1 + 1 = 2 Ans: ( 2)

26. We know that tan (90 – θ) = cotθ


tan89° = tan(90 – 1°) = cot1°
tan88° = tan(90 – 2°) = cot2° Pole
tan46° = tan (90° – 44°) = cot44°
∴ tan1° tan2° tan3° . . . . tan46° tan47° . . . tan8 8° tan89°
= tan1° tan2° . . . . tan 44° cot 44° cot43° . . .c ot2° cot1°
= (tan1° cot1°) (tan2° cot2°) (tan3° cot3°)…………. A Shadow B
(cot44° tan44°) = 1 Ans: (1) Let AC be the height and AB be the length of the shadow
4 and θ be the angle of elevation of the sun.
27. cosβ = − ⇒ 180° < β < 270° ⇒ sinβ < 0 – (1) If m is the height of the tower then
5
2 AB = 1 h
 4 9 3
sin2β = 1 – cos2β = 1 –  −  =
 5 25
tanθ = AC = h
−3 AB 1
sinβ = (From (1))
5 3
−5 cos β −4 / 5 4 ⇒ tanθ = 3 and θ = 60°. Ans: (60)
∴ cosecβ = and cotβ = = =
3 sin β − 3/5 3
−5 4 −1 33. Let AB denote the pole with B being its foot.
cosecβ + cotβ= + = Choice (A) Let C denote the point. A
3 3 3
AB
= tan 60°
28. The complement of θ is 90 – θ BC
The supplement of θ is 180 – θ 300
= 3 300 m
2 BC
Given, 90° – θ = (180° – θ)
5 300
BC = = 100 3 m
450° – 5 θ = 360° – 2 θ 60°
3
90° = 3 θ ⇒ 30° = θ Ans: (30) C
Required distance B
29. Given, sinα + sinβ + sinγ = 3 ⇒ sinα = sinβ = sinγ = 1 = 100 3m
α = β = γ = 90°. Choice (C)
α β γ
= = = 45° 34.
2 2 2
B
α β γ
cot + cot + cot = cot45° + cot45° + cot45°
2 2 2 60°
=1+1+1=3 Ans: (3)
18
1
30. We know that area of ∆BC = absinc
2 30°
Here a = 5, b = 3 2 and ∠C = 45° A C

⇒∆=
1
.(5).(3 2 ) sin45° Let AB be the ladder. Given, ∠ABC = 60°
2 ∴ ∠BAC = 30°
1 18
= (.5) (3 2 ) 1 = 7.5 sq. untis. Choice (D) In ∆ABC sin30° =
2 2 AB
18
AB = = 36 m Choice (A)
30. Given a = 5, b = 3 2 and ∠C = 45° 1
By cosine rule we have c2 = a2 + b2 − 2abcosC 2

( ) 2
= 52 + 3 2 − 2 (5) (3 2 ).cos45 = 25 + 18 − 30 35. Let RS be the pole.
Let P and Q be R
c = 13 ⇒ c = 13
2
Choice (B) points
as shown above.
31. C PS + SQ = 100
In ∆PSR, In ∆RSQ
RS
tan60° =
PS
RS 60° 45°
tan45° =
SQ
P S Q
RS
PS = 100 m
tan 60 °
60° RS
A B SQ =
5 tan 45 °
Triumphant Institute of Management Education Pvt. Ltd. (T.I.M.E.) HO: 95B, 2nd Floor, Siddamsetty Complex, Secunderabad – 500 003.
Tel : 040–27898195 Fax : 040–27847334 email : info@time4education.com website : www.time4education.com SM1001962/55
RS RS secθ + tanθ = p then secθ − tanθ = 1
+ = 100 8.
tan 60 ° tan 45° p
 1  (As sec2θ − tan2θ = 1)
RS + 1 = 100 p2 + 1
 3  ⇒ 2secθ = p + 1 ⇒ secθ =
 
p 2p
100 3 100 3 ( 3 − 1) 2 p
RS = = = 50 3 ( 3 − 1) ∴ cosθ = . Choice (C)
3 +1 ( 3 + 1)( 3 − 1) p2 + 1
Choice (C)
9. Since x and 1 are positive,
Exercise – 7(a) x
1
Solutions for questions 1 to 30: x+ ≥ 2 x × 1 (∵ A.M ≥ G.M)
x x
1. Let the measures of angles of the given triangle be 20°, 1
⇒ x+ ≥ 2.
(20 + d)° and (20 + 2d)°; d being the common differ ence x
∴ 20° + (20° + d) + (20° + 2d) = 180°
1
⇒ d = 40° Now sinθ = x + ,
x
∴ The measure of the greatest angle
⇒ sinθ ≥ 2, this is not possible as the maximum value that
5 πc
= 20° + 2 × 40° = 100° = sinθ can take is 1.Hence, no such value of x exists.
9 Choice (B)
Ans: (5)
10. Consider an acute angle triangle ABC. Each angle is less
−b than 90° and A + B + C = 180°.
2. sinθ + cosθ = , sinθ cosθ = c 1
a a If sinθ > , then θ > 45° and if
Now, (sinθ+ cosθ)2 = sin2θ + cos2θ + 2sinθ cosθ 2
b2 b2 a + 2c 1
2c sinθ < , then θ < 45°
= 1+ ⇒ =
a 2 a a2 a 2
∴ b2 = a2 + 2ac. Choice (A) Choice (A): If sinA <
1
, sinB <
1
and sinC <
1
, then
2 2 2
° A, B, C < 45° and hence A + B + C ≠ 180°
3. tan 22 1 = 1 − cos 45° = 1 − (1/ 2 )
1 + cos 45° ∴Choice (A) cannot be true.
2 1 + (1/ 2 )
1 1
Choice (B): Let sinA < , sinB < , hence A < 45° and
2 −1 2 −1 2 2
= = = 2 − 1. Choice (C)
2 +1 1 B < 45°, or A + B < 90°.
This implies that C > 90°, this is not possible as ∆ABC is
acute angle triangle.
4. 3tan2θ − 1 = 0, ⇒ tanθ = 1 ∴ Choice (B) is not true.
3 1 1 1
since ‘θ’ lies in the III quadrant cosecθ = −2. Choice (C): If sinA > , sin B > and sinC > ,
Choice (D) 2 2 2
then A, B, C > 45°, this is possible for instance f or
5. 13sinθ − 12 = 0 A = B = C = 60°
12
∴ choice (C) can be true.
⇒ sinθ = and ‘θ’ is acute 1 1
13 Choice (D): If cosA > , cosB > , then A, B < 45°,
2 2
hence A + B < 90° Or C > 90°
13
12 This is not possible as ∆ ABC is acute angle triangle.
∴ Choice (D) is not true. Choice (C)
θ 11. x = cos50° + cos55° + cos60°
5 y = sin20° + sin25° + sin30°
5 12 Since cos60° = sin30°, we compare the remaining ter ms.
− The cosine function is a decreasing function from 0° to 90°,
cot θ − tan θ 12 5 = 25 − 144 = −17 .
∴ = 1 1
sec θ − cos ecθ 13 13 156 − 65 13 and since cos60° = ; cos50°, cos55° > ;
− 2 2
5 12
⇒ cos50° + cos55° > 1.
Choice (B) The sine function is a increasing function from 0° to 90°,
6. In the cyclic quadrilateral ABCD, sum of the opposite 1
and since sin30° = ; , sin20°+ sin25° < 1.
angles is 180° hence A + C = 180° and B + D = 180° 2
As A + C = 180°, cosC = (180° − A) = −cosA x
∴ x > y ⇒ > 1. Choice (A)
As B + D = 180°, cosD = cos (180° − B) = −cosB y
∴ cosA + cosB + cosC + cosD = 0.
Ans: (0) 1 1 27
12. ∆ = ; ab sinC = ; ⋅ 9⋅6 sin45° = sq. units.
2 2 2
7. Since sinθ and cosecθ are the roots of the equation
Choice (C)
cx2 + ax + b = 0; sinθ ⋅cosecθ = b (product of roots)
c 13. Since the triangle is a right-angled triangle at C, the
⇒ b = c, as sinθ ⋅ cosecθ = 1. hypotenuse ‘C’ is the longest side.
Choice (A) C= a2 + b2 ⇒ c = 65 units. Choice (B)
nd
Triumphant Institute of Management Education Pvt. Ltd. (T.I.M.E.) HO: 95B, 2 Floor, Siddamsetty Complex, Secunderabad – 500 003.
Tel : 040–27898195 Fax : 040–27847334 email : info@time4education.com website : www.time4education.com SM1001962/56
14. (i) The given graph is a reflection of the graph of y = cosx, 19. Let AB and CD represent the two towers.
in x-axis, hence the equation of the given graph can D
be obtained by changing the sign of y in y = cosx, i.e.,
y = –cosx. Choice (B) B
(ii) The given graph represents the absolute values taken h2
by cos2x (as cos(2π/4) = 1), hence the equation h1
y = cos 2x. Choice (B)
30° 45°
(iii) The graph represents a case where in the function A 300 P 300 C
y = sinx, the variables are interchanged, hence the
equation x = siny. Choice (C)
h1
In ∆APB, tan30° =
cos( 90 − 70 ) + sin 50 sin 70 + sin 50 2 sin 60 cos 10 300
15. = =
sin 20 + cos( 90 − 40 ) sin 20 + sin 40 2 sin 30 cos 10 300
h1 = m
3 /2 3
= = 3 Choice (B)
1/ 2 h2
In ∆CPD, tan45° = ⇒ h2 = 300 m
6 6 2 2 3 2 2 2 2
300
16. sin x + cos x = (sin x + cos x) – 3 sin x cos x (sin x + cos x)
300
∴ h1 : h2 = : 300 = 1 : 3 Choice (B)
= 1− (sin 2 x )2
3
3
4
3
Max value = 1 − .0 (∴ maximum is obtained when sin 2x 20. A x C
4
is minimum, i.e, sin 2x = 0)
∴Maximum value = 1
Ans: (1)

[
17. h(y) = 3 sin y + cos y ] 45°
30°

sin2x ≤ | sin x| P 1.5 B x D


cos2x ≤ | cos x| Let A and C represent the initial and final positions of the
∴ sin2 x + cos2 x ≤ | sin x| + | cos x| aeroplane and P the point of observation.
Distance travelled = AC = x
i.e 1 ≤ | sin x| + | cos x| ⇒ | sin x| + | cos x| ≥ 1
Given, CD = AB = 1.5 km.
 
2  sin x + cos x 
In ∆PCD; tan30° = 1 ⋅ 5
Also sinx + cos x =
 
 2 2  1⋅ 5 + x
= 2 ( sinx cos π/4 + cosx sinπ/4) = 2 sin  x + π  ⇒ 1⋅5 + x = (1⋅ 5) 3

( 3 − 1)
 4  3
∴x = 3 (1 ⋅ 5) − 1 ⋅ 5 =
∴The maximum sum is 2 ; | sin x|+| cos x| ≤ 2 2
Given h(y) = 3 [| sin y | + | cos y |] dis tan ce 3 ( 3 − 1) 3 −1
speed = = = km/sec.
∴ minimum value of h(y) is = 3 and maximum value is 3 2 time 2 9 6
Choice (A)
∴ 3 ≤ h(y) ≤ 3 2 Choice (A)
21.
18. Let AE and BD represent the pole and the tower A
respectively.
D
30°
x
x
30°
E
C
(POLE)

20 m 30° 60°
20 m •B
P x Q x/2
(TOWER)
60°
Let AB represent the lighthouse, P and Q be the points of
observation. In ∆ APQ,
A B
∠APQ = ∠PAQ = 30°
x ⇒ AQ = PQ = x
In ∆DCE; tan 30° = , and In ∆ AQB,
AB
x
x + 20 x x + 20 cos60° = BQ ⇒ BQ =
In ∆ABD; tan 60° = = x 2
AB tan 30° tan 60° The steamer takes 10 minutes to travel from P to Q (i.e., a
x + 20 10
⇒ 3x = distance of x), then it will take i.e., 5 minutes to travel
3 2
⇒ 2x = 20 or x = 10 m ; QB (x/2).
⇒ x = 10. ∴ It takes 15 minutes for the steamer to travel from P to B.
∴height of tower is 30m. Hence, at the instant when the steamer is at P, the time is
Ans: (30) 11:45 a.m. Ans: (11, 45)

Triumphant Institute of Management Education Pvt. Ltd. (T.I.M.E.) HO: 95B, 2nd Floor, Siddamsetty Complex, Secunderabad – 500 003.
Tel : 040–27898195 Fax : 040–27847334 email : info@time4education.com website : www.time4education.com SM1001962/57
22. Let PQ be the tower and A, B be the points of observation. 26. tan 2 πx + cos 2 πy = 0 ⇒ tan 2 πx = cos 2 πy = 0
Now, consider the figure below;
P ⇒ tanπx = cosπy = 0
tanπx = 0 implies πx = 0, ±π , ±2π ……and
60° 30°
π 3π 5π
cosπy = 0 implies πy = ± , ± ,± ..
2 2 2
45°
∴x = 0, ±1, ±2, ……..and y = ± 1 , ± 3 , ± 5 ……..
2 2 2
The points which satisfy x2 + y2 ≤ 9 must be such that
x2 ≤ 9 and y2 ≤ 9 i.e, x ≤ 3 and y ≤ 3
60° 30°
• The possible values of (x, y) are
B Q C A  1  3  5  1
30 m  0, ± ,  0, ± ,  0,± ,  ± 1, ± ,
 2  2  2  2
Given ∠APB = 2 ∠APC.
∴ PC is the angle bisector of ∠APB.  3 5 1  3
 ± 1, ± ,  ± 1, ± ,  ± 2, ±  ± 2, ± 
⇒ AP : BP = AC : BC  2  2 2  2
PQ PQ ∴ (x, y) has 26 possible values. Ans: (26)
In ∆BPQ, sin60° = and in ∆APQ, sin30° =
BP AP
∴ AP : BP = sin60° : sin30° = 3 :1
2 + 2 + 2 + 2 cos 2θ = 2 + 2 + 2( 2 cos θ)
2
27.
⇒ AC : BC = 3 :1
But given, BC = 30 m, hence AC = 30 3 m . = 2 + 2.2 cos 2 θ / 2 = 2 + 2 cos θ / 2
Choice (A)
= 2.2 cos2 θ / 4 = 2 cos θ/4 Choice (C)
23. Given L = 3 h C
28. 4 sinA + 6 cosB = 8 and 4 cosA + 6 sinB = 6
(4 sinA + 6 cosB)2 + (4 cosA + 6 sinB)2 = 82 + 62
h 1
tanθ = = 16(sin2A + cos2A) + 36 (cos2B + sin2B) + 48 sinA cosB + 48
L 3 sinB cosA = 100
h 16(1) + 36(1) + 48 sin(A + B) = 100
 1  Sin(A + B) = 1⇒ A + B = 90° ⇒ C = 90°. Choice (C)
θ = tan−1  
 θ
 3 5 sinQ + 4 sin R 33
B L A
29. =
⇒ θ = 30°. Ans: (30) 5 sinQ − 4 sin R 13
24. Given side AB subtends and angle of 60° at the top of the ⇒ 65 sinQ + 52 sinR = 165 sinQ – 132 sinR
pole P. sinQ 46
⇒ =
∴ APB is an equilateral triangle. sin R 25
∴ AP = 5m
q r  p 
E is mid point of diagonal From the sin rule, =  = 
1 5 sinQ sin R  sin P 
⇒ AE = 5 2 = m
2 2 q sinQ 44
= =
In right triangle APE r sin R 25
2 p, q, r are integers
 5 
PE2 = AP2 – AE2 = 52 –   Min (PQ + PR) = Min(q + r) = 46 + 25 = 71. Ans: (71)
 2 
 
30. E = 10 sinx cosx (5 + sinx cosx)
5
∴ PE = m or 2.5 2 m. Ans : (2.5) sin 2 x
2 = 5 sin2x (5 + )
2
25. From the given data in ∆ PCD ∠CPD = 30°, CP = DP –1 ≤ sin 2x ≤ 1
∠PCD = ∠PDC = 75° and CD = 6m Max (E) occurs when sin2x is maximum.
∴ CD PC  11  55
= (sine rule of triangle) Max (E) = 5   = 2 Choice (C)
sin ∠CPD sin 75 o 2
6 PC 6 3 + 1 = PC
i.e., = ⇒ ×
sin 30 0 sin 75 0 1 2 2 Exercise – 7(b)
2
  Solutions for questions 1 to 40:
∴ PC = 6  3 + 1  m
 2 
 5 1 5
From ∆ BCE, ∠B = 90° 1. Given sinθ +cosecθ = ⇒ sinθ + =
2 sin θ 2
⇒ EC2 = EB2 = 32 + 62
2sin2θ – 5 sinθ + 2 = 0
Right EC = 3 5 m 2sin2θ – 4 sinθ – sinθ + 2 = 0
In ∆ EPC, Height of the tower EP 2sinθ (sinθ – 2) –1(sinθ – 2) =0
 
2 (2sinθ – 1) (sinθ – 2) = 0
EP2 = PC2 – EC2 =  3 + 1  − (3 5 )2 1
 2  2sinθ – 1 = 0 or sinθ – 2 = 0 ⇒ sinθ = or 2
 2
= 18 (3 + 1 + 2 3 ) – 45 = 27 + 36 3 1
As –1 ≤ sinθ ≤ 1, sinθ =
= 9 (3 + 4 3 ) m. Choice (D) 2

Triumphant Institute of Management Education Pvt. Ltd. (T.I.M.E.) HO: 95B, 2nd Floor, Siddamsetty Complex, Secunderabad – 500 003.
Tel : 040–27898195 Fax : 040–27847334 email : info@time4education.com website : www.time4education.com SM1001962/58
1 cos 2 θ 1°
sec2θ + cot2θ = + 7. cot 22 =
cos θ
2
sin2 θ 2

 1
2  10  1
1−   1 + cos2 22  1+
1 − sin θ 2  2  1 + cos 450 2 +1
1
+ =
1
+ 2   2
= = = =
1 − sin 2 θ sin 2 θ  1
2 1  10  1 − cos 450 1 2 −1
1−   1 − cos2 22  1−
4  2  2
 
2  

( 2 + 1)
4 3 13
= + = Ans: (13) 2
3 1 3

2. –1 ≤ sinθ or cosθ ≤ 1
=
( 2 − 1)( 2 + 1) = 2 + 1. Choice (A)

3 ∠P + ∠Q + ∠R = 180°
∴ From the given options sin θ = is not possible 8.
2 Triangle PQR is right angled and isosceles.
Choice (D) ∴ one of ∠P, ∠Q and ∠R must 90° and each of the other
180° − 90°
5 two must be = 45°
3. cosec θ = 2
3 cos∠P + cos∠Q + cos∠R = cos90° + cos45°+ cos45°
cosec2θ – cot2θ = 1 1 1
16 = 0+ + = 2 Choice (A)
cot2θ = cosec2θ – 1 = 2 2
9
As θ lies in the 1st quadrant 9. Given that cosecθ + cotθ = x – 1 → (1)
cot θ > 0 As cosec2θ – cot2θ = 1
4 1
∴ cotθ = Choice (A) cosecθ – cotθ = .
3 cos ecθ + cot θ
1
4. sinθ + cosecθ + sin2θ + cosec2θ = 0 ⇒ cosecθ – cotθ = → (2)
x −1
1 1
⇒ sinθ + sin θ + 2
+ =0 By [ (1) + (2)] ÷ 2, we get
sin θ sin2 θ
1
sin θ + 1 ( x − 1) +
⇒ sinθ (sinθ + 1)+ =0 ( x − 1) ( x − 1)2 + 1
sin2 θ cosecθ = =
2 2( x − 1)
 1 
⇒ (sinθ + 1)  sin θ +  = 0
 sin 2 θ  By [ (1) – (2)] ÷ 2, we get
1
1 ( x − 1) +
⇒ sinθ + 1 = 0 or sinθ + =0 ( x − 1) ( x − 1)2 + 1
sin θ
2 cotθ = =
2 2( x − 1)
⇒ sinθ + 1 = 0 or sin3 θ + 1 = 0 ⇒ sinθ = –1
we know that –cot2 θ + cosec2 θ = 1 ⇒ cot2 θ = cosec2 θ – 1 ( x − 1)2 − 1 x 2 − 2x
Now, cosθ = = 2 Choice (C)
 1 
2 2( x − 1) x − 2x + 2
∴ cot θ = 
2
 –1 = 0.
 − 1 ( x − 1)2 + 1
Cotθ = 0. Choice (D) 2( x − 1)

b 12 13
5. secθ + tan θ = – 10. 13 sin θ – 12 = 0 ⇒ sinθ = , i.e cosecθ =
a 13 12
c  12 
2
 25 
sec θ tan θ =
a cos2θ = 1 – sin2θ = 1 –   =  
 13   169 
(secθ – tanθ) (secθ + tanθ) = 1
5

 (sec θ + tan θ)2 − 4 sec θ tan θ   −b  =1
∴ cosθ =
13
(As θ is acute, cosθ >0)
  a 
sin θ 12 5
2 tanθ = = and cot θ =
−b c a cos θ 5 12
  − 4 = −
 a  a b  5   12 
2  + 3 
Squaring on both the sides, 2 cos θ + 3 tan θ  13   5  = 1036
=
b 2 − 4ac a2
2 cos ecθ + cot θ 13 5 195
 a +
⇒ =  −  = 2 12 12
a2  b b Ans: (1036)
⇒ b2(b2 – 4ac) = a4
⇒ b4 = 4ab2c + a4 Choice (A) 11. Given PQRS is a cyclic quadrilateral ∠P + ∠R = ∠Q + ∠S = 180°
sin ∠P + sin ∠Q + sin (180° – ∠P) + sin (180° – ∠Q)
6. sin2θ. Sec3θ – cosec2θ. Cotθ = sin ∠P + sin ∠Q + sin∠P + sin∠Q
= sin120° sec180° – cosec120° cot60° (As sin (180° – θ) = sinθ) = 2 (sin ∠P + sin ∠Q)
 3   
(− 1) −  2  1  = − 3 − 2
Since the relation between P and Q, is not known the value
= Choice (B) of the given expression cannot be found
 2   3  3  2 3
     Choice (D)

Triumphant Institute of Management Education Pvt. Ltd. (T.I.M.E.) HO: 95B, 2nd Floor, Siddamsetty Complex, Secunderabad – 500 003.
Tel : 040–27898195 Fax : 040–27847334 email : info@time4education.com website : www.time4education.com SM1001962/59
12. cot6θ – cosec6θ + 3cosec2θ cot2θ 2 2
= – [cosec6θ – cot6θ – 3cosec2θ cot2θ] = = . Choice (A)
cot θ b
= – [(cosec2θ)3 – (cot2θ)3 – 3cosec2θ cot2θ]
− [(cosec2 θ) − (cot2)3 − 3 cosec2θ cot2 θ (cosec2θ − cot2θ)] 21. We know that
= – [+ 1 + 3 cosec2θ cot2θ – 3cosec2θ cot2θ]
= – 1. Ans: (–1) sin4 x + cos 4 x
≥ sin 4 x. cos 4 x (∴AM ≥ GM)
2
13. Given tan(α – 45°) + tan( α + 45°) = 0. sin x + cos x ≥ 2 Sin xcos x
4 4 2 2
From options put α = 0, tan (–45°) + tan(45) = 0 1
Put α = 90°, tan(90 – 45°) + tan(90 + 45) ≥ (sin2x)2
= cot45°–cot45° = 0 2
∴ α = 0°and 90°satisfy the given equation. Choice (D ) 1
But max value of sin 2x = 1 ⇒ sin4x + cos4x ≥
2
3
14. sinα = − 1
5 ∴ the minimum value of sin4x + cos4x is Ans: (0.5)
2
270° < α < 360° ⇒ cosα > 0. . .– (1)
2
 3 16 22. Given α + β = 180° and
cos2α = 1 – sin2α = 1 –  −  =
 5 25 −q
sum of the roots cosecα + cosec β =
4 p
cosα =
5 −q
⇒ cosecα + cosec (180 – α) =
5 sin α −3 / 5 3 p
∴ secα = and tanα = = =−
4 cos α 4/5 4 −q
1 cosecα + cosecα =
secα + tanα= Choice (A) p
2
−q
2cosec α = → (1)
2
p
 7  24
15. cosα = 1 − sin2 α = 1 −   = r
 25  25 Product of the roots cosecα.cosecβ =
p
sin α 7
tanα = = r
cos α 24 ⇒ cosec α cosec β = cosec α (cosec α) =
p
24 25 25
cotα = , cosecα = , secα = r
7 7 24 ⇒ cos ec 2α = → (2)
24 25
p
− 2
Required value = 7 7 = 4 Choice (A) −q r
7 25 21 From (1) and (2),   =
−  2p  p
24 24
⇒ q2 = 4pr Choice (C)

3 −1 1 23. PQRS is a cyclic quadrilateral


16. Given, cos(x + y) = siny =
2 2 2 ∠P + ∠R = ∠Q + ∠S = 180°
⇒ x + y = 75° ⇒ y = 30° cot∠P + cot ∠R = cot ∠P – cot∠P
∴ x = 45° (As cot(180 – P) = cot(P) = 0
∴ x = 45°, y = 30° Choice (A) similarly cot ∠Q + Cot ∠S = 0
∴ cot ∠P + cot ∠R – (cot ∠Q + cot ∠S) = 0
Ans: (0)
17. The minute hand covers an angle of 6° per minute.
∴ In 18 minutes the angle covered is 108°. Choice (B )
1 1
24. Area of the triangle = bc sin A = (4) (6) sin 30°
2 2
1 1 3  3  1  3 
18. log  ⋅ ⋅  = log   = log = 6 sq units. Ans: (6)
 .
 2 2 2   32  2  32 
25. Least angle of the triangle is the angle opposite to the least
Choice (D) side. Let this be θ
19. Let the angles be x, x + 30 and x + 60. (6 3 ) + 8 − 4
2 2 2
1.625
2(6 3 )(8 )
∴ x + (x + 30) + (x + 60) = 180 ⇒ x = 30 cosθ = =
Since one of the angles is 90°, say angle C, the pr oduct 3
cosA cosB cosC = 0, as cos90° = 0.  1.625 

θ = Cos–1 

Choice (A)  Ans: (1.625)
 3 
20. Given a = cosec θ
b = cotθ. 26. r2 = p2 + q2 – 2pq cos∠R
a +1 a −1  5 + 1
Now − = 82 + 102 – 2(8) (10)  
a −1 a +1  4 
 
=
a + 1 − (a − 1)
a2 − 1
= 164 – 40 ( 5 + 1)
= 124 – 40 5
2
=
cos ce2θ − 1 r = 2 31− 10 5 Choice (C)

Triumphant Institute of Management Education Pvt. Ltd. (T.I.M.E.) HO: 95B, 2nd Floor, Siddamsetty Complex, Secunderabad – 500 003.
Tel : 040–27898195 Fax : 040–27847334 email : info@time4education.com website : www.time4education.com SM1001962/60
27. The relation which best describes the graph is y = sin x 32. Let AE be the hill and BC be the tower

Choice (D) E 60o


15o
28. The relation which best describes the graph is y = cos x x
Choice (D) 15o C
90 D
29.
h
60o
A A B
Let ED = xm
ED x
From ∆EDC, Tan 15o = ⇒ 2- 3 =
DC DC
x
DC = (
= x 2+ 3 ) → (1)
B C 2- 3
AE 90
From ∆AEB, Tan 60o = ⇒ 3= (∵ AB = DC)
Let AB represent Ajay and BC represent his shadow. AB DC
AB 90
AB = BC tan ∠ACB = =1 DC = = 30 3 → (2)
BC 3
∠ACB = 45° Ans: (45)
From (1) and (2) (
x 2 + 3 = 30 3 )
30.
A E x=
30 3
= 30 3 2 - 3 ( )
2+ 3
height of the tower h = AD = AE − DE
( )
= 90 - 30 3 2 - 3 m = 30 3 - 3 2 - 3 ( ( )) m
2 km
(
= 60 3 - 3 m ) Choice (B)
75°
33. sec6θ – tan6θ – 3sec2θtan2θ
60°
= (sec2θ)3 – (tan2θ)3 – 3sec2θtan2θ(sec2θ – tan2θ)
B C D = (sec2θ – tan2θ)3 = 13 = 1
∵ ((a – b)3 = a3 – b3 – 3ab(a – b) Ans: (1)

Dis tan ce AE(in km) 34. In a triangle ABC, by Sine rule we have
Speed = = a b c
time 1
2
(in hrs) = =
sin A sinB sin C
= 2R

60 Given a = 3 3 and ∠A = 60°


2 2 2 2
Now AE = BD – BC = − = − a 3 3
tan 600 tan 75 0 3 2+ 3 ∴ = 2R ⇒ = 2R
SinA sin 60
4
=
(
3 2+ 3 )km R=
3 3
3
= 3 units. Choice (C)
4 480
Speed =
(
3 2+ 3 ) km/hr = 3 (2 + 3 ) km/hr 2

1 35. Given a = 4 2 , b = 4 3 , ∠A = 45°


120 a b
By sine rule =
Ans: (480) sin A sin B
4 2 4 3 4 2 4 3
31. ⇒ = = =
A sin 45 sinB 1 sin B
2
10 m 3
⇒ sinB = ⇒ ∠B = 60° or 120°.
2
15°
C Since ∠C is the greatest angle ∠B = 60°
B ∠C = 75°
Using sine rule we have

Let the ladder be AC Let the wall be AB a c 4 2 c


= =
( )
, i.e
10 10 10 2 + 3 sin A sin C sin 45o sin 75o
BC = =
tan 15° 2 − 3
=
2+ 3 2− 3
=
( )( ) =
4 2
1
=
c
⇒ c = AB = 2 ( 6 + 2 )
3 +1
10(2 + 3 )m 2
Choice (C) 2 2
Choice (D)
Triumphant Institute of Management Education Pvt. Ltd. (T.I.M.E.) HO: 95B, 2nd Floor, Siddamsetty Complex, Secunderabad – 500 003.
Tel : 040–27898195 Fax : 040–27847334 email : info@time4education.com website : www.time4education.com SM1001962/61
36. Let PQ be the pole and RQ be 39. Given 5cosθ + 12sinθ = 13
Q
the upper part of the pole. Dividing both sides by 13, we get
From the diagram RQ = RS. 5 12
cos θ + sin θ = 1………(1)
13 13
PS 5 12
In ∆PRS cos 30° = Let = cosα and sinα =
RS R 13 13
(1) ⇒ cosα cosθ + sinα sinθ = 1
2 3 20 ⇒ cos(θ – α) = 1 ⇒ θ – α = 2πn
= ⇒ θ = α + 2πn
2 RS
sin α 12
30° ⇒ tanθ = tanα = = Choice (B)
40 cos α 5
RS = m P 20 S
3 40. Given α, β are complementary angles, α + β = 90°
Ans : (40) cosα = cos(90° – β) = sinβ
cos2α + sin2β = sin2β + sin2β = 2sin2β
37. ∴ The maximum value of sinβ is 1
A D
∴ The maximum value of cos2α + sin2β is 2.
Ans: (2)

H H Solutions for questions 41 to 45:


C
30° 45°
41. Using statement Ι alone, cosθ > 0
B 90 m
E ⇒ θ is in 1st or 4th quadrant.
Hence, it is not in the third quadrant.
Choice (A)
Let the height of the buildings be H m.
42. Using statement ΙΙ alone, we can not answer as sinθ < 0
AB 1 ⇒ θ is in
In ∆ABC, = tan 30° =
BC 3 The expression equals 4cosAsinA.
When A = 0°, 4 cosA sinA = 0.
H 1 ∴ Statement Ι alone is sufficient to answer Q3 or Q4
∴ = i.e., BC = H 3m
BC 3 statement ΙΙ we can not answer. Choice (A)
DE 43. Using statement ΙΙ alone,
In ∆CDE, = tan 45° = 1
CE A = B = C = 60°.
H tanA + tanB + tanC = 3 3 .
= 1 i.e., CE = H m
CE With statement Ι alone we cannot answer
from the diagram, BC + CE = BE = 90 m Choice (A)
∴H 3 + H = 90 44. From statement Ι, we know that the angle of elevation and
90 the distance. So, we can find the height of the Statue of
H= Liberty.
3 +1 Similarly from statement ΙΙ, we can find the height of Eiffel
tower.
H = 45 ( 3 – 1) m Ans: (45)
∴ Using both the statements, we can find which is taller.
Choice (C)
sin5 x − cos5 x
38. f(x) = and
cos 2 x sin2 x 45. From the statement Ι,
In ∆ AQP, BQ = 30m, ∠A = 60° P
cos 5 x − sin5 x
g(x) = ∴ PQ = 30 3 m
sin3 x cos 3 x
π π ∴ Statement Ι is alone sufficient
< x < ----------------(1)
4 2
From the statement ΙΙ 60°
π
As θ Increases from 0 to , sinθ increases from 0 to 1 A Q
2 30 m
P
and cosθ decreases from 1 to 0. Also
π π 1
sin = cos = . M
4 4 2
π
∴ sinθ < cosθ when 0 ≤ θ ≤ and sinθ > cosθ when
4 30°
π π
≤θ≤ Q 20 m B
4 2
(1) implies sinx > cosx
In ∆BQM, BQ = 20m, ∠B = 30°
∴ sin5x > cos5x
A positive value ∴ QM = 20/ 3 and PQ = 40/ 3
f(x) = = a positive value
A positive value ∴ Statement ΙΙ alone is also sufficient.
Hence, either of the statements is sufficient to answer the
Also g(x) is a negative value.
question. Choice (B)
∴ f(x) > g(x) Choice (C)

Triumphant Institute of Management Education Pvt. Ltd. (T.I.M.E.) HO: 95B, 2nd Floor, Siddamsetty Complex, Secunderabad – 500 003.
Tel : 040–27898195 Fax : 040–27847334 email : info@time4education.com website : www.time4education.com SM1001962/62
Chapter – 8 a+b
11. Given: a⋆b =
(Operator Based Questions) a−b
Concept Review Questions 5+3 8 + 6 14
5∗3 = = 4 and 8∗6 = = =7
5−3 8−6 2
Solutions for questions 1 to 15:
4 + 7 − 11
1. Given: a α b = (ab) − (a + b) ∴ (5∗3)⋆(8∗6) = (4∗7) = =
Here, a = 8, b = 5
4−7 3
⇒ 8 α 5 = (8 . 5) − (8 + 5) = 27 Ans: (27) − 11
x= ⇒ [x] = [−3.66] = −3 Choice (A)
3
xy
2. Given: x − y =
x+y 12. Given: a $ b = (a + b)a−b
Here x = 8, y = 6 5 $ 3 = (5 + 3)5−3 = 82 = 64 Ans: (64)
8 × 6 48 24
∴x−y= = = Choice (B) 13. a × b = a2 − ab + b2 and a ÷ b = a + b
8 + 6 14 7
⇒ (a × b) (a ÷ b) = (a2 − ab + b2) (a + b) = a3 + b3
and a α β = a2 + ab + b2 and a β b = a − b
a+b (a α b) (a β b) = (a2 + ab + b2) (a − b) = a3 − b3
3. Given: a ↑ b =
a−b
(a × b) (a ÷ b) − (a α b) (a β b) a3 + b 3 − a3 + b 3 2b3
4+3 2
= 3
= = 2b
4↑3= =7 Ans: (7) b b b2
4−3 Choice (C)

4. Given: f (x, y) = y 14. Given: f (x, y) = xy and g (x, y) = yx


∴ f [3, f (4, f (5, 7) ) ] = f (3, f (4, 7) ) f (3, 4) = 34 = 81 and g (5, 2) = 25 = 32
= f (3, 7) = 7 Choice (D) g (2, 7) = 72 = 49
f (3, 4) − g (5, 2) 81 − 32
5. Given: a % b = (a + b) 2
∴ = =1 Choice (D)
7 % 3 = (7 + 3)2 = 100 Ans: 100 g (2, 7) 49
6. Given: x ↔ y = (xy)2
15. Given [1001%25 + 1002%25 + - - - - + 1025%25]%25
3 ↔ 4 = (3 × 4)2 = 144
i.e., 1001%25 ⇒ R when 1001 is divided by 25 which is 1
2 2
a 6 9 1002%25 ⇒ R when 1002 is divided by 25 which is 2
also a ↕ b =   ⇒ (6 ↕ 4) =   = “ “ “ “ “ “ “ “ “ “ “
b 4 4
“ “ “ “ “ “ “ “ “ “ “
(3 ↔ 4) 144 1024%25 ⇒ R when 1024 is divided by 25 which is 24
= = 64 Choice (C)
(6 b 4) 9 and 1025%25 ⇒ R when 1025 is divided by 25 which is 0
4 ∴ 1001%25 + 1002%25 + - - - - 1025%25
24(24 + 1)
7. Given: f (x, y) = Max of (x, y) = 1 + 2 + 3 + - - - - + 24 = = 300
⇒ f (8, 12) = Max (8, 12) = 12 → (1) 2
g (x, y) = L. C. M of (x, y) Required result is 300%25
g (24, 36) = L. C. M of (24, 36) = 72 → (2) ⇒ Remainder when 300 is divided by 25 which is zero.
h (x, y, z) = average of (x, y, z) Ans: (0)
8 + 21+ 7
h (8, 21, 7) = average of (8, 21, 7) = = 12 Exercise – 8(a)
3
∴ h {f (8, 12). g (24, 36), h (8, 21, 7) Solutions for questions 1 to 25:
= h (12, 72, 12)
12 + 72 + 12 24
= = 32 Ans: (32) 1. 24 → 3 = =4
3 2×3
4←2=4+2×2=8
8. Given: a ⊕ b = a2 + b2 8 ↓ 4 = 4 × 8 – 4 = 28
⇒ (5 ⊕ 3) = 52 + 32 = 34 28 ↑ 12 = 3 × 28 × 12 = 84 × 12 = 1008 Ans: (1008)
and (a ⊙ b) = 2ab
2. Consider choice (A):
(5 ⊙ 3) = 2.5.3 = 30 21 ↓ 7 = 4 × 21 – 7 = 77
∴ (5 ⊕ 3) − (5 ⊙ 3) = 34 − 30 = 4 Choice (D) 77 77
77 → 9 = =
2×9 18
9. Given: X (x, y) = x2 − y2
X (32, 20) = 322 − 202 = 1024 − 400 = 624 77/18 ↑ 6 = 3 × 77/18 × 6 = 77,
Y (a, b) = a + b which is a multiple of 11.
Y (32, 20) = 32 + 20 = 52 Choice (B):
X (32, 20) 624 21 21 3
= =12 Choice (C) 21 → 7 = = =
Y (32, 20) 52 2×7 14 2
3/2 ↑ 9 = 3 × 3/2 × 9 = 81/2
10. Given: P (x, y) = 2x + 3y
81/2 ↓ 6 = 4 × 81/2 – 6 = 156,
P (2, 5) = 2.2 + 3.5 = 19
which is not a multiple of 11.
Q (x, y) = 3x − 2y
Q (1, 1) = 3 − 2 = 1 Choice (C) ⇒ 21 ← 7 = 21 + 2 × 7 = 35
P (P (2, 5), Q (1, 1)) = P (19, 1) 35 ↓ 9 = 4 × 35 – 9 = 131
= 2 × 19 + 3.1 131 → 6 = 131/12
= 38 + 3 = 41 Choice (B) which is not a multiple of 11 Choice (A)
nd
Triumphant Institute of Management Education Pvt. Ltd. (T.I.M.E.) HO: 95B, 2 Floor, Siddamsetty Complex, Secunderabad – 500 003.
Tel : 040–27898195 Fax : 040–27847334 email : info@time4education.com website : www.time4education.com SM1001962/63
3. 14 ↓ 7 = 4 × 14 – 7 = 56 – 7 = 49 17. Consider choice (A):
a $ b = 3 $ 2 = (9 + 4)5 = (13)5 ≠ 55 . 5
∴ 14 ↓ 7 = 49 = 7
∴ Choice (A) is false.
9 ↑ 27= 3 × 9 × 27 = 729 Consider choice (B)
∴ 3
9 ↑ 27 = 3 729 = 9 a ∆ b = 39 – 4 + 29 – 4 = 35 × 25
= 243 + 32 = 275 = 55(a + b)
∴ 14 ↓ 7 − 3 9 ↑ 27 = 7 − 9 = −2 Ans: (–2) ∴ Choice (B) is true.
Choice (B)
4. In the given numbers 2 is repeated 9 times
⇒ honoured card occurred 9 times. Ans: (9) 18. Consider choice (A)
2 −b 2 2 −b 2
5. In the given series of numbers 5 is not followed by 3. aa = ba
Hence, the required number is zero. Ans: (0) This is true when a2 = b2.
∴ a ∨ b can be equal to a ∧ b
6. If he picks up a honoured card he writes it as 2 ∴ Choice (A) is false.
In the given number series, 2 occurs consecutively only Consider choice (B)
once. 2 2 2 2
∴ The required number is one Ans: (1) a ∆ b = aa −b + ba −b
Which is always greater than 0 as a, b are greater than 0
7. For a positive number 2x ≥ x + x, ∴ Choice (B) is true
for example, 1.2 + 1.2 = 2 × 1.2 = 2.4 = 2 Choice (B)
whereas 1.8 + 1.8 = 2 < 2 × 1.8 = 3.6 = 3
⇒ 2x + 2y ≥ x + x + y + y 19. Consider choice (A)
≥ x + y + x + y
≥ x + y + x + y as x + y ≥ x + y (
a $ b = 22 + 12 )
22 −12
= (5)3 = 125
Hence R(x, y) ≥ L(x, y) Choice (D) 2 −12 2 −12
a ∆ b = 22 + 12 = 23 + 1 = 9
8. For all integral values of x and y; R(x, y) = T(x, y). In the a$b 125
interval (0, 5) there are 6 integers. ∴ =
a∆b 9
So, the number of such pairs 6 × 6 = 36.
Choice (C) ∴Choice (A) is false.
Consider choice (B):
2 2
9. The order in which operations should be performed: for a = 1 and b = 1, a ∨ b = 11 −1 = 1
BOSAMD. 2 2
∴ 13 × 5 + 35 ÷ 8 − (2 × 5) and a ∧ b = 11 −1 = 1
= 13 × 5 + 35 ÷ 8 − 10 = 13 × 5 + 35 ÷ (−2) ∴a∧b+a∧b=1+1=2
= 13 × 40 ÷ (−2) = 520 ÷ (−2) = −260 ∴ Choice (B) is false Choice (D)
Ans: (–260)
20. Here a = 3, b = 4, h = 3

(h )
10. (15% 6) = L.C.M. of 15, 6 = 30
1
(20 ∼ 8) = G.C.D. of 20, 8 = 4 ∆2 = 2
+ ab
2 2
∴ 30 ∆ 4 = 303 – 43 = 27000 – 64 = 26936. Choice (A) a b
= 1/144 (9 + 12) = 21/144 > 0
11. L.C.M. of two distinct positive numbers is always greater
than their G.C.D. and ∇2 =
1
a 2b 2
(h 2
− ab )
∴ choices (A) and (B) are wrong
(a $ b) = (a + b)3 – (a – b)3 is always positive, as = 1/144 (9 – 12) = –3/144 < 0
Choices (A) and (B) are false.
a + b > a – b, ⇒ (a + b)3 > (a – b)3. Choice (C)
Also 21/144 > –3/144
12. Here a = 6, b = 36 ∴ ∆ 2 > ∇2 Choice (C)
Choice (A): 2 2
21. With the coefficients of x and y being interchanged the
⇒ (a % b) ÷ (a ∼ b) + a ∆ b = 36 ÷ 6 + (a3 – b3)
new equation will have a = –7, h = 4, b = 4
= 6 + (216 – 46856) = 6 – 46640 = –46634
Choice (B): ∆2 =
1
a 2b 2
(h 2
+ ab )
⇒ 3 (a % b) × (a ~ b) = 3 36 × 6 = 6 = a
1 −12
∴ Choice (B) is true Choice (B) = (16 – 7 × 4) =
49 × 16 49 × 16
13. G.C.D. × L.C.M. = product of two numbers
(a ∼ b) × (a % b) = ab ∇2 =
1
a 2b 2
h 2 − ab ( )
⇒ (a ∼ b) × (a % b) is divisible by both a and b.
Choice (D) 1 44
= (16 – (–7) × 4) =
49 × 16 49 × 16
14. p(f(x, x), g(x, –x)) = p(e2x, e2x)
∴ ∆ 2 < ∇2 Choice (C)
= loge(e2x, e2x)
= logee4x = 4x
= q(e6x, e2x) Choice (D) 2 2
22. D = h − ab
b
15. f(p(x, y), q(x, y)) = f(logxy, logx/y)
2
= elogxy + logx/y = elogx² = x² = 25 Ans: (25) ⇒ D/a = h2 − ab
ab

( )a2 −b2 2
= (0 + 1) = (1) ⇒ D/2a = h2 − ab = ∇
0 −1 −1
16. a $ b = a 2 + b 2 =1 ab
a∆b=0 +1 =1 0–1 0–1
⇒ D/2a = ∇ ⇒ D = 2a∇
∴a$b–a∆b=1–1=0 Ans: (0) Choice (A)
nd
Triumphant Institute of Management Education Pvt. Ltd. (T.I.M.E.) HO: 95B, 2 Floor, Siddamsetty Complex, Secunderabad – 500 003.
Tel : 040–27898195 Fax : 040–27847334 email : info@time4education.com website : www.time4education.com SM1001962/64
23. By definition c2 = c ⊗ c = d 5. Choice (A)
c3 = c2 ⊗ c = d ⊗ c = b 2
c4 = c3 ⊗ c = b ⊗ c = a 2↑3= (2) (3) = 4
3
∴ c4 = a
the least value of n = 4.  4  12
4 ↓ 5 = 3   =
Choice (B) 5 5
24. From table a5 = a, b4 = a, 4c = a and 9d = d 12  12 
→ 6 = 3   + 4(6) = 31.2
∴ (a5 ⊗ b4) ⊕ (4c ⊗ 9d) 5  5 
= (a ⊗ a) ⊕ (a ⊗ d) 31.2 ← 7 = 4(31.2) – 5(7) which is not an integer
=a⊕d=d=c⊗c
Choice (C) Choice (B)
2 → 3 = 3(2) + 4(3) = 18
25. ((b ⊗ c) ⊗ a) ⊕ ((a ⊕ b) ⊕ d) 18 ← 5 = 4(18) – 5(5) = 47
= (a ⊗ a) ⊕ (b ⊕ b)
 47 
=a⊕a =a Choice (D) 47 ↓ 6 = 3  = 23.5
 6 
Exercise – 8(b) (23.5) (7), 2
23.5↑ 7 =
Solutions for questions 1 to 30: 3
is not an integer.
1. 6 ⊕ 8 = HCF(63, 83) = [HCF(6, 8)]3 =8
Choice (C)
1
2
(6 ⊕ 8) 1/3
= 83 =2 2↑3= (2) (3) = 4
3
(2 ⊖ 4) 1/3 3
= [LCM (2 , 4 )] 3 1/3
= LCM(2,4) = 4
4 → 5 = 3(4) + 4(5) = 32
4 ⊗ 1 = 42 + 12 – 42 12 = 1  32 
32 ↓ 6 = 3  = 16
1⊖ 1 = LCM (13, 1)3 = 1 Ans: (1)  6 
16 ← 7 = 4(16) – 5 (7) = 29 is an integer
2. a ⊖ b = LCM (43, 63) = [LCM (4,6)]3 = 123 = 1728 Choice (D) is not an integer. Choice (C)
a ⊕ b = HCF (43, 63) = [HCF(4, 6)]3 = 23 = 8
6. Choice (A)
a⊖b 1728 2
= = 216 which exceeds 200 6↑1= (6) (1) = 4
a⊕b 8 3
∴ Choice (A) is false. 4 → 4 = 3(4) + 4(4) = 28
Choice (B)
(a + 1) ⊗ (b − 2) = (4 + 1) ⊗ (6 − 2) = 5 ⊗ 4
 28 
28 ↓ 1 = 3   = 84
= 52 + 42 − (5)2 (4)2 = –359  1 
∴ Choice (B) is false. 84 ← 7 = 4(84) – 5(7) = 336 – 35 = 301
Choice (C) ∴Choice (A) is not a perfect square.
(a − 1) ÷ (b + 2) = 3 ÷ 8 = 32 + 82 + (3)2 (8)2 = 649 Choice (B)
∴ Choice (C) is false. Choice (D) 2
6↑4= (6) (4) = 16
3. Choice (A)
3
Any two natural numbers have their HCF less than or equal 16 → 1 = 3(16) + 4(1) = 52
to their LCM  52 
∴ Choice (A) is always true. 52 ↓ 1 = 3   = 156
 1 
Choice (B)
p3 = q3 ⇒ p = q 2
156 ↑ 7= (156) (7) = 728
p ÷ q = 2q2 + (q2)2 = 99 3
(q2)2 + 2q2 − 99 = 0 ∴ Choice (B) is not a perfect square.
(q2 + 11) (q2 − 9) = 0 Choice (C)
q2 > 0 2
∴ q2 = 9 6↑7= (6) (7) = 28
3
∴ q = ±3
28 → 1 = 3(28) + 4(1) = 88
∴ Choice (B) is true.
Choice (C)  88 
88 ↓ 1 = 3   = 264
p3 = q3 ⇒ p = q  1 
p ⊗ q = 2q2 − (q2)2 = −8 264 ← 4 = 4(264) − 5(4) = 1036
(q2)2 − 2q2 − 8 = 0 ∴ Choice (C) is not a perfect square.
(q2 − 4) (q2 + 2) = 0 ⇒ q2 > 0 Choice (D) follows.
∴ q2 = 4 ∴q = ±2 Choice (D)
∴ Choice (C) is true.
Choice (D) px + px
7. a(x, x) = = px
2
2
4. 9 ↑ 12 = (9) (12) = 72
3 px − px
b(x, x) = =0
72 ↓ 2 =
(3) (72) = 108 2
2 px
c(a (x, x), b(x, x) = logp =0
108 → 3 = 3(108) + 4(3) = 336 px
336 ← 1 = 4(336) − 5(1) = 1339. Ans: (1339)
Choice (B)
nd
Triumphant Institute of Management Education Pvt. Ltd. (T.I.M.E.) HO: 95B, 2 Floor, Siddamsetty Complex, Secunderabad – 500 003.
Tel : 040–27898195 Fax : 040–27847334 email : info@time4education.com website : www.time4education.com SM1001962/65
x+y p5 + p3q3 − p2q2 − q5 = p5 − p3q3 + p2q2 – q5
8. c(x, y) = logp 2p2q2 (pq − 1) = 0
x−y
⇒ p = 0, q = 0 or pq = 1
x−y ∴ pq = 0 or 1.
c(x, −y) = logp
x+y Choice (D) is true. Choice (D)
 x+y x−y
Numerator = a  log , log  12. c(p, p) = 448
 x−y x + y  p3 − p2 = 448 = 8(56)
x+y x−y ⇒ p2(p − 1) = 82(8 − 1)
= + Comparing the two sides, 8 is a possible value for p.
x−y x+y Choice (D)
= 2(x2 + y2) /x2– y2
x+y x−y 4 xy 13. 45 + 90 ÷ 45 × 5 − 6
Similarly, the denominator = − =
x − y x + y x2 − y2 = 45 + 2 × 5 − 6
= 45 + 10 − 6
x2 + y2 = 55 − 6 = 49
∴ The given expression is Choice (A) (performing the order of operations is BODMAS).
2xy
Ans: (49)

p8 + p6 Solutions for questions 14 to 16:


9. a(8, 6) =
2
p8 − p6 Let i be an integer, f be a proper fraction (i.e., 0 ≤ f < 1) and r
b(8, 6)= be any real number. We have the following basic results,
2 following from the definition of [x] and (x).
c(a(8, 6), b(8, 6) = (I) [i] = i (V) (i + f)] = i +1
p8 + p6 p8 − p6 (II) [i + f ] = i (VI) [i + r] = i + (r)
+ (III) [i + r] = i + [r] (VII) [r] + 1 = (r)
logp 2 2 (IV) (i) = i + 1
p +p
8 6  p8 − p6 
− 
2  2  14. Let A = [A] +a
  B = [B] + b
p8 C = [C] + c
= logp =2 D = [D] + d
p6 0 ≤ a + b + c+ d = s < 4 . . . (1) and
3 A + B + C + D = [A] + [B] +[C] + [D] +s
c(c(a(8, 6), b(8, 6), 1) = logp = logp3 Choice (A) ∴ [A] + [B] + [C] + k
1 Where k = 0, 1, 2 or 3 (from (1))
∴ 4 is not a possible value for k. Choice (D)
10. a(2, 1) = 22 + 13 = 5
c(2, 1) = 23 − 12 = 7 15. Let A = [A] +a
b(2, 1) = 23 + 12 = 9 B = [B] + b
d(2, 1) = 22 – 13 = 3 C = [C] + c
5+7 ∴0 ≤ a+b+c=s<3 → (1)
The required value = =2 Ans: (2)
9−3 (A + B + C) = ([A] + [B] +[C] + s)
= [A] + [B] + [C] + (s)
11. Choice (A) = [A] + [B] + [C] + k → (2)
a(p, q) = p2 + q3 Where k = 1, 2 or 3 (from (1))
Also (A) + (B) + (C)
d(p, q) = p2 − q3
= [A] + [B] + [C] + 3 → (3)
Given p2 + q3 − (p2 − q3) = 128
Comparing (2) and (3), l can be 0, –1 or –2. Among the
⇒ q3 = 64 ⇒ q = 4
options, only –1 is there. Choice (D)
∴ Choice (A) is false

Choice (B) 16. From the basic result (VII), m = 1.


Choice (D)

p + p
3
( ) 2 2
p (1 + p )
3
= 4 ⇒ 1 + p = 4 – 4p
( )
= 17. Since all the numbers provided are even and LCM, HCF or
p3 − p2
2
p 3 (1 − p) the Arithmetic mean of two even numbers is even, the
given question reduces to π(even, 2) = 0
⇒ p = 3/5 Ans: (0)
∴ Choice (B) is false.
Choice (C) 18. GOS (a, b, c) = 3
a.b.c

( )= p
a p 2 ,p 4
+ p3 p +1 GOSS (a, b, c) =
3
a2.b2.c 2
d(p , p ) p
= =3
2 4
−p 3 p −1 3
GOC (a, b, c) = a 3 .b 3 .c 3
4 For a, b, c > 1; a b c ≥ a2b2c2 ≥ abc
3 3 3
⇒ Using, componendo/dividendo p = =2
2 3 3 3
⇒ a3b3c 3 ≥ a2b2c 2 ≥ abc
∴ Choice (C) is false.
i.e., GOC (a, b, c) ≥ GOSS (a, b, c) ≥ GOS (a, b, c).
Choice (D) Choices (A), (B) and (C) are clearly true.
3
p2 + q3 p3 + q2 While choice (D) i.e., a2.b2.c 2 ≥ abc is not true.
=
p −q2 3
p −q
3 2 Choice (D)
nd
Triumphant Institute of Management Education Pvt. Ltd. (T.I.M.E.) HO: 95B, 2 Floor, Siddamsetty Complex, Secunderabad – 500 003.
Tel : 040–27898195 Fax : 040–27847334 email : info@time4education.com website : www.time4education.com SM1001962/66
19. Consider choice (A): 24. a(−x, x) = 3−x + x = 1
If one of a, b is negative a ∨ b is negative. b(−x, −x) = 3−x − (−x) = 1
Since a × b is negative while ea × eb is not. 1
Hence choice (A) is false. d(1, 1) = log3 =0 Ans: (0)
1
Consider choice (B):
a ∧ b = ea eb can be less than 1 when both a, b are 25. a(2, 1) = 32+1 = 33
negative. b(3, 2) = 33−2 = 31
Hence choice (B) is false. c (33, 31) = log333.31 = log334 = 4 Ans: (4)
Consider choice (C)
a ∨ b ≠ a ∧ b for a = b 26. a(3, 4) = 33+4 = 37 and b (5, 2) =35−2 = 33
c[a(3, 4), b(5, 2)]
Choice (C) is true only when a = b = 1 and not for other values.
∴Choice (C) is false
c(37 ,33 )
d[a(3, 4), b(5, 2))]
=
Now as ea and eb are always positive, a ∧ b = eaeb is d(37 ,33 )
always positive. Choice (D)
=
( ) ( ) = 10 = 5
log 3 37 33
Ans: (2.5)
Solutions for questions 20 to 23: 37 4 2
log 3
33
P>Q>R
a(P, Q, R) = min(P, Q, P) = Q
b(P, Q, R) = max(Q, Q, P) = P  XY 
27. 8(X # Y) = 8   = 2XY
c(P, Q, R) = max(Q, R, R) = Q  4 
d(P, Q, R) = min(Q, R, P) = R
e(P, Q, R) = min(P, Q, R) = R Given: 8(X # Y) = x ⊕ y
f(P, Q, R) = min(Q, R, R) = R ∴ 2XY = X + Y + XY
XY − X − Y = 0
20. Choice (A) XY − X − Y + 1 = 1
a(P, Q, R ) + c (P, Q, R ) 2Q Q X(Y − 1) − 1(Y − 1) = 1
= (X − 1) (Y − 1) = 1
d(P, Q, R ) + e(P, Q, R ) 2R R
= which exceeds 1.
X and Y are integers
∴ Choice (A) exceeds 1. ∴ X − 1 and Y − 1 are factors of 1.
Choice (B) ∴ X − 1 = Y − 1 = ±1
a(P, Q, R ) − e(P, Q, R ) Q − R  Q  ∴ X = Y = 0 or 2
= =< 1 Q , <1 ∴ (X, Y) = (0, 0) or (2, 2)
b(P, Q, R ) − f (P, Q, R ) P − R  P  The number of values of (X, Y) is 2. Ans: (2)
Choice (C)
e(P, Q, R) + f(P, Q, R) = 2R 28. 1 ⊕ 2 = 1 + 2 + 1(2) = 5
a(P, Q, R) + c(P, Q, R) = 2Q (5) (4 ) = 5
a(P, Q, R ) + c (P, Q, R ) Q
5#4=
4
= , which exceeds 1.
e(P, Q, R ) + f (P, Q, R ) R 5 ⊕ 3 = 5 + 3 + (5) (3) = 23 Ans: (23)
∴ Choice (C) exceeds 1
29. p = 1, q = 1, r = 2, s = 3 and t = 4
Choice (D) follows. Choice (D)
⊕= 2 2 − (1) ( 4) = 0
21. The expression which is undefined must have a
denominator of 0. ⊝= 3 2 − (1) ( 4) = 5
In choice (D), Choice (A)
d(P, Q, R) – e(P, Q, R) = 0 ⊕2 > ⊝2 Choice (A) is false.
∴ Denominator = 0 Choice (B)
∴ Choice (D) is undefined. Choice (D) ⊕⊝>0
22. Each of the first three choices is equal to 1. ∴ Choice (B) is false.
Choice (D)
Choice (C)
23. Choice (A) ⊕ < ⊝ ∴ Choice (C) is true. Choice (C)
P−Q
Given expression is , which is
Q−R 30. ⊕ = a 2 − ac and ⊝ = b 2 − bc
positive
= positive. Choice (A) ⊕ = ⊝
positive Taking the square for both the expressions, we get
a2 − ac = b2 − bc
Choice (B) ⇒ a2 − b2 = c(a − b) ; ⇒ (a − b) [c − (a + b)] = 0
Q −P P −Q ⇒ a − b = 0 or c − (a + b) = 0 ; a = b or c = a + b,
Given expression is = , which is positive.
R −Q Q −R which is > 0 since a, b > 0
∴ Choice (A) is not necessarily true.
Choice (C)
Q − P (P − Q ) Choice (B)
Given expression is = , which is negative. ⊕2 = a2 − ac > 0
Q−R Q −R
a (a − c) > 0
Choice (D) ∴ a − c > 0, i.e. a > 0
Q−R ∴ Choice (B) is true.
Given expression is = 1 which is positive.
Q−R Choice (C)
Choice (C)
⊝2 = b2 − bc < 0

Triumphant Institute of Management Education Pvt. Ltd. (T.I.M.E.) HO: 95B, 2nd Floor, Siddamsetty Complex, Secunderabad – 500 003.
Tel : 040–27898195 Fax : 040–27847334 email : info@time4education.com website : www.time4education.com SM1001962/67
b(b − c) < 0 16. The geometric mean of a and b is ab .
As b > 0, ∴ b − c < 0
∴ Choice (C) is not true. Choice (B) Here a = 12 and b = 3. = (12) (3) = 6
Ans: (6)
Chapter – 9
(Statistics) n
17. The harmonic mean of x1, x2, . . . . . xn is
1 1 1
Concept Review + + ..... +
x1 x 2 xn
Solutions for questions 1 to 30: here n = 4
1 1 1 1
1. Individual or raw data Choice (C) ∴ The harmonic mean of , , , is
2 4 6 8
2. Grouped data Choice (B) 4 4
= = 0.2
2 +4+6+8 20
3. Size of the class = difference between the lower (or upper)
limits of two successive classes, Ans: (0.2)
i.e. 19 – 9 = 10 Choice (A)
18. The harmonic mean of 3 and 5 is
4. Mid value = Average of the limits of a class. 2ab
Harmonic mean of a, b is
14.5 + 25.5 a+b
= = 20 Ans: (20)
2 2 2.5.3
= = 3.75 Choice (B)
1 +1 5+3
Sum of scores 3 5
5. Arithmetic mean =
number of scores
19. The relation between arithmetic mean (A, M), geometric
12 + 17 + 15 + 21+ 36 + 40 mean (GM) and harmonic mean (H. M) is
=
6 A.M ≥ G.M ≥ H.M (Standard result)
Choice (A)
141
= = 23.5 Choice (D)
6 20. We know that
G2 = AH
6. Arithmetic mean of an arithmetic progression (6)2 = 12H or H = 3 Choice (C)
first term + last term
= 21. The empirical relation between arithmetic mean, median
2
and mode is
∴ Arithmetic mean of 8, 15, 22, 29, 36, 43, 50, is
Mode = 3 Median − 2 Mean
8 + 50 24 = 3 Median − 2 (26)
= = 29 Choice (B)
2 24 + 52 76
Median = =
3 3
7. Given arithmetic mean of x1, x2, . . . . . xn is 50. Then the
Choice (D)
arithmetic mean of x1 − 10, x2 − 10, . . . . . xn − 10 is 50 − 10
= 40 Ans: (40)
22. For a symmetric distribution,
Mean = Median = Mode Choice (D)
8. The arithmetic mean of x1, x2 . . . . . x50 is k.
Than the arithmetic mean of cx1 + 8, cx2 + 8, . . . . , cx50 + 8 23. For a moderately symmetric distribution we have
is ck + 8 Choice (C)
Mode = 3 Median − 2 Mean
9. The sum of deviations about mean is always equal to zero Mode − Median = 2 (Median − Mean)
Choice (D) 24 = 2 (Median − Mean)
24
∴ Median − Mean = = 12
10. The middle observation of the first 49 natural numbers is 25. 2
∴ Median = 25 Ans: (25) Ans: (12)
11. When a, b, c are in arithmetic progression, then b is the 24. Range = Maximum − Minimum
arithmetic mean of a and c Here the maximum = 83 and the minimum = 19
∴ Arithmetic mean of x1, x3 is x2 Choice (A) ∴ Range = 83 − 19 = 64
Choice (B)
12. In the given data, the most frequently occurring value
= 3 irrespective of the value of x. 25. If a constant is added to every observation, then the range
∴ Mode = 3 Choice (B) is unaltered.
∴ Range = 28
13. The number of observations is seven (odd) Ans: (28)
∴ when arranged in order we have 6, 8, 10, 12, 14, 16
(i) if 10 < x < 12,the median is x 26. The given data, arranged in order is 6, 8, 12, 14, 17, 21, 26.
(ii) if x < 10, the median is 10 th
(iii) if x > 12, the median is 12  n + 1
If there are 4n – 1 observations in the data, the  
∴ y ∈ [10, 12]  4 
or 10 ≤ y ≤ 12 Choice (B) observation is Q1
th
14. In the given data, 3 and 5 occur the greatest number of  7 +1
i.e.,   = 2nd item i.e., 8
times ∴ This data has two modes, 3 and 5.
 4 
Such data is called bimodal data. Choice (D)
∴ Q1 = 8
15. Bimodal data Choice (B) Choice (B)
nd
Triumphant Institute of Management Education Pvt. Ltd. (T.I.M.E.) HO: 95B, 2 Floor, Siddamsetty Complex, Secunderabad – 500 003.
Tel : 040–27898195 Fax : 040–27847334 email : info@time4education.com website : www.time4education.com SM1001962/68
27. The given data arranged in ascending order is 14, 21, 23, n
26, 29, 30, 38, 42, 47, 56, 72. If there are 4n – 1 6. HM (x1, x2, x3, . . . . , xn) =
1 1 1
th + + .... +
 n + 1 x1 x 2 xn
observations, the 3   observation of the data is Q3,
 4  5 5 1
HM (1, 1/4, 1/7, 1/8, 1/10) = = = .
 11+ 1  1 + 4 + 7 + 8 + 10 30 6
the 3rd quartile is the 3   or the 9 observation.
th

 4  Choice (C)
∴ Q3 = 47 Ans: (47)
7. G.M. (G.P.) = first term × last term
a −b G.M. (31, 32, 33, . . . . . .399)
28. Mean deviation of two numbers a, b is
2 = 3 × 399 = 350 = 925. Choice (C)
36 − 24
∴ Mean deviation of 24 and 36 is =6
2 8. Arranging the numbers in ascending order we have,
Ans: (6) 15.2413, 15.3125, 15.3142, 15.3143, 15.3215, 15.4132,
th
 7 + 1
29. If σ is the standard deviation (S.D) of x1, x2, . . . . . xn then 15.5213. Since there are 7 terms, the median is  
S.D of cx1 + p, cx2 + p, . . . . , cxn + p (where c, p are real  2 
numbers) is given by c σ term i.e., 4th term.
∴ Required S. D is 3 σ Choice (D) ∴ median = 15.3143. Ans: (15.3143)

30. The given observations are arranged in ascending order, 4, 9. Arranging the given elements in order, we have 4, 6, 8, 14,
8, 12, 16, 20, 24, 28, which is an arithmetic progression 15, 16, 20, 22
n2 − 1 Since the number of elements is 8, the median is the
∴ Standard deviation = c.d , th th
12 8 8 
average of   and  + 1 elements
where c.d is common difference = 4 2 2 
and, n is number of observation p = 7
14 + 15
∴ Median = = 14.5 Choice (C)
72 −1 2
∴ S. D = 4 = 8. Choice (C)
12
th
 17 + 1 
Exercise – 9(a) 10. We need to know the   prime or the 9th prime i.e.,
 2 
Solutions for questions 1 to 25: 23. In fact, we need not list all the 17 primes.
Choice (D)
1. The sum of the squares of first 'n' natural numbers is
n(n + 1) (2n + 1)/6 11. As the two numbers included into the series flank the
If n = 9, we get the sum as median (M), the position of the median is not affected.
9(9 + 1) (2(9) + 1)/6 = 285 Hence the median remains the same. Choice (B)
We deduct 12 + 22 + 32 from 285 to get 271
We are given 6 numbers. The mean of the required 12. We first plot the known numbers on the number line as follows.
numbers is 271/6 = 45.16. Choice (D)

2. The numbers are in geometric progression. The sum of


these numbers is 5 7 12 14 16 18
46 − 1 4095 Now if x ≤ 12 then 12 would be the median.
4⋅ =4⋅ = 5460
4 −1 3 If x ≥ 14 then 14 would be the median.
The arithmetic mean is 5460/6 = 910 Ans: (910) If 12 ≤ x ≤ 14 then x itself would be the median.
So 12 ≤ median ≤ 14. Choice (B)
3. The numbers form an arithmetic progression. Hence the
arithmetic mean is (9 + 105)/2 = 57. 13. 10 has occurred for a maximum number of times (4 times).
Note: AM (A.P) = AM (the first and the last term) Inspite of x being an unknown, 10 is still the most found
Choice (C) number. Choice (B)

4. x = Σx/n 14. Mode = 3 Median – 2 Mean empirical formula:


So Σx = n x Mode = 3(4.5) – 2(1.25) = 13.5 – 2.5 = 11 Ans: (11)
The sum of the numbers is 50 × 42 = 2100
On subtracting 75 and 105 from 2100, 15. On adding or subtracting a constant, range does not
we get 2100 – (105 + 75) = 1920. change. Hence range is r. Choice (D)
So, the sum of the remaining set of 48 observations is 1920.
Hence the mean of the remaining set is 1920/48 = 40.
16. Here n = 7. Arranging the data in ascending order, we have
Choice (A)
5, 9, 10, 13, 15, 16, 20.
th
5. Given n1 = 7, x1 = 36; n2 = 13, x 2 = 46.  n + 1
Now, Q1 =   term = 2nd term = 9
 4 
n1 x1 + n2 x 2 7 × 36 + 13 × 46
Combined mean x c = = th
n1 + n2 7 + 13  n + 1
and Q3 = 3  term = 6th term = 16
850  4 
= = 42.5
20 Q3 − Q1 16 − 9
∴ Q.D. = = = 3.5. Ans: (3.5)
Choice (B) 2 2

Triumphant Institute of Management Education Pvt. Ltd. (T.I.M.E.) HO: 95B, 2nd Floor, Siddamsetty Complex, Secunderabad – 500 003.
Tel : 040–27898195 Fax : 040–27847334 email : info@time4education.com website : www.time4education.com SM1001962/69
17. The data given 1, 2, 5, 7, 11, 13, 16, 17 Exercise – 9(b)
1 + 2 + 5 + 7 + 11 + 13 + 16 + 17
Arithmetic Mean = =9 Solutions for questions 1 to 25:
8
xi − x 1 − 9 + 2 − 9 + . . . . . + 17 − 9 1. The sum of the first 80 even natural numbers = 2 + 4 + 6 + 8
Mean deviation = =
+ . . . . . + 160 = 2(1+ 2 + 3 + 4 +. . . . + 80) =
(2)(80)(81) .
8 8
2
8+7+ 4+2+2+ 4+7+8 42
= = = 5.25 The arithmetic mean of the first 80 even natural numbers
8 8
=
(2)(80)(81) = 81 Ans: (81)
Choice (D)
2 × 80
a+b
18. The arithmetic mean of a and b is 2. The sum of the first x odd natural numbers = xy. The pth
2
even natural number is 1 more than the pth odd natural

S.D =
(a − (a + b) / 2)2 + (b − (a + b) / 2)2 number.
∴ The sum of the first x even natural numbers = xy + x(1)
2 = x(y + 1)
∴ The arithmetic mean of the first x even natural numbers
S.D = (a − b)2 / 4 = a − b . As a > b, a – b = a – b x(y + 1)
2 = = y +1 Choice (B)
Thus SD = (a – b)/2 Choice (D) x

19. Let the numbers be x, x + 1, x + 2, x + 3, x + 4, x + 5, x + 6. 3. The harmonic mean of 1, 3, 6, 8, 16, and 32


Here n = 7 6 192
This being an arithmetic progression with common = = Choice (C)
1 1 1 1 1 1 55
n2 − 1 72 − 1 + + + + +
difference 1. S.D = d =1 =2 1 3 6 8 16 32
12 12

[ ( ) ( )]
1
⇒ Variance = (S.D.)2 = 22 = 4
Geometric mean = (1)(6) 62 .... 651
Choice (A)
4. 52
20. We subtract 18 from each number. The series reduces to 0,
[6 ]
1
0, 0, 0, 0,0, 0, 0, 0, 5 1+ 2 + 3...... + 51 52
1/ 2
 ∑ x2 2
 n − (x ) 
1
S.D =   = (25/10 – (5/10)2)1/2  (51)(52 )  52 51
  6 2  =62 Choice (B)
= (2.5 – 0.25) 1/2
= 1.5 Choice (D)  
 
21. Since the measures of deviation are not affected by origin
change, S.D. (axi + b) = a × SD (xi) 5. Mean of the given numbers
⇒ SD (3xi + 2) = 3SD(xi) = 15 Ans: (15) 16 + 4 + 10 + 18 + 30 + 14 + 2 + 24 + 26
= = 16
9
 n(n + 1) 
2
Mean deviation
22. 3
∑ n =   = 3025 (given)
 2   16 − 16 + 4 − 16 + 10 − 16 + 18 − 16 + 
 
n(n + 1)  30 − 16 + 14 − 16 + 2 − 16 + 24 − 16 + 26 − 16 
⇒ = 55 ; ⇒ n(n + 1) = 110, so, n = 10  
2 =
9
Now Σn /n = 3025/10 = 302.5
3
Choice (D)
68
23. Σ(x – 7) = 40 = Choice (A)
9
⇒ Σx – Σ7 = 40
Dividing throughout by 10, we get Σx/10 – Σ7/10 = 40/10 6. Wrong total of the numbers = (10) (20) = 200. Correct total
x – 7 = 4 ; x = 11 Ans: (11) of the numbers = 200 + 14 + 12 + 16–(6 + 8 + 10) = 218.
24. For two positive integers x and y 218
Actual mean = = 21.8 Ans: (21.8)
10
G= xy
H = (2xy) / (x + y) ; A = (x + y)/2 7. The average wage of all the employees is
Now, G2 = xy. (60 )(4000 ) + (20 )(5000 ) = `4250 Choice (D)
x+y 2xy
and AH = × = xy . 80
2 x+y
Hence G2 = AH. Choice (A) 6 + 6 + 6 + 6 + 10 + 10 + 8 + 8 + 12
8. Arithmetic mean = =8
9
25. AM (a, b) = (a + b)/2
Case (i) x ≥ 0 ∑(xi − x )
2
a = max (x, – x) = x Standard deviation (S.D) =
b = min (x, – x) = –x
n
a + b (x ) + (− x )
= 4(− 8 + 6 ) + 2(− 8 + 10 ) + 2(− 8 + 8 ) + (− 8 + 12 ) = 40
2 2 2 2
So, = =0
2 2 9 3
Case (ii) x < 0 Choice (D)
a = max (x, – x) = –x ⇒ b = min (x, – x) = x
a + b (− x ) + x 9. The first 15 prime numbers are 2, 3, 5, 7, 11, 13, 17, 19,
So, = =0 23, 29, 31, 37, 41, 43 and 47.
2 2 Their median = Middle prime number = 8th prime number 19.
Thus A.M (a, b) = 0 Ans: (0) Ans: (19)

Triumphant Institute of Management Education Pvt. Ltd. (T.I.M.E.) HO: 95B, 2nd Floor, Siddamsetty Complex, Secunderabad – 500 003.
Tel : 040–27898195 Fax : 040–27847334 email : info@time4education.com website : www.time4education.com SM1001962/70
10. AM (ax1 + b1, ax2 + b2 . . . axn + b) = a(AM(x1, x2 . . xn)+ b. Y1 + Y2 + Y3 + Y4
Given AM(y1, y2, y3, . . . yn) = A 20. Z1 =
4
 3y1 + 2 3y2 + 2 3yn + 2  3A + 2
∴ AM  , ... = Y2 + Y3 + Y4 + Y5
 4 4 4  4 Z2=
4
Choice (A)
Y3 + Y4 + Y5 + Y6
Z3 = .....
11. Sum of all the observations = (25) (24) = 600. 4
Arithmetic mean of the remaining numbers Y + Yn −1 + Yn + Y1
Zn–2 = n − 2
600 − 35 − 36 4
= = 23 Choice (B)
23 Yn −1 + Yn + Y1 + Y2
Zn–1 =
4
12. Range = Maximum observation – Minimum observation = 60
Yn + Y1 + Y2 + Y3
3 Maximum observation – 3 minimum observation Zn =
= 3(60) = 180. Ans: 180 4
It would be seen that each Yi where 1 ≤ i ≤ n occurs exactly
13. We know that the median of (y1, y2 + k, y3 + k, . . . yn + k) in four of the Zi s.
= the median (y1, y2, . . yn) + k ∴ arithmetic mean of Z1,Z2, . . . Zn
∴ Median of the new set = 60 – 3 = 57 4(Y1 + Y2 + .... + . Yn )
Choice (A) = =A Ans: 1
4n
14. There are 11 terms arranged in an ascending order 21. The sum of the cubes of the first n even natural numbers
th
 11 + 1   n(n + 1) 
2
Q1 =   term = 3rd term = 7. = 8 
 4   2 
th
 11 + 1   n(n + 1) 
2
Q3 = 3   term = 9th term = 29.  
 4  2 
Their arithmetic mean = 8  = 2n(n + 1)2
Q3 − Q1 29 − 7 n
The Quartile Deviation (Q.D) = = = 11
2 2 Choice (B)
Ans: (11)
Y1 + Y2 + .... + Yn
2(5) + 3(4) + 4(6) 46
22. =M
15. Arithmetic mean = = n
2+3+ 4 9 Y1 + Y2 + . . . Yn –2 + Yn –1 Yn = Mn
Choice (D) Y1 + Y2 +. . . Yn – 2 + Yn = Mn – Yn – 1
Y1 + Y2 + . . . .Yn – 2 + Yr + Yn = Mn – Yn – 1 + Yr
∴ the arithmetic mean of the new series will be
16. 8 occurs the maximum number of times in the data.
∴ mode = 8. Ans: (8) Mn − Yn −1 + Yr
Choice (D)
n
17. The least multiple of 8 greater than 100 = 104 = (13) (8)
The greatest multiple of 8 less than 200 = 192 = (24) (8). x1 + x2 + ..... xn
There are 12 multiples of 8 between 100 and 200. 23. =x
n
12
∴ their sum = [104 + 192] ⇒ x1 + x2 + . . . + xn = nx . . . (1)
2
y1 + y2 + ..... + yn
[104 + 192]
12 =y
n
∴ their arithmetic mean = 2 = 148 ⇒ y1 + y2 + . . . . + yn = n y . . . . (2)
12 Subtracting (2) from (1),
Choice (C) (x1 – y1) + (x2 – y2) + . . . . . + (xn – yn) = n(x – y)

18. If each observation of a series is divided by k their standard x1 − y1 + x 2 − y 2 + ..... + x n − y n


deviation will also be divided by k. ∴ the standard deviation Required mean =
n
would be divided by 3.
σ n(x − y )
= =x–y Choice (A)
∴ Hence the standard deviation of the new series is . n
3
Choice (B)
24. Sum of all the observations = (150) (30)
19. Let the side of ABCD be x pm. Sum of 100 of them = (100) (30)
Sum of the remaining = (150 – 100) (30)
x
Time taken by Raju to travel AB =
S1
hours
Arithmetic mean of the remaining =
(150 − 100 )(30 ) = 30
x 50
Time taken by Raju to travel BC = hours. Ans: 30
S2
2x 2S1S2 25. Given, n = 11; ∑(xi – x)2 = 110
Average speed = = .
x
+
x
S1 S2
S1 + S2
Standard deviation =
(
∑ xi − x )
2
=
110
= 10
n 11
which is the harmonic mean of S1 and S2
Choice (C) Choice (B)

Triumphant Institute of Management Education Pvt. Ltd. (T.I.M.E.) HO: 95B, 2nd Floor, Siddamsetty Complex, Secunderabad – 500 003.
Tel : 040–27898195 Fax : 040–27847334 email : info@time4education.com website : www.time4education.com SM1001962/71

You might also like